q_id
string | title
string | selftext
string | document
string | subreddit
string | url
string | answers
dict | title_urls
list | selftext_urls
list | answers_urls
list |
---|---|---|---|---|---|---|---|---|---|
2v4zxo | Why do quasars emit so much light? | askscience | http://www.reddit.com/r/askscience/comments/2v4zxo/why_do_quasars_emit_so_much_light/ | {
"a_id": [
"coekqep"
],
"score": [
4
],
"text": [
"Essentially, because there is a lot of material falling into the black hole. When things fall down a gravity well, they lose gravitational potential energy. Drop something off a cliff, and gravitational energy gets transformed into kinetic energy, which upon hitting the ground will partly turn into thermal energy. Of course, if you drop something off a cliff, then even ignoring air resistance it will probably only be going tens of meters per second. If you drop something into a black hole from very far away, then as it approaches the event horizon, its speed will approach the speed of light. Now, typically things don't just fall in on a single trajectory, it's usually a longer process of orbital decay and tidal forces that eventually usher the particles to their ultimate fate, but nevertheless they gain a tremendous amount of kinetic energy, and much of that gets transformed into radiation. That's why quasars are so bright.\n\nThe specifics of how energy gets turned into light (and particle jets) in an Active Galactic Nucleus is more complicated. First of all, you've got a rapidly spinning [accretion disk](_URL_0_) of matter around the black hole. As you get closer to the inner edge of this disk, it gets hotter and denser, so that the inner edge is radiating mainly in the X-ray regime. Since this material is a plasma, you've got magnetohydrodynamical effects that come into play. That's a fancy term for a combination of fluid dynamics and electromagnetism. We don't yet have a precise understanding of the process, but basically the disk creates powerful electromagnetic fields which shoot some of the infalling particles out in jets rather than letting them fall toward the event horizon."
]
} | []
| []
| [
[
"http://en.wikipedia.org/wiki/Accretion_disc"
]
]
|
||
4970ho | why is it that "time flies when you're having fun"? | Why is it that we experience time going faster while other times it feels so slow? What are some psychologist / neuroscientist explanations of our perception of time?
I would also appreciate paper recommendations so I can read more about it.
Thank you!
| explainlikeimfive | https://www.reddit.com/r/explainlikeimfive/comments/4970ho/eli5_why_is_it_that_time_flies_when_youre_having/ | {
"a_id": [
"d0pis0y",
"d0plakr"
],
"score": [
4,
3
],
"text": [
"I think because you focus much harder on the task at hand you stop thinking about the time so much.",
"The psychology term you are looking for is [flow](_URL_0_\n \nWhen you are engaged in something and our attendion is focused on it. Our perception of the passage of time is altered. "
]
} | []
| []
| [
[],
[
"https://en.m.wikipedia.org/wiki/Flow_(psychology)"
]
]
|
|
ci65v2 | what are the grey areas that we can see in the moon from down here? | explainlikeimfive | https://www.reddit.com/r/explainlikeimfive/comments/ci65v2/eli5_what_are_the_grey_areas_that_we_can_see_in/ | {
"a_id": [
"ev1smdl",
"ev395j1"
],
"score": [
3,
2
],
"text": [
"They're called mares. It means \"sea,\" but these are actually more like patches of the surface that had melted and re-hardened in the distant past.",
"They're called maria. They're large, flat basaltic lava planes from from when the moon was geologically active and had volcanoes."
]
} | []
| []
| [
[],
[]
]
|
||
5uezov | since blue whales have aortas large enough to swim in, do they have fewer issues with blood clots? | explainlikeimfive | https://www.reddit.com/r/explainlikeimfive/comments/5uezov/eli5_since_blue_whales_have_aortas_large_enough/ | {
"a_id": [
"ddtgkpy",
"ddtj0tn",
"ddtto9y",
"dduh6wu"
],
"score": [
5,
64,
12,
2
],
"text": [
"Or do they just have man-sized blood clots?",
"considering they have an extremely active lifestyle (swimming constantly all the time) and a relatively low cholesterol diet (mostly plankton and krill iirc) they are presumably at very low risk for blood clots in the first place. ",
"Also, humans don't tend to develop blood clots in our aortas. The aorta is susceptible to other things, like aneurysms, dissections and ruptures, while blood clots are typically found in much smaller arteries, for example the cerebral arteries or the cornary arteries.\n\nSource: me, I'm not a doctor.",
"They do have large vessels, but they also have small vessels. Not every artery in a blue whale is the size of their aorta, and those are the vessels that would be vulnerable to clots. \n\nThey would likely have decreased clot risk because they are less likely to have hemostasis (where your blood flow is poor or pools). "
]
} | []
| []
| [
[],
[],
[],
[]
]
|
||
c8j7cp | When heating something to high temperatures it becomes “red hot” and then continues to change color to orange, yellow, and then white; why doesn’t the hot object start to glow green or blue after yellow? | askscience | https://www.reddit.com/r/askscience/comments/c8j7cp/when_heating_something_to_high_temperatures_it/ | {
"a_id": [
"esnl5r2",
"esnpl0p",
"esnrs57",
"esnt994",
"esnyeej"
],
"score": [
53,
5,
4,
3,
14
],
"text": [
"The \"red hot\" appearance of an object is due to its thermal radiation. This is EM radiation emitted by every object and the spectrum of which is determined by the temperature. Objects at room temperature emit thermal radiation in the infrared part of the spectrum, but once objects get hotter, the spectrum of thermal radiation shifts more towards the visible part.\n\nHowever, this radiation is a broader spectrum, not a single line. So a red hot object will emit red visible light along with plenty of IR light. Since the red light is the only thing visible to us, we see the object as red. If you heat up the object further, shorter wavelength visible light gets added to the mixture, making the combined color shift towards orange.\n\nIf you heat the object enough, the thermal radiation will cover the entire visible light part of the spectrum, making the combined radiation appear white. At no point does the object only emit green or blue light. By the time it's hot enough to emit green or blue, it's also emitting plenty of light in other colors (with longer wavelengths) and what we see is the mixture of all visible light that is emitted.",
"Electric arc welders and blue giant stars like Sirius are hot enough to glow blackbody blue.\n\nHeat is disordered motion. Moving charged particles emit light. Vibrate matter faster, recieve light with shorter wavelengths. This is blackbody radiation, and this is why hot iron glows red then yellow then white and would eventually glow blue if it didn't boil first.\n\nOur sun is green. We are unaware of that fact because its blackbody emissions peak in the green part of the rainbow, but the heat also makes enough red and blue light to balance that out to human vision. An object with its emissions centered on blue must be significantly hotter than the sun. See [Wein's displacement law](_URL_1_) and [color temperature.](_URL_0_)\n\nIf something is blue and hot there's a good chance it's a UV hazard (even shorter wavelengths than blue) and you should not stare at it up close without goggles. Very hot things can glow mostly in x-rays (shorter than UV...) with spillover into UV and visible, but an exploding nuclear bomb still looks blue-white. Nothing will be a really pure blue because if matter is hot enough to glow blue it's also hot enough to make lesser amounts of the other colors, and a blackbody is still blue-white near infinite temperatures.",
"The spectrum of a hot body forms a _hump_ in the electromagnetic spectrum. This hump is (as a function of _wavenumber_ - the number of radians of wavemotion per metre (in the region of about 12 million per metre for visible light)) _decaying exponential_ on the _blue_ (short-wavelength or high wavenumber) side, and a _cube_ function on the red (long-wavelength or low wavenumber) side. As the temperature increases, the hump migrates _up_ the wavenumber scale ... _and_ becomes more spread-out at the same time. At low temoeratures, the bulk of the hump lies below the window of visibility ... but at about 1000 K or 700°C it starts to cross it. At this stage, because it is very steep & exponential on its blue side (which is the first part to begin to cross the window of visibility) it is _very much_ more intense at the red end than at the blue end, so that the light is _very predominantly red_. As the temperature rises, it comprises more & more of the window of visibility from the red end, and because its curve is steep, is much like a _sharp front_ moving across that window ... so that at first only _red_ is comprised in it; then _red & yellow_ - so that the light appears _orange_; & then green is comprised, so that the light becomes yellowish: and by the time the temperature reaches about 5000K the hump is prettymuch filling the window of visibility with the _peak of the hump right in the middle of it, so that now _all_ colours are comprised roughly equally - & the light appears _white_. \n\nAs the temperature increases _yet further_, the hump is now _moving-off the blue_ end of the visible window, and the curve is sloping the other way, so that the light appears slightly _blue_ ... but the hump is of relatively _shallow_ slope on this side, so that the light appears only _slightly_ blue: and this situation does not really change very much thereafter with rising temperature, because the curve is a shallow _cubic_ function of wavenumber all the way to zero. This time there is no 'sharp front' passing across the window of visibility (in reverse of course it _would_ be) and the proportions of the different colours change little - a slow decrease from blue to red, even as the temperature becomes _extremely_ high.",
"The phenomena behind hot objects glowing is known as blackbody radiation, and the spectrum produced is governed by Planck's Law. While Planck's Law is a mathematical description of the spectrum, a few key points can be inferred from it:\n\n* The spectrum is broad and continuous - it tapers off slowly for both shorter and longer wavelengths of the peak.\n* The wavelength of the peak is inversely proportional to the temperature (above absolute zero).\n* Green light corresponds to about 6000 K (the temperature of the sun's photosphere aka surface).\n* A hotter source is more intense at all wavelengths than a cooler source.\n\nMost objects commonly observed to glow from blackbody radiation are substantially cooler than the surface of the sun (as they would be vapourised otherwise). While this means their peak is in the infrared (i.e. is invisible), the broad spectrum means that they can still produce significant visible light in the tail of the spectrum. But as the spectrum is tapering off, there is less green light than red and less blue respectively so the glow is clearly reddish.\n\nAs the object gets hotter, it produces proportionally more green, yielding more noticably yellow or orange light, but with the peak at green wavelengths (as with the sun), the broad spectrum yields enough blue and red light for it to look white instead of green.\n\nYou can get \"bluish\" light at higher temperatures and the effect is perhaps most obvious for hot and bright stars (e.g. Rigel), and the reasoning is similar to that for red-glowing objects but with the peak being in the ultraviolet rather than the infrared, but that also comes with plenty of red light too (see the last point) so blackbody radiation will never yield a deep blue spectrum either.",
"This is how the color actually progresses as you increase temperature:\n\n|Color of Glow|What We See|\n|:-|:-|\n|IR (infrared)|= Invisible (humans can't see IR)|\n|IR + Red|= Red|\n|IR + Red + Orange|= Reddish-orange|\n|IR + Red + Orange + Yellow|= Orange|\n|IR + Red + Orange + Yellow + Green|= Yellow|\n|IR + Red + Orange + Yellow + Green + Blue|= Bluish-white|\n|IR + Red + Orange + Yellow + Green + Blue + Violet|= White|\n|IR + Red + Orange + Yellow + Green + Blue + Violet + UV|= White (humans can't see UV)|\n\nAs you can see, objects will eventually glow green. It's just that by that time, the object would also glow with other colors. These colors would mix and the end result does not look green to us.\n\nKeep in mind that this is overly simplified and the physiology of the human eyes affect things as well (this is the same reason why the sky appears blue instead of violet). Furthermore, this only applies to blackbody radiation. Some substances could glow other colors due to their emission spectrum. For example, copper would produce green flame when burnt."
]
} | []
| []
| [
[],
[
"http://hyperphysics.phy-astr.gsu.edu/hbase/phyopt/coltemp.html",
"https://en.wikipedia.org/wiki/Wien%27s_displacement_law"
],
[],
[],
[]
]
|
||
154xvd | why does fabric (temporally) change colour when it gets wet? | My backpack got wet this afternoon by some rain and the wet spots become a lot darker. I couldn't really tell why, though. Anyone who can explain this? | explainlikeimfive | http://www.reddit.com/r/explainlikeimfive/comments/154xvd/eli5_why_does_fabric_temporally_change_colour/ | {
"a_id": [
"c7jc2uf",
"c7jcf2v",
"c7jf1fu"
],
"score": [
2,
2,
2
],
"text": [
"First of all, you have to look at how colour works. When light hits something, most of the light is absorbed by whatever is there, in your case, the backpack. What you see is only the light that isn't absorbed, it's reflected back at you. So, when you add water (rain) the reflection back is different because the water has changed how the reflection works. When you see something white, you see everything reflected back, when you see something black, thats because no light is reflected back. ",
"Ever look at the wet spot on fabric from the other side? The wet spot reflects less light to you because it is transmitting some of the light through it to the other side. Just like oil making paper translucent or even transparent.",
"[Check out these threads](_URL_0_)."
]
} | []
| []
| [
[],
[],
[
"http://www.reddit.com/r/explainlikeimfive/search?q=wet+darker&restrict_sr=on&sort=relevance&t=all"
]
]
|
|
407qel | what will happen if a republican president is elected in 2016 and signs the obamacare repeal bill? | Could the Affordable Care Act actually be repealed? What would happen? | explainlikeimfive | https://www.reddit.com/r/explainlikeimfive/comments/407qel/eli5_what_will_happen_if_a_republican_president/ | {
"a_id": [
"cys2vo2"
],
"score": [
10
],
"text": [
"Yes, it could be repealed, and what would happen depends on what exactly is in the bill repealing it.\n\nWhile the Affordable Care Act is one discrete public law, it is codified by inserting provisions into various titles and sections of the U.S. Code (statutes) that deal with health care and insurance. So whatever act repeals it needs either to show what part of the current code needs to be repealed or it can repeal all of the provisions of the ACA at once and leave [someone else](_URL_0_) to do the dirty work of tracking it all down and updating the Code.\n\nThen I suppose _URL_1_ goes down, no one gets insurance subsidies anymore, you can get jacked out of qualifying for insurance based on health condition, and premiums could be underwritten again. You might have to pay for preventive care, you might have a lifetime maximum benefit again, and there's no guarantee that the plans you do qualify for will offer decent coverage. A lot of poor people would get kicked off Medicaid.\n\nIt would be like 2008 again."
]
} | []
| []
| [
[
"https://en.m.wikipedia.org/wiki/Office_of_the_Law_Revision_Counsel",
"healthcare.gov"
]
]
|
|
3e6jks | why there is no radio function in iphones (not sure about androids) radio was avaliable in almost all old phones | explainlikeimfive | http://www.reddit.com/r/explainlikeimfive/comments/3e6jks/eli5why_there_is_no_radio_function_in_iphones_not/ | {
"a_id": [
"ctby0sm"
],
"score": [
2
],
"text": [
"Because you need to add yet another chip which can receive the right wavelength, which adds complexity, costs, and takes away space for things you'd rather put in."
]
} | []
| []
| [
[]
]
|
||
8wp994 | Did any Native American civilization possibly know that there was land and people beyond the Americas? Did they have any mythological tales about the rest of the world? | AskHistorians | https://www.reddit.com/r/AskHistorians/comments/8wp994/did_any_native_american_civilization_possibly/ | {
"a_id": [
"e1ysxx6"
],
"score": [
72
],
"text": [
"I think, from my understanding, you are asking about concepts of aboriginal knowledge- this is \" including traditional teachings, empirical observation, and revelation\" (1). I caution that I have only an amateur background in the histories, but hope to answer the concept of your question.\n\nOf the concepts mentioned the most relevant, I believe, is that of *traditional knowledge* as it is the preservation of understanding handed down, \"more or less intact... And it memorizes battles, boundaries, and treaties and attitudes... Toward neighboring nations\" (1).\n\nSo where is such contact recorded? First, there is the acknowledging that such information, while flawed, is of value. This is illustrated with the oral preservation of accounts of the Franklin Expedition: \n > > The key methodological shift in Franklin expedition scholarship in recent years came with David Woodman’s Unravelling the Franklin Mystery: Inuit Testimony (1991). Woodman went back to Hall’s notes and journals from the 1860s to challenge the traditional account of the disaster and carefully piece together a narrative based on the oral testimony of Inuit. Woodman also emphasized the location of relics, bodies, and other artifacts to paint a picture of multiple abandonments of the ships and multiple groups moving in different directions, some of which returned to the Erebus and piloted it south to where it sank in the Utjulik region. This narrative would account for stories of separate groups of white men encountered by Inuit far from the Great Fish River and stories of survivors still interacting with Inuit well into the 1850s. > > ... Potter believes Inuit testimony about the expedition to be “the single most important body of evidence we have” (2).\n\nIn addition, the accuracy and usefulness of Aboriginal Songlines are also respected and well as the impact of \"continual cartographic encounters, exchanges, and translations between American Indians and Euro-Americans\"(3, 4). \n\nThis leads to... Is there evidence within traditional knowledge respecting the existence of external cultures? First, I looked to see if the Native American of the North American Content were aware of the Southern Content or vice versa - but it is important to remember the scope and newness of current geographic boundaries. For this section \"pre-contact\" is to define the physical presence of sustained non-Native populations. \n\nAccording to an *Historical Survey of First Nation Market Culture* there evidence to suggest a working from of communication across Continents for there to be the usage of non-local materials. Two notable examples are that of trade between two divisions of Mayans of the\n\n > > Northern Lowlands and the Highlands was well established via water routes on the Gulf of Mexico and the Gulf of Honduras as well as various inland routes. The Northern Lowlands Maya imported corn, fruit, cacao, various manufactured goods, flint, game, cotton, cloth, obsidian, metates, and manos. The Highland Maya imported honey, salt, ceramics, various tools, salted fish, marine shells, dye, incense, jade, and rubber.\n\n\nAdditionally the use of a soft stone (pipestone) found in the current Minnesota region as was important to the creation of objects for religious reasons was found as far as South Dakota, Ohio, and Kansas. After contact and with it the introduction of the horse, the material was found as far as Arizona and Kentucky (5).\n\nAll this contact is important as it allows for the development and subsequent preservation of intricate trade agreements. An example of this is that found in the pre-contact legislation of the Great Law of the Iroquois (now commonly referred to as Haudenosaunee). The Iroquois Confederacy had such an extensive documented sytem that it is still very much under legislative analysis. As noted in the brillant thesis *INTERNATIONAL LAW/THE GREAT LAW OF PEACE* by Beverley Jacobs, \"European diffusionism will be defined through an elaboration of brief historical accounts and an analysis of law that has affected O:gweho:we. Eurocentric values, ideals, language, laws and institutions are an integral part of diffusionism, which has tried to replace O:gweho:we history, language, religion, laws and philosophy.\"(6).\n\nNow that we have determined that there was contact, trade, and therefore knowledge across sections of the American Continents, and the risk that there is and will continue to be effects of non-Native interpretations of traditional knowledge, I attempted to source information about ex-Continental knowledge, firstly with that of the natives of Hawai'i. It might almost be cheating, in a sense, as the oral traditions of Hawai'i and New Zealand are quite well preserved compare to other Nations, and were treated with an unusual level of respect and importance to historical research. \n\n\n > > Some of the data provided by recent stratigraphic excavations in Poly- nesia is pertinent to this problem, however, for it allows some very sound inferences as to certain aspects of Polynesian voyaging. For example, as a result of intensive excavations for the last ten years, Emory believes that the Hawaiian archipelago was settled by a planned, well-equipped expedition of some size (Emory 1959). Pigs and dogs are found in the earliest sites so far discovered there, and it is believed that the domesticated plants must have also been present to support the population. > > In the Marquesas, excavations at site NHaal, the oldest known site in that archipelago to date, (ca 120 B.C.) yielded evidence that pigs and dogs accompanied the settlers. Furthermore, the presence of coconut grating tools and knives of a type generally used for peeling breadfruit and starchy root crops demonstrate that the main Polynesian staples also arrived with the settlers (7).\n\nThe trade and communication within the Polynesian region is not just known but documented (8, 9, 10). Referencing non-Polynesian sources was difficult so I plan on providing more later in the day if that is alright. \n\n--added--- \nWhile I did not want to deviate from your question, which was strictly about the North American continent, as I was limited to the papers I currently have access to. I have copied a section of a paper to illustrate part of the difficulties of studies into indigenous knowledge: \n\nAn important task of an \"Indigenous paradigm\" would be to challenge these notions according to which the world is divided along lines of Western \"high culture\" and non-Western ''folkloric'' traditions. Although many contemporary practices of poststructuralism, feminism, postmodern and postcolonial theories have undermined and rejected these assumptions, unfortunately they still guide much of people's everyday thinking and actions. Unlike many Western scholars who can ignore this since it is not as common an academic approach as it used to be, we as Indigenous peoples cannot remain indifferent, since it affects us directly in various ways through dismissive and biased attitudes on our selfhood, our culture and its products (11).\n\nAs my studies are in the field of criminology and legal development, I can say that indigenous people the world over are inheritances to some of the most historic and under studied legal systems on the planet with unique concepts of civil and criminal law. Further, the usage and impact of these legislative systems upon the cultures - and the impact this had on colonialism - is, I think, too often ignored. \n--added--\n\nTo be honest I am hesitant to touch the aspect of mythological tales - it is in no way my forte. In addition, sorry for the formatting and writing style, this was done in my phone on the train. Finally, I'm going to do my best to find more Indigenous sources as I felt there could be more I wasn't able to access on my mobile. \n\n\n(1) Indigenous Knowledges in Global Contexts: Multiple Readings of Our World edited by Budd L. Hall, George Jerry Sefa Dei, Dorothy Goldin Rosenberg\n\n(2) Finding Franklin: The Untold Story of a 165-Year Search by Russell A. Potter (review) Shane McCorristine\n\n(3) Songlines and navigation in Wardaman and other Australian Aboriginal cultures by RP Norris, BY Harney\n\n(4) Theorizing indigital geographic information networks by Mark Palmer\n\n\n(5) First Nations Trade, Specialization, and Market Institutions: A Historical Survey of First Nation Market Culture by Andre Le Dressay\n\n(6) INTERNATIONAL LAW/THE GREAT LAW OF PEACEA Thesis Submitted to the College of Graduate Studies and Research in Partial Fulfillment of the Requirement for a Masters Degreein the College of Law, University of Saskatchewan By Beverley Jacobs\n\n(7) Histmical Traditions and Archeology in Polynesia by Robert C. Suggs\n\n(8)Oral Tradition as History By Jan M. Vansina\n\n(9) Myth, Experiment, and the Reinvention of Polynesian Voyaging by Ben Finney\n\n(10) Polynesian settlement and palaeotsunamis by James Goff, Bruce G McFadgen,Catherine Chagué-Goff etc\n\n(11) TOWARDS AN \"INDIGENOUS PARADIGM\" FROM A SAMI PERSPECTIVE by Rauna Kuokkanen\n\n--added--\n\nAdditional Sources:\n\nTsilhqot’in Nation v. British Columbia (2014)\n\nHarry Daniels, et al v Her Majesty the Queen as represented by The Minister of Indian Affairs and Northern Development, et al (2016)\n\n\n\n\nEdits: Spellings, additional context, more sources."
]
} | []
| []
| [
[]
]
|
||
9mtitf | why are australia's vast empty planes simply not covered in solar panels to help with their power productions crisis? | explainlikeimfive | https://www.reddit.com/r/explainlikeimfive/comments/9mtitf/eli5_why_are_australias_vast_empty_planes_simply/ | {
"a_id": [
"e7h7o2l",
"e7h7oku",
"e7h7r1s"
],
"score": [
8,
9,
2
],
"text": [
"The same reason they're not spinning up new gas plants - Money and maintenance. It's not like you just stick solar panels out in the middle of a dusty wasteland and that's that. They need to be continuously cleaned to deliver anything close to their rated output.",
"A couple of reasons:\n\n* it's expensive as fuck\n* you lose a lot of the power having to transfer it across lines to where it is useful. \n* despite it being fairly inhospitable to people, there's still a great deal of wildlife there that would be impacted by such a huge construction project. \n\nI mean yeah put a solar plant or two as close as you can to civilization, but don't cover the land with them. ",
"*plains\n\nPartly because those huge deserts are nowhere near where people need the power to be. You can generate lots of power in the middle of nowhere, but you'll lose most of it in the process of transporting it to the cities.\n\nSpain has been fairly successful at utilising it's desert-like areas for solar power, but it is much more densely populated so the efficiencies work out better. (Plus they don't have a mining boom to prop up the otherwise dying coal industry, but that's another matter)"
]
} | []
| []
| [
[],
[],
[]
]
|
||
7j0dnk | why is it recommended to cook with cold water instead of hot water? | explainlikeimfive | https://www.reddit.com/r/explainlikeimfive/comments/7j0dnk/eli5why_is_it_recommended_to_cook_with_cold_water/ | {
"a_id": [
"dr2pzd4",
"dr2q0r7",
"dr2q1dh"
],
"score": [
2,
2,
3
],
"text": [
"There are 2 reasons, that I know of, \n\n1. Hot water can leach minerals and metals out of pipes/ lining of the hot water heater affecting taste/ heavy metals are no fun.\n\n2. If a hot water heater does not properly cycle to high temps (160 I think) it can become a reservoir for legionnaires disease",
"The notion is that the cold water is fresher. (This means it has more chlorine in it) It also has not picked up anything while traveling through the hot water tank.",
"Yo ho ho! Yer not alone in askin', and kind strangers have explained:\n\n1. [ELI5: Why is it better to start with cold water and boil it when making pasta or coffee, rather than starting with hot water? ](_URL_3_) ^(_30 comments_)\n1. [ELI5: When I boil water, why am i supposed to use cold water? ](_URL_2_) ^(_29 comments_)\n1. [When cooking, why do we not boil hot water from the faucet? Won't it be faster and doesn't the heat kill any bacteria anyway? ](_URL_1_) ^(_4 comments_)\n1. [ELI5: Why do packages often say bring cold water to a boil? ](_URL_0_) ^(_11 comments_)\n"
]
} | []
| []
| [
[],
[],
[
"https://www.reddit.com/r/explainlikeimfive/comments/39nfrs/eli5_why_do_packages_often_say_bring_cold_water/",
"https://www.reddit.com/r/NoStupidQuestions/comments/4bg2du/when_cooking_why_do_we_not_boil_hot_water_from/",
"https://www.reddit.com/r/explainlikeimfive/comments/2ha2pj/eli5_when_i_boil_water_why_am_i_supposed_to_use/",
"https://www.reddit.com/r/explainlikeimfive/comments/1ugshq/eli5_why_is_it_better_to_start_with_cold_water/"
]
]
|
||
1ae3eu | What properties make an object heat faster in a microwave? | I have noticed that different plate materials will have very different reactions to being microwaved, as will different types of food. What properties make an object heat faster or slower in a microwave? | askscience | http://www.reddit.com/r/askscience/comments/1ae3eu/what_properties_make_an_object_heat_faster_in_a/ | {
"a_id": [
"c8x23xt"
],
"score": [
2
],
"text": [
"You are observing the dielectric properties of the various materials. Some materials like dry ice are transparent to microwaves and will not heat up in a microwave because it has low dielectric loss. Some ceramic plates and bowls have higher dielectric loss than others meaning they couple with the microwaves and begin to vibrate generating heat.\n\n\n\n\nMetals reflect most of the microwaves but the surface layer can build a considerable charge creating sparks if the voltage is high enough to arc through air."
]
} | []
| []
| [
[]
]
|
|
3zjfk3 | How much of Real Madrid's early success was because of Francisco Franco? | AskHistorians | https://www.reddit.com/r/AskHistorians/comments/3zjfk3/how_much_of_real_madrids_early_success_was/ | {
"a_id": [
"cymm7w4",
"cymwba9"
],
"score": [
2,
4
],
"text": [
"Hi, just a note that this sub isn't always the deepest in sports historians, so if you don't get answers here - or even if you do - consider x-posting this to /r/realmadrid, /r/soccer, or /r/football",
"Sid Lowe's book 'fear and loathing in la liga' goes into great detail on this. Essentially people on both sides will claim lots and nothing.\n\nAn important point to note is that real madrid were one of the best teams in the country directly before the civil war and were gutted by the civil war. Real madrid's president was a key part of the republican government and so was exiled upon franco winning the civil war and the vast majority of real's trophy winning team broke up and either left spain or went to other clubs. In particular, their star player Zamora ended up in a prisoner of war camp and was appointed manager of atletico madrid after the war finished, where he won them their first league title despite the fact that atletico had been relegated prior to the civil war and therefore probably shouldn't have been allowed to play in la liga that year. \n\nSo as result, the first 15 years of franco's reign were real's worst ever as a club (they won no league titles and only two cups), because while Atletico had the reputation as the establishment's choice and barcelona were assembling their firstly truly great team, Real were doing very little at all.\n\nThe tipping point, though it wasn't obvious at the time, was a copa del rey semi final with Barcelona. Barcelona won the first leg 3-0 thanks, in part, to a great support from their fans resulting in the ref being a bit of a homer. Real made a big deal in the papers about this and encouraged their own fans to do the same in the return leg and they won 11-1. Barcelona players have claimed they were basically told before the game by officials not to try and win that game and were threatened into losing.\n\nNow during the civil war, barcelona and madrid were the two main centres of opposition to franco. It was only when first barcelona and then madrid finally fell that the war was won. Franco was advised to not base his capital in madrid after the war but in a loyalist city instead and he choose to move there precisely to conquer his enemy, to make madrid his. Both Real and barca had funded the republican side in the civil war but after the war you started seeing franco's narrative taking from, in which the catalans did that because they were traitors but the madridstas did it because they were victims of the republicans looting their club. But Franco's dream was a united castilian spain, run from the biggest castilian city. He didn't want madrid and barcelona united against him but he also didn't want them fighting among each other. He wanted the catalans and the basques to forget they were catalans and basques and become spanish, which is why he even changed the name of barcelona fc. \n\nAnd so this squabbling between a catalan club and a castilian club really didn't suit him especially when both clubs were calling upon their fans and thus the spector of regionalism. So he acted by deploring spanish clubs hating each other when they should be united, had the madrid journalist who'd told real fans to make their stadium a cauldron of noise blacklisted and asked for the resignation of the president of both barcelona and real madrid.\n\nThe first thing the new presidents did were give a speach in which they talked about a new era of friendship and peace between the two clubs. And both of the new men had served in franco's army during the civil war so this was him taking control of the football clubs. Barca's new man was Colonel Josep Vendrell and real's was their ex player and decorated war veteran Santiago Bernabéu.\n\nBernabeu's first major act was to build the Santiago Bernabéu Stadium with money raised by local businesses so that real could have the attendences they needed to afford the best players. He then used that money to buy a bunch of players, threw his support behind the new european cup and turned real into the biggest club in the world. He pretty much did that single handedly and he was only in charge of the club because franco had exiled the president who was there in 1939 for treason and then asked his replacement to stand down for beating barcelona 11-1. So in that respect, 100% of their success was because of Franco.\n\nAlso Bernabeu, like Vendrell, was trusted by the regime, which probably helped him a lot in terms of being able to fight his corner. Sid Lowe uses the fact that bernabeu banned a major francosist figure from the ground as proof that bernabeu had connections, that an outsider would never be able to do that but someone within the regime could without fear.\n\nAnd once Real became successful, Franco used them as much as possible. Real being the best club in the world was fantastic pr for Franco. They were essentially his diplomats in an era where he was trying to build bridges with the west, they were a great source of prestige, and even within spain, it suited propoganda that the best team were castillian and from madrid and not catalan or basque. But again, barcelona were also used for propaganda, the great hungarian footballer Kubala who played for barca starred in a francosist propoganda film depicting his flight from communist hungary to the better land of fascist spain.\n\nSo in the 1950s you had spanish embasseys welcoming the real madrid team when they were playing away in austria or germany say and giving them dossiers of the opposing team before asking them to do a photoshoot or visit an ambassador. The regime wanted Real to remain dominant because it made the regime look good.\n\nA francoist minister said at the time that di stefano's goals, bull fighting and opera singers were there three best weapons in the fight for the hearts of the west.\n\nBarcelona fans argue that as a result Real got the benefit of the doubt in referee decisions and transfer desputes (in particular that di stefano ended up at real rather than barca) because everyone wanted Real do do well.\n\nThere's not much in the way of actual proof of that, though. My own opinion is that it's probably true to an extent.\n\nI do think that if bernabeu had gone to valencia or atletico or seville and made them the best club in europe instead, Franco wouldn't have intervened to help Real at all anymore than he did in the first 15 years of his reign."
]
} | []
| []
| [
[],
[]
]
|
||
44x4gb | how is it possible that we encounter plateaus when losing weight even if we continue to operate at a caloric deficit? | I know plateaus don't last forever I'm just wondering how this happens. What causes a temporary "stall" | explainlikeimfive | https://www.reddit.com/r/explainlikeimfive/comments/44x4gb/eli5_how_is_it_possible_that_we_encounter/ | {
"a_id": [
"czti6ya",
"cztif9i"
],
"score": [
3,
2
],
"text": [
"There are a lot of possible explanations. One big thing to keep in mind is water weight.\n\nWhat are the physical things that make up your weight? Bones... Fat... Muscles... Organ tissues........ and Water! You're actually mostly water, by the way. And the amount of water sitting inside of you can vary DRAMATICALLY.\n\nYou ever really, really have to pee? A full bladder can potentially fill multiple bottles of water when emptied. There's a significant weight difference right there, especially since urine is a little bit heavier than plain water.\n\nBut when you pee, you're not peeing out all of the water inside of you. Your body actually really wants to keep a certain amount of water most of the time.\n\nWe depend on a water-to-salt balance. If you eat A LOT OF SALT one day, your body decides it needs A LOT MORE WATER to compensate. So the water that you drink, or eat in the form of fruit, etc., will just STAY there inside of you.\n\nIf you eat HARDLY ANY salt, and especially if you drink A LOT more water than usual, your body says hey! We can dump all this OLD water in place of the NEW water! And you'll pee a lot.\n\nYes, that's right. The more water you drink, the less water you retain. Putting MORE of something into your body can actually make you LIGHTER (temporarily). Weird.\n\nThe difference between a \"LOTS OF SALT\" day and a \"NO SALT, lots of water\" day can be HUGE. Many pounds.\n\nBut the most important thing to remember is that this is all TEMPORARY. You're NOT gaining and losing fat. It's an ILLUSION.\n\nFor example, you can't do the \"no salt, lots of water\" trick multiple days in a row and expect to lose 5 pounds every day. It doesn't work. Your body only stores so much water. It's best not to think of this as contributing to your \"real weight\" at all.\n\nIt's more like, say, on Tuesday morning Bob steps on the scale, and it reads 205 lbs. Bob has no idea how close this number is to his real weight. His real weight is a total mystery. Bob figures 205 must be pretty close, so that's the number he writes down. (There are actually only 200 pounds of average-day-Bob, but his water stores allow plus-or-minus 5 pounds based on water retention, but Bob doesn't really need to know this.)\n\nBasically, this is the reason it's really kind of silly to record your weight every day. Every week is a little better. But even then, don't take every number for gospel. Come back a month or two later and take a look at the trend line. You'll probably see plateaus, maybe even ups and downs, even if you've been super consistent in caloric intake and exercise. If you eat a lot of salt the day before a weigh-in, you'll \"balloon up\" - who cares? You're still getting skinnier, and your pants are falling down. You've still lost \"real weight\". The scale is just guesstimating.\n\nOne really common thing is to lose weight REALLY, REALLY QUICKLY at the start of a diet. You're eating less than usual, so you're probably eating less salt than usual, so you lose a lot of that \"fake\" water weight RIGHT AWAY and feel super good about yourself. In the first month, maybe the scale says you've lost 10 pounds!... Hooray!... but actually no. You've probably lost like 5 pounds. A week later, the scale hasn't moved. A week after that, the scale still hasn't moved. You think you're plateauing....... You're probably not. You're probably still losing like 1 pound a week. Your water levels are just fluctuating and hiding your progress.\n\n**tl;dr - your scale is lying to you, water weight can fluctuate up and down like crazy, don't trust day-to-day numbers, just look at the trend line over a period of multiple months**",
"A good explanation might be that you did not adjust your calorie defict after losing weight the first time. For example, let's say when you started, your TDEE was 2,000 calories, if you ate anything less, you are losing weight depending on the amount you have a deficit by. Remember that you need to burn about 3,500 calories in order to lose 1 pound of fat. So, at a 2,000 TDEE, you would need to eat at 1,500 calories a day, in order to lose 1 pound a week. BUT, this is where your question comes in. Since you lost weight, you have to adjust your TDEE in order to reflect your current situation. So now instead of your TDEE being 2,000, it is now 1,700. So, if you are still eating 1,500 calories, you only have a 200 calories deficit a day, or 1,000 calories a week. So, now instead of losing 1lb a week, it will take you about 3 and a half weeks to lose the same amount. It is essentially a numbers game. "
]
} | []
| []
| [
[],
[]
]
|
|
3xzax9 | How does our body keep track of time? And how might this effect space travel? | Ageing is a bio-chemical process which seems to be unidirectional. How does our body keep track of time? I know that we have an internal clock...but is it synchronized to something external like the sun, or is it something internal like a quartz crystal in a watch?
How will this effect our ageing process in space...especially with the whole idea of traveling at the speed of light and coming back to find that everyone else has aged so much and you haven't. If the clock is internal, how does it matter how fast the body is moving. | askscience | https://www.reddit.com/r/askscience/comments/3xzax9/how_does_our_body_keep_track_of_time_and_how/ | {
"a_id": [
"cy943ub",
"cy94atn",
"cy94tfo",
"cy9550q",
"cy97h44",
"cy97i0v",
"cy97pu8",
"cy99jeo",
"cy9aztd",
"cy9goai",
"cy9jt53",
"cy9la7h",
"cy9oyb8",
"cy9rszz",
"cy9ubeu"
],
"score": [
5,
1327,
8,
48,
5,
3,
2,
113,
6,
4,
2,
2,
5,
2,
2
],
"text": [
"_URL_0_\nInteresting read",
"On your question about the role of the sun and the implications for space travel, there were a series of interesting studies a while back that provides part of the answer. The goal of these studies was to determine if our normal [circadian rhythm](_URL_3_) (as measured by physiological markers like the body temperature) as well the sleep/wake cycle was set by some kind of an internal clock or by external factors such as sunlight and other temporal cues. To answer this question researchers placed a group of individuals in a controlled environment e.g. in bunkers), where at first they gave them temporal cues (by providing access to TV, radio, etc) about the time of day. \nDuring this adaptation period, both the circadian rhythm and the sleep/wake cycle remained matched to the 24 hour cycle. But then, the researchers removed these cues to see how the individuals would adapt. \n\nInterestingly what happened is that the [sleep cycle of many of the individuals gradually lengthened](_URL_4_) ([source](_URL_1_)). In fact further studies have a shown that the sleep/wake cycles can vary significantly among individuals. In contrast, when the circadian rhythm was monitored by measuring physiological markers (such as the core body temperature), [researchers found that individuals maintained a circadian rhythm of about 24 hours (on average 24.18hrs)](_URL_0_), with much less variation. These results suggest that while external cues and stimuli can cause the activity cycle (or sleep/wake cycle) to change dramatically, the internal clock that sets the circadian rhythm is much more stable and in most people is tightly aligned with the 24 hour daily cycle. Nevertheless, this fact does not mean that external stimuli do not matter as far as the circadian rhythm is concerned. In fact over time the circadian rhythm can respond to match external stimuli such as the light/dark cycle in a process called [entrainment](_URL_2_). This fact makes it possible to gradually shift the circadian rhythm (within a certain range) from the normal 24 hour cycle by changing the external conditions.\n\nEdit: Added in the original sources of the studies referenced, and made a key correction as pointed out by /u/whatthefat ",
"You're mixing a few different topics that makes your question difficult to answer clearly. I'm going to attempt to tackle some of the parts of it, but I cannot offer a clear answer. Hopefully someone with expertise in ageing can help.\n\nYour \"internal clock\" is your circadian rhythm. It's synchronized to outward stimuli like the sun, but it's trainable and adaptable. When you disrupt your circadian rhythm (like flying across the world), you experience jet lag. In space, it's been suggested to set up a light-dark cycle artificially on long duration exploration flights ( > 6 months) to help give astronauts a sense of normalcy, but that's for sub-light speed travel.\n\nNow, to address relativity, or the effects of time dilation from travelling above the speed of light. [Here] (_URL_0_) is a super cool video explaining relativity to a layperson, including addressing ageing due to lightspeed travel. He explains it very succinctly and clearly, but here is an explanation for the lazy. All of the normal processes (like ageing) that happen in the body can be broken down into a series of interactions at the quantum level. To interact, these particles move at the speed of light over a very short distance to interact with each other, which happens very quickly. However, if your body is moving at the speed of light, the particles have to travel their normal tiny distance, plus the large distance your body has traveled, to get to their destination. Since distance increases with a constant velocity, the time for this interaction to occur happens increases. The faster you travel, the longer it takes for these interactions to occur, and, theoretically, the slower you age.\n\nNow, I'm unsure how the ageing process in particular works and how it's changed in space. Hopefully someone else can shed some light on the matter.",
"Circadian clocks are built into the majority of cells in the body, and are synchronized in large part by the suprachiasmatic nucleus in the brain (which is in turn entrained by light from photosensitive cells in the eyes) and circulating melatonin (which is produced in the pineal gland when it's dark out). Circadian rhythms are thus largely regulated by the 24-hour light/dark cycle (though other cues, such as typical meal times, also play a role).\n\nWithout photoentrainment, circadian rhythms run longer than 24 hours, though this varies across individuals (usually around 25 hours). For example, folks who are totally blind (i.e., no light is processed by photosensitive cells in the eyes to the brain) often have trouble sleeping because their internal clocks run out of sync with the rest of the world. It has been suggested that daily administration of melatonin (at the same time each day) assists in entraining the clock in these individuals and relieving sleep issues. See: Sack, R. L., et al. (2000). Entrainment of free-running circadian rhythms by melatonin in blind people. New England Journal of Medicine, 343, 1070-1077.\n\nAs for \"how does it matter how fast the body is moving\", that's more a physics question than a biological one. A physicist will do a much better job than I could, but suffice to say that physical time (such as it exists) is relative to your perspective. As far as your body is concerned, I expect that travelling near light speed would be processed in the same way as travelling at any other speed, and the only difference would be that time would have been moving more slowly for others relative to you. As far as you are concerned, x time has passed (because it has relative to you); and as far as others are concerned, y time has passed (because it has relative to them). \n\nKeep in mind that relativistic speeds at the macro level are not something that cell biology normally needs to account for.",
"I will try to adress the last part of your question, about near light speed travel. Let's say you are travelling at a constant speed very close to the speed of light. As you are traveling very quickly compared to the people left on earth, you and everything on your spaceship will experience time dilation. Time will pass more slowly on your spaceship than on earth, you will age slower. \n\nNow let us imagine you wanted to measure that effect from inside your ship. What kind of test could you perform? You could try keeping track of \"earth time\" by bringing a piece of radioactive rock and measuring its decay. But all the atoms in this rock would experience time dilation at exactly the same rate as you. If you brought an earth pocket watch it too would slow down by exactly the same factor. Even the neurons firing in your brain would fire more slowly and hence even your thinking would be slowed down by the same amount as the watch and the rock. No matter how you tried finding something to measure time dilation from inside your spaceship you would run into this same problem. In fact, given that you are traveling at a constant velocity, there is NO experiment whatsoever you could perform that would behave any differently than if you were sitting still on earth's surface in your spaceship doing the same experiment. (Let's assume there is some way of replicating earth gravity in your ship)\n\nSo since it is physically impossible for any kind of test to separate the two cases this means there is litterally no way you could experience six months passing on your spaceship any differently than you would have on earth. ",
"I'd like to piggyback this question by asking, how do our brains regulate time in a more immediate sense? Like if I'm clapping at an even tempo, how does the brain tell how much time has passed since we last clapped and when it's time to clap again?",
"Biochemical reactions (including ageing) would occur at the same rate as your perception of time - after all, your perception of *everything* is due to biochemical reactions. Although ageing is currently a hot topic in current research, I think the general feeling is that it is due to an accumulation of reactions and occasional misreactions in your cells, ie protein misfolding, DNA damage, leading to cell senescence, depletion of stem cells, etc. The internal clock is just due to circadian rhythm, as mentioned in other posts - for the first number of \"days\", a space travellers wake/sleep cycle might be messed up, and I think astronauts would enforce a Earth-like light/dark cycle to keep this normal. A prolonged period without light would mess up your circadian rhythm and psychology, and may have knock-on effects to inrease ageing, but I don't think that's specific to travel - just ask anyone who lives North of the Arctic circle.\n\nAt relativistic speeds, all of the molecules in your body are within the same frame of reference, so there shouldn't be any difference to biochemistry as you perceive it. Your body would be expected to age to the same amount of time that you have experienced personally. (This is ignoring any effect of increased radiation due to being outside of the protective atmosphere of Earth)",
"Neurobiologist here. It's early, I'm hungover, but allow me to try and explain this the best I can. As others on this thread have commented there are various ways your body keeps track of time, the most notable are circadian rhythms. But this is somewhat wrong, you see it's not circadian rhythms that *produce* our sense of time, but rather are a *product* of various biochemical events in the brain (notably the superchiasmatic nuclei) which give rise to frames of reference that we perceive as time.\n\n\nHow this works in its entirety is still somewhat a mystery. My old phd mentor was actually looking at how subregions of the frontal lobe may use various methods to check and compare events that have recently occurred to give a reference frame for length of duration between events. Others have pointed to a series of chemicals in the brain for giving that 'value' for duration of time elapsed, one of which is adenosine, the others are the circadian chemicals 'Clock' and 'BMal' (plus one more I can't remember for the life of me). These chemicals and their density in certain receptor sites encode values for duration awake and roughly can translate into time elapsed since your last 'Brain event' which signals a 'checkpoint' like sleep or other cycles. \n\n\nLet's start with adenosine. You may not know it but this little bugger is something we all hate and try to inhibit on a daily basis. When adenosine triphosphate (atp) breaks down over the day from atp to adp to amp it finally rests with just a single adenosine molecule. These adenosine packets then float around the brain till they fine their receptor site and bind there, with enough binding you start to feel groggy and sleepy. It's also the receptor site that caffeine binds to and prevents said tiredness. While this isn't translated directly into 'time perception' as we know it, it is one more daily cycle that allows your brain to make reference points and determine how much time has gone by in your single wake cycle, and as such the amount of free floating adenosine may allow for the brain to reference around what time events have occurred.\n\n\nNow for the slightly confusing parts with our circadian rhythm making friends BMal and clock. These chemicals have little known use beyond producing a slow, continuous wave of increase and decrease in each other. We think this rise and fall of the two chemicals produce circadian rhythms; they achieve this in a pretty cool manner, clock and BMal both affect each others production amount. As one increases in concentration, the other decreases, doesn't sound complex right? Wrong! As the decrease occurs it inhibits production, which then inhibits the inhibiting factor of the other, which in turn increases the amount, which in turn inhibits itself. Bah, go look up the fine points, my head is throbbing too hard to think about this too hard. But the underlying point is that the two regulate each other, when one gets to be too concentrated, it produces the other, which decreases the first, then the second becomes too concentrated and the cycle continues with each regulating the other. The byproduct of this ebon flow of ups and downs is a predictable, constant wave. With the wave having roughly the same amplitude and frequency each time, it allows your brain to use other reference points from your long term or short term memory, encode them to a point in the 'wave' and then reference the cycles to determine how much time has passed. (This is my most sloppy part of my post, bear with me I'll edit more points in after I wake up more.) \n\n\nSomething you may have noticed is that I have constantly said 'reference' that's because the brain needs cues to operate. You need light and other factors to tell the brain 'it's this time in the day'. Or else we would be bound to one internal clock and jet lag would suck so much harder. (I just realized I left out a part on melatonin, while it helps set cycles I don't think it has that much to do with time perception) because we need cues our brains, and that of other animals, have developed strategies. For instance some species of lizards have a very very thin skull right above their pineal gland; they use this to tell if it's light outside and to deliver light directly to the brain! Yes, you heard me, lizards have photosensitive brains...that act like... Well, a third eye (cue spooky music). We, on the other hand, are not as cool. We use our eyes and optic nerve to deliver light to the brain and let it know \"dear brain, it's X brightness outside, leading me to believe it's X time of day\". But even then it's all to do with frame of reference.\n\nTo conclude, your brain constantly communicates with a plethora of biochemical and physical indicators of duration/frame of reference. The amalgamation of all these things I mentioned (and many more I did not) lead your body to have a sense of time, or at lease how many 'cycles' have occurred since last frame of reference. The body clock isn't perfect, it can make time feel very slow (like being hungover listening to the garbage truck crash around outside your house, ugh) or very fast (like explaining your scientific passion to someone who cares to ask), but at the end of the day it's really a hard question to answer. How do we perceive time? The best I can do is explain what I know, what others have observed and what the community thinks. For all we know it's could be all wrong... I guess time will tell ;)\n\n",
"Aging has nothing to do with the body \"keeping track\" of time, other than in some very metaphorical sense perhaps. Aging is basically cells degrading because of things like junk accumulating within and betweem cells making them work more and more poorly. This is in principle \"curable\" in which case time could go on indefinitely without any biological aging.",
"This may or may not be relevant of a question to post here, but I always wondered how my brain knows to wake me up 5 minutes before an alarm, or just before I need to wake up. Unless I'm sick or tired (or drunk), I never use an alarm and wake up an hour or before I need to leave for work just fine. It's like my brain 'knows' I need to wake up. ",
"You're confusing two different things. Our physical age is not determined by our sense of time. Or perception of time is completely separate. \nFor example, people who are unconscious for short periods of time or in long term comas. You can wake up from a coma and not realize a year has passed, your body might feel like it's been a few days, but this doesn't mean your body has only aged based on its perception of a few days. Cell processes still occur as normal, aging continues to happen. You cannot age differently by messing with your body's clock, which gives us our sense of time. \n\nSomeone else mentioned things like puberty and hormone changes based on age. This is again independent of your sense of time. You don't turn 14, your body realize you're at the right age and then start altering hormone levels. Hormone changes occur based on certain brain functions which occur as your brain reaches certain development points and certain biological events are triggered. \n\n\nAll this means is that with space travel, you'll be largely unaffected. Regardless of your speed, time passes the same within your reference point. Within your reference point, 1 year may pass but to those traveling at slower speeds, 10 years will pass. Your body will age one year because that is how much time has passed for you. Your body won't be confused and think \"shit 10 years has passed for everyone else, I better catch up\". ",
"This is more of a follow up question than an answer.\nI have studied linguistics and touched onto the subject of the construal of time by the brain (in order to understand the metaphoric system of TIME AS SPACE in language (ie. moving FORWARD in time, phrases like ACROSS time etc)). In some of the literature they talk about certain neurons firing in bursts, than having an empty space, and firing bursts again. This is hypothesized to induce a sort of feeling of time passing. It is argued that it is a sort of sense that we have to perceive time, as opposed to it being an illusion our brain puts together from the sequence of events around us.\nAdditionally, they talk about a perceptual moment, the smallest unit of time we feel as passing. It is somewhere between one and two seconds. An example where this becomes obvious is that we dont hear single ticks of a clock, ie. tick tick tick. Instead, we pair them up as tick tock tick tock. One tick is shorter than our perceptual moment, therefore we make two ticks into one unit.\nNow I'm not sure how up to date this is. cognitive linguistics is sometimes a bit weird when it comes to actual cognitive sciences. Can someone tell me if this is somewhat accurate or makes sense at all?",
"Our bodies are adapted to life on a planet where there is a 24h light cycle, with the light/dark ratio varying depending on how close to the equator you are. Many processes in the body, such as temperature, athletic performance, cognitive function, and most obviously sleep oscillate daily. There is actually an entire field dedicated to this called chronobiology.\n\nCurrent studies suggests that there are peripheral oscillators in our bodies, but our main biological rhythm is driven by a part of the brain called the suprachiasmatic nucleus (SCN), which is in turn stimulated by input from the eyes. There are many components to the proposed molecular clock, but it is proposed that this light causes degredation of a protein that builds up during the night, which in turn alters transcription of clock genes, which control expression of clock-driven genes. So for your question, it's both! We all have an endogenous clock due to molecular mechanisms, but it can be \"entrained\" to external light, such as the sun. \n\nThe circadian rhythms Wikipedia page is pretty through, if you want a well-worded explanation.\n\n\n",
"I am several hours late, so hopefully this is helpful. I have tried to read through some of these answers, and like a lot of people mentioned I can't speak to all these questions. However, I have done quite a bit of work on the body clock (i.e. Circadian clock) found in the Suprachiasmatic Nucleus (SCN). I will try to keep it relatively simple (it is a complex topic). \n\nIt is internal: The 24 rhythm is accumulation of various processes from the translation, transcription, dimerization, localization and degradation of various genes and their products (such as BMAL, PER, CLOCK and various other parts). Literally anything and everything that can happen to them that affects their relative concentration in various parts of the cells will affect the timing of the clock. These processes are a feedback loops that are self-regulating (i.e. these components all feedback and control each other in a big loop). \n\nThis accumulation of processes within the cell, and within the population of cells within the SCN give around a 24 hours rhythm to the organism. But *any change* to any of these processes will affect the timing of accumulation/degradation and will therefore affect the systems timing^1. And if fact it is this principle that helps organism to adapt to the an environment (i.e. entrain to specific cue/light/time). And light is something that entrains us (and most animals at least) to a specific time (and is the strongest entrainer). But so does food and other things (e.g. exercise). \n\nAs for space travel, my physics is limited, so others can answer it. But since the circadian clock is a relative marker of time (it marks cycles, so that it knows where you are relative to the 24 cycle of earth's rotation) rather than \"time\", I doubt it would notice any specific issues with space travel. I would be guessing as to what happens, but I am pretty sure so would anyone else. But I am pretty sure any affect you feel would be similar to how you feel overall. \n\n\n-----------------\n^1 Any mutation in any of these processes will also affect either the timing, entrainment ability or \"free running\" rhythm (timing in absence of any external timing cues). ",
"You have a few concepts wrapped together that are actually unrelated. Others have explained pretty thoroughly the biological component of the question, how the body keeps track of time. However, it's important to note that the body's internal clock does more to regulate your day to day activity than the process of aging, aging is largely the accumulation of errors as your cells replicate (I can go into more detail if you'd like), not something the body does on purpose and largely unrelated to the day to day rhythms. \n\nIn space, the cues which inform your body on how to set your day to day cycle can be different, but the rate at which your body degrades (read: ages) remains the same. If you strap a watch to someone on earth and another to someone in space, after 80 years the two people will have both aged by 80 years. But, and this is very important, the watches will be off from eachother by about 5 seconds. \n\nEven if the watches keep time perfectly, there will still be this difference. Einstein's theory of relativity states that time moves more slowly in the presence of a strong gravitational field (relative to someone in a weaker gravitational field), and that time acts more slowly on something moving close to the speed of light (relative to an observer). This is what people are talking about when they say you can be gone 2 years on a space ship and return to find everyone aged 80 years, time itself has run differently for you. As far as your body rhythms are aware, they go at the same rate as they always have, and your environment is being affected by the same time-warping effects as your body, so every method you use to tell time is warped as well. There is litterally no way to know until you return to the reference frame in which you started."
]
} | []
| []
| [
[
"https://www.quantamagazine.org/20150915-circadian-clocks/"
],
[
"http://www.sciencemag.org/content/284/5423/2177.short",
"http://www.sciencemag.org/content/148/3676/1427.short",
"https://en.wikipedia.org/wiki/Entrainment_%28chronobiology%29",
"https://en.wikipedia.org/wiki/Circadian_rhythm",
"http://i.imgur.com/sde0o4f.jpg"
],
[
"https://www.youtube.com/watch?v=CYv5GsXEf1o"
],
[],
[],
[],
[],
[],
[],
[],
[],
[],
[],
[],
[]
]
|
|
2kp9m9 | How does graphite bind to paper, and how does an eraser remove it? | When you write on paper with a pencil, does it chemically bind to the paper. how does an eraser remove the marks without changing the paper. | askscience | http://www.reddit.com/r/askscience/comments/2kp9m9/how_does_graphite_bind_to_paper_and_how_does_an/ | {
"a_id": [
"clnwq8q",
"clogswh"
],
"score": [
3,
3
],
"text": [
"[So this is what paper looks like close up](_URL_0_)\n\n[This is what graphite might look like close up](_URL_1_)\n\nWhen you write, the graphite becomes all dusty and lands on the paper, and into the holes of the paper and all over it. But it isn't bonded to it. This is why, after long periods of time, lots of things like books and papers and letters fade and become harder to read, because this dust that is on the paper flies off. \n\nAn eraser rubs right on the paper, and pulls some of the dust off. This is why the eraser leaves little rolled up rubber side-effects, with graphite on them, because they got stuck to the sticky rubber. \n\nThere is no bonding, just being stuck in the holes of the paper and all around it, and so on. Also, pressing a pencil on the paper makes nice little indents for the graphite to stick into, so it's not just like you can shake your paper and have the graphite fall off!",
"To the best of my knowledge, the bond between the graphite and the paper isn't chemical. The bond should primarily be through van der Waals forces, which are electrostatic as opposed to chemical. What's happening is that graphite has a structure like [this](_URL_0_) with lots of layers. \n\nThese layers are only weakly bonded (compared to a covalent or ionic bond) to eachother by van der Waals forces, allowing them to shear easily away onto the paper when you use your pencil, which is why we use graphite in pencils in the first place- most solids won't rub off onto a surface so easily. So after being sheared off of the pencil tip, the graphite planes (referred to individually as graphene planes) will bond with the paper using the same van der Waals forces. The reason paper makes such a good writing surface is, as you can see from /u/PoorAuthor9's post, it is very rough on a molecular scale- lots of little edges and loops that the graphite planes can get stuck on while you're sliding your pencil across the surface. This is why pencils work rather poorly on very smooth surfaces.\n\nAn eraser works by giving the graphite that has adhered to the paper something more attractive to stick to. I'm not exactly sure on the molecular level details, but something about the structure of the rubber material makes the graphite stick more strongly to it than to the paper. Because it has a stronger attraction to the graphite than the paper, the stronger bond will remove the graphite from the paper when you move the eraser.\n\nSimply washing with soap and water will also remove the graphite- water & the surfactant will interrupt the van der Waals bonding between the graphite planes and the water, causing it to come off the surface. This would probably be easier than erasing if you got a lot of pencil drawn onto yourself or your clothing. Obviously not a good approach to fix mistakes on your homework, though. :)"
]
} | []
| []
| [
[
"http://paperproject.org/semgallery/images/silk1x100.jpg",
"http://www.canadacarbon.com/images/miller_sem07.jpg"
],
[
"http://batteryblog.ca/wp-content/uploads/2010/06/graphite.gif"
]
]
|
|
3ovsqp | Royal chefs in Middle Ages | Were important cooks for the court always men? Were there female hear cooks in charge of royal feasts, etc? I understand there was some superstition about women on ships, so that explains male cooks in one context. But in other important roles, what was the gender breakdown on professional cooking? | AskHistorians | https://www.reddit.com/r/AskHistorians/comments/3ovsqp/royal_chefs_in_middle_ages/ | {
"a_id": [
"cw0y5d6"
],
"score": [
3
],
"text": [
"The first German language cookbook was written by a woman named Sabina Welserin in 1553 (you can read it [here](_URL_0_)). Not much is known about her life but she mentioned in one recipe that the cook for Count of Leuchtenberg instructed her to cook fish a certain way hinting she is a professional cook of some kind. From the recipes she wrote (many recipes ask for large quantities of spices and sugar and some recipes were clearly for fancy banquets) it suggests she cooks for a very wealthy (if not downright aristocrat) household. "
]
} | []
| []
| [
[
"http://www.daviddfriedman.com/Medieval/Cookbooks/Sabrina_Welserin.html"
]
]
|
|
2c1k27 | why would human poop be white? | When I joined the Navy, eating a drastically different diet gave my bowel movements a variety of strange colors, most notably green. That, I learned, was due to increased iron content. On occasion, though, I've had poops that were pure white. What causes this? | explainlikeimfive | http://www.reddit.com/r/explainlikeimfive/comments/2c1k27/eli5_why_would_human_poop_be_white/ | {
"a_id": [
"cjazjek"
],
"score": [
7
],
"text": [
"White stool is not normal and should be evaluated promptly by a doctor. White stool is caused by a lack of bile, which may indicate a serious underlying problem.\n\nBile is a digestive fluid produced by the liver and stored in the gallbladder. Stool gets its normal brownish color from bile, which is excreted from the liver into the small intestine during the digestive process. If the liver doesn't produce bile or if bile is obstructed from leaving the liver, stool will be light colored or white.\n\nLiver diseases, such as hepatitis and cirrhosis, can cause white stool. In some cases, the problem lies not in the liver but in the tube (duct) that delivers the bile to the intestines. This tube can be squeezed shut or blocked — for example, by a tumor or a gallstone — which prevents the bile from entering the intestines. Some babies are born with constricted bile ducts.\n\nmayo clinic"
]
} | []
| []
| [
[]
]
|
|
fik4y3 | what is actually in contraband (cheap) cigarettes and why are they so cheap and illegal? | explainlikeimfive | https://www.reddit.com/r/explainlikeimfive/comments/fik4y3/eli5_what_is_actually_in_contraband_cheap/ | {
"a_id": [
"fkhkj3a"
],
"score": [
9
],
"text": [
"Because some states like New York tax cigarettes heavily. Contraband cigarettes are purchased out of state to avoid the taxes and sold at a discounted rate."
]
} | []
| []
| [
[]
]
|
||
5l3lf2 | Winston Churchill often spoke about his "little black dog" (i.e. depression) that constantly followed him around. Did any Nazi leaders suffer from depression or other clinical mental disorders? | Looking into it, seems like a few Allied leaders were being treated for mental health, Churchill being the most famous. But I never hear about that on the Axis side. Did any German/Japanese/Italian politicians or generals suffer from mental illness? | AskHistorians | https://www.reddit.com/r/AskHistorians/comments/5l3lf2/winston_churchill_often_spoke_about_his_little/ | {
"a_id": [
"dbsr8be"
],
"score": [
69
],
"text": [
"Suicidal tendencies were a sufficient problem in the SS that Himmler made a point of saying that funerals shouldn't be given for officers who committed it for \"trivial\" reasons. Between July and September of 1942, 30 members of the SS committed it. One particular case was a SS Brigadier named Fritsch who killed himself in 1944 because he cheated on his wife and felt guilty. Himmler ordered the SS insignia removed from his headstone. \n\nSource: Suicide in Nazi Germany\nBy Christian Goeschel\n_URL_0_\n\n"
]
} | []
| []
| [
[
"https://books.google.com/books?id=oysUDAAAQBAJ&pg=PA61&dq=SS+Officers+depression+suicide&hl=en&sa=X&ved=0ahUKEwiNkPTNppzRAhUH3IMKHdrjAR0Q6AEIGzAA#v=onepage&q=SS%20Officers%20depression%20suicide&f=false"
]
]
|
|
32nv6x | If I unraveled the DNA in a human cell, how strong would it be? | Could I make a rope from it? What would it look like if I gathered enough to make a visible length of it? | askscience | http://www.reddit.com/r/askscience/comments/32nv6x/if_i_unraveled_the_dna_in_a_human_cell_how_strong/ | {
"a_id": [
"cqdplzk"
],
"score": [
6
],
"text": [
"What you're looking for is the tensile strength of DNA compared to the tensile strength of something like, say, nylon. It takes about 1 nN to break a strand of DNA, which at around 2nm wide and roughly circular, gives you a tensile strength of about 318 MPa. This is compared to somewhere in the neighborhood of ~100 MPa for nylon and other common polymers used in ropes. If we assume you can weave DNA strands into a cable, they would be quite strong, barring degradation and the thermodynamic nightmare that would be building such a large DNA structure.\n\nEven though DNA is pretty rigid as a double-strand, once you get very long, thermodynamics make it want to curl up on itself, so you've got that working against you.\n\nSource: Materials science researcher with a background in DNA nanotechnology"
]
} | []
| []
| [
[]
]
|
|
nusvn | why everyone seems to hate /r/politics. | I mean, often times in other subreddits when /r/politics comes up it's talked about with distaste. | explainlikeimfive | http://www.reddit.com/r/explainlikeimfive/comments/nusvn/eli5_why_everyone_seems_to_hate_rpolitics/ | {
"a_id": [
"c3c3jzf",
"c3c3lwh",
"c3c44yn",
"c3c46wq",
"c3c4zlz",
"c3c57hl",
"c3c5gcx",
"c3c5vdo",
"c3c5zvo",
"c3c3jzf",
"c3c3lwh",
"c3c44yn",
"c3c46wq",
"c3c4zlz",
"c3c57hl",
"c3c5gcx",
"c3c5vdo",
"c3c5zvo"
],
"score": [
43,
18,
10,
12,
4,
2,
2,
6,
2,
43,
18,
10,
12,
4,
2,
2,
6,
2
],
"text": [
"Reddit as a website skews young, especially on the default subreddits. Reddit also skews liberal (which I am too).\n\nAs a result, you get a bunch of 15-18 year olds yelling about politics they have only recently started to be interested in and learn about and which they barely understand. They lack an awareness of context/historical precedents. Further, [/r/politics](/r/politics) has turned into a witchhunt for any sign of conservativism while it derides conservatives' endless debasing of liberals. The whole of the subreddit doesn't seem to grasp the irony that they're as distorted and left wing as Fox news is distorted and right wing.",
"It just turned into a bit of a circlejerk, so i gave up",
"stay out of all subreddits whose primary topics are sex, religion or politics. They are doomed because of the nature of the internet.",
"Any attempt at expressing an opinion that differs from that of the Hive is ruthlessly punished by moderation and any opinion that plays to the hive is upvoted without regard to its factual merit.",
"First of all, as MySuperLove explained, the subreddit is particularly suited toward young liberals. On any given day, if you look at the articles on the front page of r/politics, you'll see that just about every article supports the same side of an issue or candidate in an election. I haven't been keeping up to date with it since I unsubscribed, but I can't recall seeing a link showing how a Republican presidential candidate, other than Ron Paul, did something good.\n\nAnother aspect is the constant accusations of America being a police state or something along those lines. While I'm not one of those Americans who is so patriotic that I wear red, white, and blue every day, have a flag waving on my front lawn, or possess an incubator for baby bald eagles, I still have some respect for the way my country functions. Simply put, particularly during the Occupy Wall Street protests, many articles and comments were upvoted that referred to the US as a police state, where freedom of speech was non-existant and police could arrest whomever they wanted. Needless to say, not everyone agrees with that view, and I find such views a bit insulting. It's hard to take a group of people seriously when Occupy protests lasted a few weeks and continue to be discussed on a relatively well-known site like Reddit, and they still insist that the government has surpressed all free speech against the status quo. ",
"Obviously Reddit skews liberal so /r/politics is going to skew that way as well which can’t be helped. But a conservative or anyone that has a thought different than that of the hivemind is immediately attacked and derided. Even worse there are many instances of moderators removing posts they don’t agree with, which is pretty much a slap in the face of the entire core of Reddit as a place for people to discuss ideas and opinions. /r/Politics is simply a joke at this point in time and will be until Reddit removes the offending moderators and works to instill some basic Kindergarten rules of “play nice with others” and “don’t call people Nazis just because you don’t like what someone says”.",
"Because politics and politicians in general just suck.",
"I find the problem being that it is the type of politics which is pretty nonacademic, reactionary and sensationalist. So they tend to discuss things in a very aggressive way which never leaves a good result of any debate, they tend to base their opinion more on gut feeling and fail to look at the details and causes rather than just being outraged at x forcing this on y. It's all american politics and very partisan, liberals and conservatives who stick very much to their party lines. \nAn example was \"That's it, Congress has made Pizza a vegetable\". Although eating it counted as a fruit portion, there was a lack of analysis of why it was being seen as a fruit portion (the tomato pureé and the lobbying power, as well as an ideological battle against the big state), instead it was quite a big shouting match about something that wasn't so well defined.",
"I should add that that r/politics constantly leaks to other subreddits, and that is annoying for some people.\n\n ",
"Reddit as a website skews young, especially on the default subreddits. Reddit also skews liberal (which I am too).\n\nAs a result, you get a bunch of 15-18 year olds yelling about politics they have only recently started to be interested in and learn about and which they barely understand. They lack an awareness of context/historical precedents. Further, [/r/politics](/r/politics) has turned into a witchhunt for any sign of conservativism while it derides conservatives' endless debasing of liberals. The whole of the subreddit doesn't seem to grasp the irony that they're as distorted and left wing as Fox news is distorted and right wing.",
"It just turned into a bit of a circlejerk, so i gave up",
"stay out of all subreddits whose primary topics are sex, religion or politics. They are doomed because of the nature of the internet.",
"Any attempt at expressing an opinion that differs from that of the Hive is ruthlessly punished by moderation and any opinion that plays to the hive is upvoted without regard to its factual merit.",
"First of all, as MySuperLove explained, the subreddit is particularly suited toward young liberals. On any given day, if you look at the articles on the front page of r/politics, you'll see that just about every article supports the same side of an issue or candidate in an election. I haven't been keeping up to date with it since I unsubscribed, but I can't recall seeing a link showing how a Republican presidential candidate, other than Ron Paul, did something good.\n\nAnother aspect is the constant accusations of America being a police state or something along those lines. While I'm not one of those Americans who is so patriotic that I wear red, white, and blue every day, have a flag waving on my front lawn, or possess an incubator for baby bald eagles, I still have some respect for the way my country functions. Simply put, particularly during the Occupy Wall Street protests, many articles and comments were upvoted that referred to the US as a police state, where freedom of speech was non-existant and police could arrest whomever they wanted. Needless to say, not everyone agrees with that view, and I find such views a bit insulting. It's hard to take a group of people seriously when Occupy protests lasted a few weeks and continue to be discussed on a relatively well-known site like Reddit, and they still insist that the government has surpressed all free speech against the status quo. ",
"Obviously Reddit skews liberal so /r/politics is going to skew that way as well which can’t be helped. But a conservative or anyone that has a thought different than that of the hivemind is immediately attacked and derided. Even worse there are many instances of moderators removing posts they don’t agree with, which is pretty much a slap in the face of the entire core of Reddit as a place for people to discuss ideas and opinions. /r/Politics is simply a joke at this point in time and will be until Reddit removes the offending moderators and works to instill some basic Kindergarten rules of “play nice with others” and “don’t call people Nazis just because you don’t like what someone says”.",
"Because politics and politicians in general just suck.",
"I find the problem being that it is the type of politics which is pretty nonacademic, reactionary and sensationalist. So they tend to discuss things in a very aggressive way which never leaves a good result of any debate, they tend to base their opinion more on gut feeling and fail to look at the details and causes rather than just being outraged at x forcing this on y. It's all american politics and very partisan, liberals and conservatives who stick very much to their party lines. \nAn example was \"That's it, Congress has made Pizza a vegetable\". Although eating it counted as a fruit portion, there was a lack of analysis of why it was being seen as a fruit portion (the tomato pureé and the lobbying power, as well as an ideological battle against the big state), instead it was quite a big shouting match about something that wasn't so well defined.",
"I should add that that r/politics constantly leaks to other subreddits, and that is annoying for some people.\n\n "
]
} | []
| []
| [
[],
[],
[],
[],
[],
[],
[],
[],
[],
[],
[],
[],
[],
[],
[],
[],
[],
[]
]
|
|
5eimdc | during the cold war, how did the united states (and by extent, the soviet union) know that a nuclear missile was fired at thier respective country early enough so that they could deploy countermeasures? | explainlikeimfive | https://www.reddit.com/r/explainlikeimfive/comments/5eimdc/eli5during_the_cold_war_how_did_the_united_states/ | {
"a_id": [
"dacokyt",
"dacouc7",
"dacpro0",
"dacqurq",
"dacu7r9",
"daczyhc"
],
"score": [
3,
3,
3,
8,
2,
4
],
"text": [
"Both countries deployed radar stations and other surveillance set ups around each other, looking for the distinctive sounds or signatures of a missile launch. This wasn't a perfect system, we nearly had some false alarms that ended humanity.",
"Radar systems. Also the \"counter measures\" we're launching your own. There wasn't a viable set up to completely intercept a massive incoming nuclear attack. ",
"ICBMs weren't really a threat b until around 1960. For the first part of the Cold War, bombers were the real threat and much easier to detect in advance.",
"A combination of huge long range radar stations and satellite early warning systems.\n\nYou can't really hide a rocket launch from a satellite. Large rockets like ICBMs have a *massive* thermal signature that is easy to spot. That is why the US and USSR (and now Russia) notify each other of peaceful rocket launches in advance.",
"The USA had the [Ballistic Missile Early Warning System] (_URL_0_), a series of powerful RADAR stations at Anderson (Alaska), Thule (Greenland), and \nFylingdales (Yorkshire), and others. ",
"There were no countermeasures. Neither the USSR nor the US had any method whatsoever to shoot down or even deflect an incoming nuclear missile. And still don't even today.\n\nThe only response to an incoming nuclear missile is to launch a nuclear missile of your own back. This is known, in complete seriousness, as the MAD doctrine, \"Mutual Assured Destruction.\"\n\nAnd MAD is how it's still working, right now."
]
} | []
| []
| [
[],
[],
[],
[],
[
"https://en.wikipedia.org/wiki/Ballistic_Missile_Early_Warning_System"
],
[]
]
|
||
69mjxt | Do rainbows also have sections in the infrared and/or ultraviolet spectrum? | askscience | https://www.reddit.com/r/askscience/comments/69mjxt/do_rainbows_also_have_sections_in_the_infrared/ | {
"a_id": [
"dh7qufa",
"dh7sjap",
"dh863j1",
"dh8jjkf",
"dh8qv4j"
],
"score": [
4242,
564,
114,
12,
5
],
"text": [
"Yes, definitely! You can easily see that in [this series of images](_URL_0_) taken in the ultraviolet (UV), visible, and infrared (IR) parts of the spectrum. As you can see there is a UV band below the violet edge and an IR band above the red edge, which you obviously can't see with the naked eye.\n\nThis result is exactly what we would expect. The way rainbows work is that when sunlight strikes water droplets suspended in the air, part of the light is reflected at the air/water interface at the back of each droplet, [as shown in this diagram](_URL_1_). Since water is dispersive (meaning that the rerfractive index varies by wavelength), each droplet effectively acts as a small prism spreading the white light into its spectral components. Now our eyes our only sensitive to the visible (by definition), which is why a rainbow looks like a colorful transition from violet to red. However, sunlight also contains [IR and UV components in addition to visible light](_URL_2_). While the water droplets absorb some of this light, much of it ends up reflected, as part of this extended rainbow that you can see from the IR and UV images posted above.",
"They do, and this is how William Herschel discovered infrared radiation.\n\nHe was conducting an experiment to see if the different colors of a rainbow (as produced by a prism) had different temperatures. He placed a series of thermometers in the different colors of the rainbow and another to the side, next to the red in the rainbow, as a control. When he discovered that his control thermometer indicated a higher temperature than any of the others, he correctly theorized that there was an invisible form of light that was being split off past the red component.",
"Yes! In fact, that's how we accidentally discovered IR radiation. William Herschel was experimenting with color temperature to see if any color was \"hotter\" by breaking sunlight up with a prism and putting a thermometer in the different color zones. He put a thermometer next to the red as a control thinking there was nothing there, but to his surprise that one got the hottest. If he had put it next to the purple side, we may not have known about the extended spectrum for another 50-100 years. ",
"Well, it's been about 45 years, but I remember from school that the way infrared was originally discovered/proven was when some scientist put a thermometer just past the red on a spectrum coming through a prism, and the temperature rose.\n\nAnd while I imagine in works for lower frequencies too, I know with higher frequencies the ability to lens them gets lost because of the shortness of their wavelength. Which is why x-ray cameras don't have lenses.",
"Yes, this is actually how we discovered IR/UV. (Newton? I think?) Placed thermometers under a spectrum to measure each colors' temperature, believing some might be warmer than others. His control thermometers on either end of the spectrum also heated up, revealing there were components to the spectrum that humans couldn't see."
]
} | []
| []
| [
[
"http://i.imgur.com/NZjWfWT.jpg",
"http://i.imgur.com/tcLYm5o.png",
"http://i.imgur.com/KbIrTzA.png"
],
[],
[],
[],
[]
]
|
||
2nos65 | how can we identify different instruments playing at the same time if it is the same air that is vibrating? | I mean that if 2 instruments are palying at the same time, they are all sending vibrations to the air... doesn't this make a unique sound or unique vibration? If so.. how can we identify the different instruments playing? | explainlikeimfive | http://www.reddit.com/r/explainlikeimfive/comments/2nos65/eli5how_can_we_identify_different_instruments/ | {
"a_id": [
"cmfk7s8",
"cmfk9ew",
"cmfkbod",
"cmfkiar",
"cmfl8si",
"cmfld6b",
"cmflg1y",
"cmfmjt1",
"cmfmoa3",
"cmfpf6k",
"cmfrs9a"
],
"score": [
5,
51,
14,
5,
6,
4,
2,
2,
10,
2,
3
],
"text": [
"The amazing human brain takes all the sound waves in as a raw input just as you say, but the truly spectacular thing is that it can decode this information for us into separate sounds all playing at once. I don't think neuro scientists have discovered exactly how the brain does such a feat but it must use previous knowledge of sounds to do so. \nFor example if you do not listen to electronic music then it would be quite difficult for you to pick up on two different sounds of synth playing over he top of one another, where as someone who does would find it quite easy to distinguish the two sounds. ",
"Vi Hart explains it better than I ever could. Basically, it's what your ear was made to do. _URL_0_",
"I've always wondered the same thing. When you overlay two or more colors, you see the new blend they create, you don't see the individual colors. But when multiple sounds are happening at the same time, they pretty much remain distinct and don't blend.\n\n(Yes, I know that all I did was re-phrase your question.)",
"Simple version:\nYour cochlea is the part of your ear that actually turns vibrations into neutral impulses and enables you to hear. Everything else (I.e. your ear) is just there to get sound to the cochlea. \n\nThe cochlea is constructed so that different parts respond to different frequencies, so when you hear a complex sound (with many different frequencies), your cochlea detects each individual part of the sound separately! Your brain is pretty good at taking multiple different aspects of a stimulus and recognizing that they are part of the same stimulus... for example, when you see an orange your brain perceives the shape and color of the orange separately, but can easily 'put them together.' Your brain can do the same thing with sounds: it can 'put together' the different sounds in different frequency ranges made by a particular instrument.\n\nSource: master's degree in neuroscience.",
"To some extent, we can't identify the two different instruments playing. There are two major attributes of the sound sources that make it possible:\n\n1) Stereo placement\n\n2) frequency spectrum\n\n3) \"filling in the gaps\" over time i.e. \"interpolation\"\n\nGiven two instruments where 1) and 2) are similar or the same, your ear really will \"hear\" only the loudest one.\nBut then, over time, your ear and brain \"figures out\" that there are two instruments because of differences in what they are playing, and fills in the gaps so that you appreciate both instruments.\n\n\nThis is largely how mp3 compression works, in that it removes information that your brain doesn't need to follow the music.\n",
"All the sound waves fill the air at the same time and become in super position with one another. (Essentially summed together) The resultant signal carries the all the information of all the sound waves together. Reaching the ear it can then be split back into its original constituent frequencies and interpreted by the brain. This can be done mathematically with a Fourier transform. I assume the brain has a specialized circuit for doing this.\n\nThe point of my statement is that this is nothing mystical, the mathematics underlying it are used every day.",
"Sound is more than just the basic note. The tone of an instrument has various characteristics. \n\nAttack: the initial intensity of the sound, often louder than the succeeding sound. A plucked guitar string has a strong attack. A bowed violin string has a soft attack.\n\nDecay: how quickly the sound fades after the initial attack. A plucked string instrument will have a fast decay (in the absence of electronic processing). A bowed violin string has a slower decay. A banjo has a fast decay, as does a drum.\n\nSustain and release: the later parts that describe the sound.\n\nIn addition, instruments have overtones. When you play a note on an instrument it also produces higher tones, often multiples of the primary tone. So, a note played at 100 hz also has tones played at 200, 300, 400, 500, 600, etc. The loudness of each overtone contributes substantially to the sound of the instrument",
"[This](_URL_0_) man is blind and autistic but has an amazing ability to hear and play piano. In the clip a guy plays upwards of 10 simultaneous notes and Derek plays them all back after a few seconds.",
"There's something called timbre. Wikipedia defines it simply as\n\n > In simple terms, timbre is what makes a particular musical sound different from another, even when they have the same pitch and loudness. For instance, it is the difference between a guitar and a piano playing the same note at the same loudness. \n\nNow, it's my understanding (and I could be wrong on the physics of it) that what's actually happening is this.\n\nEach \"note\" you hear is a specific frequency, right? But the thing is, that frequency also includes all sub-frequencies (called harmonics) of the same frequency. So something vibrating at a frequency of 120hz is *also* vibrating at 60hz. Makes sense, right? (it's also vibrating at 30hz, etc.) Something that's vibrating at a certain frequency is also vibrating at half that frequency.\n\nSo here's the thing, different instruments include different levels, or amounts, of those sub frequencies. I'm not super clear on the physics part here, but basically a specific instrument can include more or less prominently those sub-frequencies. When you make a digital tone, it's \"pure\"-- it's only the specific frequency with no additional prominence on any of the sub frequencies. But analog sounds, real instruments, include them. That's called timbre, and it what makes different instruments producing the same sound sound \"different.\" It's also how we can synthesize real instruments-- we can estimate which of the harmonics/subfrequencies are more or less prominent.\n\nThat's the basic idea. Things like how the frequency resonants in the body of the instrument, or how the sound is made (strummed, struck, plucked, blown through brass) can affect which frequencies or harmonics/sub-frequencies are more prominent.\n\nedit: Okay I think I explained more why different instruments sound different when playing the same note, but if you're asking why different instruments sound different in general, it's the same idea. One instruments has different overall timbre than another one, with many contributing factors.",
"They're playing in different frequency ranges. If they're playing similar frequencies it's hard to distinguish them (and it usually sounds like ass.) \n \nThis is from a music production standpoint, not sure if it's the precise scientific answer.",
"I'm getting to this a little late but I think I've got something that'll answer your question a little more specifically. It's something I've always wondered about myself: how can a single membrane (either the eardrum or a speaker cone) reproduce the sound of an entire orchestra? How does a single speaker cone sound like both a cello and a violin (and a flute and a trumpet and everything else) at the same time?\n\nThe thing that allows that to happen is called the Fourier Theorem. The Fourier Theorem states that any wave function, no matter how complex, can be represented as the sum of a series of sine waves. Conversely, any number of sine waves, no matter how much they differ in amplitude and frequency, can be expressed as a single complex wave function. \n\nThere are some great visual examples of how exactly that works on the [Fourier Series](_URL_0_) Wikipedia page. But the general idea (and this is a bit of an oversimplification for purposes of illustrating the concept) is that complex audio information can be \"encoded\" in a single waveform that can be reproduced by a single vibrating membrane. \n\nWhat happens next has been largely covered by the other commenters. The inner ear and brain \"decode\" the single waveform into its component parts (again, oversimplification for purposes of illustration), and those parts are analyzed as a whole to determine what we're hearing. That's where timbre and frequency spectrum come into play. If you've heard a trumpet before, your brain has created a pattern recognition schema based on its timbre, the harmonics characteristic to that instrument. The brain recognizes, decodes, and interprets patterns of harmonics within the total spectrum of what we're hearing. The fact that the ear and brain can decode and interpret that many simultaneous patterns in an audio spectrum is nothing short of miraculous, but that's how it's done.\n\nThis is something that can be done visually as well, assuming you have a good spectrum analyzer (the thingy that creates a moving, visual wave based on audio data). If you play the beginning of Mahler's 5th through a spectrum analyzer, you're going to see that first trumpet melody show up as a series of spikes, representing the trumpet's fundamental frequency and all of the harmonics above it. When the trumpet plays later on over the rest of the symphony, those spikes will pop up again among the dozens of other spikes from all the other instruments, and the brain is able to recognize that particular pattern of harmonics among everything else that's going on."
]
} | []
| []
| [
[],
[
"http://youtu.be/i_0DXxNeaQ0"
],
[],
[],
[],
[],
[],
[
"https://www.youtube.com/watch?v=r6HCXx8U6Ko#t=251"
],
[],
[],
[
"http://en.wikipedia.org/wiki/Fourier_series"
]
]
|
|
2x3vbn | what is the reasoning behind above-ground power lines? germany buries them, mostly, but the usa keeps them exposed to the elements. | They're unsightly, and result in power outages during storms if above the ground. | explainlikeimfive | http://www.reddit.com/r/explainlikeimfive/comments/2x3vbn/eli5_what_is_the_reasoning_behind_aboveground/ | {
"a_id": [
"cowms9w",
"cownea5",
"cowqecx"
],
"score": [
4,
6,
3
],
"text": [
"A few factors:\n\n1. It's cheaper initially, even if it has significant costs due to more damage.\n2. When a part of a 10 mile power line breaks, the power company has no idea where it happened. They drive down the road to find out where the failure happened. If the wires are underground that doesn't work, lots of wasted digging to resolve that issue.\n3. American is huge, and so there are a lot of areas where there are just huge stretches of power lines without connecting to anything, that makes #2 worse as well.",
"Germany only buries the low-voltage power lines between transformers and the homes. The power lines between the transformers, switching stations and power plant are above-ground lines most of the time.",
"I have underground lines in my neighborhood. As I fount out several years ago when we lost power, repairing underground lines is the homeowner's responsibility. If they were overhead lines, the power company would do it.\n\nSo I had to spend $3,000 to have my lines dug up and replaced."
]
} | []
| []
| [
[],
[],
[]
]
|
|
eukkeu | how do free mobile games make money when all the ads in the game are from other free mobile games? | Is it just a closed loop of game companies paying eachother or are they getting money from somewhere else? | explainlikeimfive | https://www.reddit.com/r/explainlikeimfive/comments/eukkeu/eli5_how_do_free_mobile_games_make_money_when_all/ | {
"a_id": [
"ffpy9k0",
"ffq0qfu",
"ffq3wpq",
"ffq5qo1",
"ffq6by7",
"ffq7ef4",
"ffqcz5m",
"ffqe8bb",
"ffqeg4z",
"ffqet7g",
"ffqhevx",
"ffqny9f",
"ffqoq93",
"ffqotex",
"ffqrolv",
"ffqx4ck",
"ffr02ro",
"ffr04y5",
"ffr0gdg",
"ffr2cuz",
"ffr3gts",
"ffr3kqd",
"ffr6tvn",
"ffra0h1",
"ffrn8h7",
"ffrsvl6",
"ffs5tbl",
"ffsms2e"
],
"score": [
359,
6071,
50,
14,
4,
9,
19,
2,
29,
5,
3,
2,
95,
3,
8,
2,
2,
2,
2,
2,
2,
2,
2,
2,
2,
2,
2,
2
],
"text": [
"In game purchases are the primary income stream for that kind of game. Advertising is a thing too - last quarter [Zynga reported 18% of revenue in advertising](_URL_1_) - but it's not what the business is built on. \n\nIt's worth remembering that a large portion of the income in this kind of in game purchase comes from a small portion of the player base, which is often [directly targeted with addictive mechanisms](_URL_0_).",
"Free mobile games make money primarily in three different ways: \n\n(1) offering in-app purchases usually used by their ‘whales’ (i.e - 20% of their customers who spend a significant amount of money on the game and keep it alive for the rest of the non paying users).\n\nEdit: Just wanted to clarify that the 20% isn’t supposed to be an exact figure, it’s a reference to the [Pareto principle](_URL_0_) also known as the law of the vital few. I’m aware that the actual amount of users who pay can be significantly fewer.\n\n(2) Is by running ads, usually bought as a advertising package (meaning you don’t have to you choose a specific game to advertise on you can just specify which customers you aim to target and how much your company is willing to spend on it and it is accordingly shown to such users. Alternatively, if your game if quite similar to another one in the App Store, you can specifically target that app as you might find a lot of users with the same interest all conveniently in one place) and shown to you based on your past user data and preferences from the App Store. They always make sure to give you the option to remove ads with a small fee - which appeals to our human need to remove a ‘pain point’ (an inbuilt aspect in many free to play mobile games that slows down the player or tries to push them towards making paid purchases - these include things like in-game wait timers). \n\n(3) That other major way they make money is buy selling your user data to other third parties (businesses) as user data is an extremely powerful tool for companies to have because it allows them to understand you and how to market and target you as a customer.",
"They get paid to host a bunch of third party SDKs on the backend and hidden away from users. There's an interesting article from the POV of a small-time app dev [here](_URL_0_)",
"What about games that only monetize with Ads ?",
"May I stop your scrolling to tell you about an amazing game? It's called Raid Shadow L...",
"They get money in various ways. Running ads of other games earns them ad revenue paid by the other game companies. The more popular a game gets the more likely people are to subscribe to them, get membership or premium features, which gets em even more income. And then there's other in-app purchases like currency to buy in-game items, to speed up in-game progress, unlock access to new areas or items or quests or characters, etc.",
"Some of these ads are so dumb....showing gameplay that has nothing to do with the advertised game. They should have rules and regulations for the dumb shit they’re allowed to show....I’m talking about you Homescapes",
"Just an fyi if anyone sees this. Disabling wifi and data before opening a game should disable adds especially if its one of those simple addictive games that have adds every level",
"Mobile game maker here, I'll try to explain to some extend (sorry for my english).\n\nFirst thing after you create your free mobile game is to find people downloading your game. Most common thing to do is to \"buy\" players, you pay money for the ads to appear on any other game or platform, and it will cost you money for each download you get, we will call this Cost per Install (CPI)\n\nBut each time an Ad appear in our game, we also got money from the Ad Network (Admob, Ironsource,...). The money, of course are from anyone who paid to get users. So basically it's a loop with Ad Network tries to improve their AI to be more effective (reach the right people), and game makers try to encourage player to watch more Ad (or IAP) so our revenue from each user higher than CPI\n\nWe game makers also use players data but not for sale, just to track and improve our game based on the data we have so our game will have better retention rate",
"Don't forget that _you're_ seeing game ads because you're being targeted. Not everyone sees all game ads",
"Even if this were the only way they could make money (it isn’t), the vast majority of game “companies” lose money buying ads for their games. They’re actively pumping money into the ecosystem while the top maybe 1% actually make money.",
"Oh I have some background here. Finally an area I can share some expertise on. \n\nGenerally speaking, every digital company is connected at this point in the advertising world. \n\nWhen you go to your local grocery store and buy some coke and use your “shoppers club” card, that is then sold to one of the thousands of companies interconnected to what is known as DMPs. These DMPs then hold a lot of this data to be resold in some sort of targeting tool. \n\nThe idea of “click through” for paid ads is old, and largely not used due to inaccuracy. They use impression methods for payouts on ads. Looking at one of these partnering networks to see if the ad was effective, then getting paid out on said ads. Effective as in you purchased the good they sold. Using one of the thousands of available IDs they use to tie you, to the marketing campaign. \n\nThe digital advertising world is a vast network of companies buying and selling data in the same method. One example of these networks is the \n\n_URL_0_\n\nSo as indicated earlier on ads, thats how some of the revenue is being generated.",
"I worked in the mobile free to play industry. The current answers I see don't reflect what I experienced.\n\nAds are nearly worthless. Usually 1 to 2 cents per view. So if we have a user base of 5000 active users a day and they each watch 5 ads it's $250. Which is not terrible but that number is basically the maximum many dev studios would see.\n\nAnd more importantly ads give resources without training purchasing behavior which is discouraged. You will often see limits on the number of ads you can volunteer to watch for instance. This is because it's not valuable to allow you to farm currency that way. It's a supplemental income, not the main focus.\n\nWhaling would be a great term for how free to play games actually make their money.\n\n99% of a studios income will be from a very small percentage of the userbase. These people are the whales, they spend literally thousands to tens of thousands of dollars on free to play games. They do this because free to play games focus on including every single mechanic they can to encourage addictive behavior. With many strategies pulled right out of casino textbooks.\n\nIt's very important to gate time. You can't have people burning through content. So time sinks need to be made. All resources are monitored to make sure that their are good gold/gem sinks. So that players are properly on the verge of just having enough to push them to buy more.\n\nAlso it is very important to encourage purchasing as early as possible to normalize the behavior. You'll be given premium currency as part of the FTUE (first time user experience) and this will maximize the likely hood that you will be tempted to purchase more premium currency.\n\nLots of people will spend a few dollars here or there. But all of the players that spend a reasonable amount combined likely do not add up to what the biggest whale is spending.\n\nSo when trying to understand these markets, remember they are whaling industries trying to land the big one. This is why so much of what they do seems so unappealing. They don't care about you shrimp, because the whales 'love' it. (Or at least they will drop their money endlessly and that's the important bit.)",
"Wow hey a topic I'm an expert at!\n\nWell, the basic idea is that when you show an ad for your game in another game, you're paying in CPM, or 'Cost Per Mile' which means 1000 views. \n\nOne ad can get millions of views per day or even hour if the budget is high enough.\n\nThe goal then, is to get the users to come to your game and filter through your business model--whether that be in game purchases, in game ads, subscriptions, rewarded ads, or offer walls.\n\nIf you run an action game called Ninja Attack and I have an RPG game called Ninja Hero Gaiden, chances are players of your game might want to play my game too. So I would contact an advertising company like Google, Facebook, Unity, or Ironsource to get advertising. \n\nThe key is the Business Model. Do I make my money from advertising or from in game purchases?\n\nIf it's from advertising, the goal is simple, make you play long enough to see more ads than I paid for. \n\nMore likely thought, it's to get you to come and play the game and spend money. \n\nOne user out of 100 coming and spending $1 will net me 10x what I spend. Worth it.",
"South Park season 18 episode 6 is about Freemium gaming.\n[Click here for the episode](_URL_0_)",
"For mobile games:\nGame companies spend most of their money on advertising and wages.\n\nThey get most of their money from in app purchases and from investors.\n\nNew mobile games spend a ton of money on advertising and get all their money from investment (can’t get IAP before releasing). So if there are 10 new mobile games all running ads for each other, you are seeing investment money being used to buy ads for games in other games.",
"In-game monetization, at least in some cases, you'll be surprised the amount of money some people can spend in this. One friend spent over $500 to pull an specific character in FE Heroes.\n\nI've played Clash Royale, I'm not proud of doing it but I have no regrets neither because I've spent around $300 total during these 3.5 years playing it. However about 2 years ago when they released Graveyard (Or some other cards), there were reports of people cashing $20k or more for maxing out that card in Day 1, and I meant not only 1 or 2 people, but hundreds or more. This last week they released a \"special pack\" with some stuff, the stuff is kinda expensive IMO, because everything could be obtainable in the store for much cheaper or easier methods, but the lure is an exclusive \"emote/emoji\", and I've seen dozens of people using it while playing against me, so basically they paid $50 for an emoji.",
"They advertise and then the advertisers advertise, when they run out of advertisements they have to buy more. Eventually they buy and circulate some Paddy's Dollars. The money keeps moving in a circle, thus creating a self-sustaining economy.",
"In actual ELI5:\n\nBecause ads cost money, any mobile game that can afford to make an ad must not actually be completely free. If it were really free, they could afford the ad. So they must make money some how, and usually it's by making the app free to try and then some parts of the game actually cost money later.\n\nIf you want a game, look for ones that cost money, ask me if I'll buy it for you, and if I approve, I'll get it for you. That way you get the entire game at once and also won't see ads for other games in your new favorite game!",
" > Is it just a closed loop of game companies paying eachother \n\nHow would this even be possible?\n\nMicro transactions is how freemium games make the most money.\n\nWhales get addicted to games and drop thousands of dollars per year on them. That's their main source of income. Its why you have obnoxious games where you pay 100 bucks for a mediocre skin on a bad mobile game.",
"And while it's not always true, there is a portion of this old adage that applies:\n\n\"If you're not paying, YOU are the product\"\n\nAs mentioned elsewhere, these games are often largely making money off of whales - a small percent of the userbase who make large purchases at the cash shop for overwhelming advantages in-game.\n\nTo keep the Whales playing, they need a never-ending supply of sheep for them to roll over. What good is buying an invincible army if you can't gank noobs? These people don't want to fight another whale's giant army, they want to embarass a free-to-play noob and crush them completely.\n\nSo you, the free-to-play player, goes through the tutorial, maybe gets one or two matches against other free-to-play people - then boom - they throw you at a whale or two in a matchup you literally cannot win.\n\nSo take these free-to-play mobile games with a grain of salt. The more pay-to-win they are, the less you need to invest emotionally into actually winning.",
"You greatly underestimate the amount of people literally addicted to pouring money into these games. They single-handedly keep those mobile game schemes running.",
"Good question. All roads eventually lead back to in app purchases. You may well get an ad for a game with no IAP (just more ads) but if you follow the chain it would eventually lead to IAP (or other real revenue apps/products like an item on Amazon, a credit card, online poker, etc).",
"To understand this, first you have to understand that these ads are managed by \"Ad Networks\". Google, Facebook, Apple, all have their own ad networks. These ad networks decide what ad to show where. And there are advertisers who just want people to know about their app/game. So, any games that show such ads have integrated one or more such networks.\n\nLet's say you're playing Subway surfer, and you see an ad for temple run, and if you install temple run using that ad, temple run guys pay some amount to the ad network because the network helped with a new install for their game. And in turn, subway surfer gets a share of the said amount.\n\nYou can actually integrate such networks in your own apps and start showing ads. If any of your app users found anything interesting for themselves in any ads your app shows, well, good for you.\n\nNow the question arises, what if no one ever clicks on any ads ever shown by your app? Well, whenever any ad is shown in an app, it's called an \"impression\". The ad networks also pay on the basis of per 1000 impressions. \nThis amount is very negligible and also depends on the region the impression was made. Also the rate of per 1000 impressions vary (like stock market). \n\nAnother way how games make money is In-app purchases. You pay real money to buy stuff in the game. It's just that simple. \n\nEven though, ad revenue seems promising at first, it's actually not. To generate a decent amount of revenue through ads would require millions of downloads and god knows how many impressions.",
"Even if the game that you are playing does not have microtransactions the games in the ads will most likely have some pay to win scheme.",
"Micro transactions. Extra loot boxes extra packs of cards extra weapons extra money for the in-game transactions. All of which make them a shit ton of money.",
"They barely make any money from ads - most income is from in-game purchases available. \n\nSpotify has a similar problem. You may notice on Free Spotify most the ads are just for premium spotify. They've always had trouble making the free version profitable due to low ad income, but premium is what makes spotify its money. Free mobile games with premium options / in game purchases work the same way.",
"Can anyone explain to me how the ad for gardenscapes, is completely different from the actual game?? I finally gave in and downloaded it because I thought man I love a good brain teaser, these look like fun puzzles and it’s all some repair game with a candy crush game??? Someone help me I’m baffled here"
]
} | []
| []
| [
[
"https://www.youtube.com/watch?v=7S-DGTBZU14",
"https://investor.zynga.com/financial-information/quarterly-results"
],
[
"https://betterexplained.com/articles/understanding-the-pareto-principle-the-8020-rule/"
],
[
"https://www.cyberpunks.com/small-app-developer/"
],
[],
[],
[],
[],
[],
[],
[],
[],
[
"https://digitaladvertisingalliance.org/participating"
],
[],
[],
[
"https://southpark.cc.com/full-episodes/s18e06-freemium-isnt-free#source=6154fc40-b7a3-4387-94cc-fc42fc47376e:c6cbd5e3-7eae-4cc3-94b7-119c8d412f99&position=6&sort=!airdate"
],
[],
[],
[],
[],
[],
[],
[],
[],
[],
[],
[],
[],
[]
]
|
|
ks1r6 | how do man-made canals work? like the suez, erie, panama canal? | How are they built? their purpose? | explainlikeimfive | http://www.reddit.com/r/explainlikeimfive/comments/ks1r6/eli5_how_do_manmade_canals_work_like_the_suez/ | {
"a_id": [
"c2mqlv9",
"c2mqlv9"
],
"score": [
5,
5
],
"text": [
"Their purpose is to move boats between places that don't directly connect. This saves a lot of money on shipping things, because then you don't have to steer your boat all the way around Africa or South America if you're shipping stuff from Greece to India or from New York to San Francisco. They are built, as you'd expect, by digging out the dirt between the two bodies of water.\n\nMost canals have locks, which are doors that close and allow the water level to change by pumping, kind of like an airlock but for water. This accounts for the water level differences between bodies of water. The Panama and Erie canals both have locks because of the need for changes in elevation during the crossing. In contrast, the Suez canal has no locks because there is no perceptible change in the elevation of the Red and Mediterranean seas.",
"Their purpose is to move boats between places that don't directly connect. This saves a lot of money on shipping things, because then you don't have to steer your boat all the way around Africa or South America if you're shipping stuff from Greece to India or from New York to San Francisco. They are built, as you'd expect, by digging out the dirt between the two bodies of water.\n\nMost canals have locks, which are doors that close and allow the water level to change by pumping, kind of like an airlock but for water. This accounts for the water level differences between bodies of water. The Panama and Erie canals both have locks because of the need for changes in elevation during the crossing. In contrast, the Suez canal has no locks because there is no perceptible change in the elevation of the Red and Mediterranean seas."
]
} | []
| []
| [
[],
[]
]
|
|
3tjvgf | why are electoral votes all or nothing(for the most part)? why are they not divvied out representative to popular vote? | explainlikeimfive | https://www.reddit.com/r/explainlikeimfive/comments/3tjvgf/eli5_why_are_electoral_votes_all_or_nothingfor/ | {
"a_id": [
"cx6rgb3"
],
"score": [
2
],
"text": [
"Because each state chooses to use all of its electoral votes to elect the person the majority of the electorate in that state voted for. Each state chooses to not divvy up its electoral vote."
]
} | []
| []
| [
[]
]
|
||
3n2we6 | Why is the strong force repulsive at small distances? | I'm an A-level student so don't throw too much maths at me but the other day my physics teacher said that at very close proximities the strong force acted to repel particles instead of attract them. Why? | askscience | https://www.reddit.com/r/askscience/comments/3n2we6/why_is_the_strong_force_repulsive_at_small/ | {
"a_id": [
"cvkpu8c"
],
"score": [
2
],
"text": [
"My favourite [link](_URL_0_) for \"why\" questions, especially of this nature.\n\nAlso the Strong force itself isn't repulsive over short distances, just an effective description of it for use between nucleons is."
]
} | []
| []
| [
[
"https://www.youtube.com/watch?v=qjmtJpzoW0o"
]
]
|
|
478d9y | What are your favorite historical books about great empires and/or historical figures. | I want to learn more about world history but don't know where to start. I'm guessing that great empires and individuals are a good place. | AskHistorians | https://www.reddit.com/r/AskHistorians/comments/478d9y/what_are_your_favorite_historical_books_about/ | {
"a_id": [
"d0bi09w"
],
"score": [
2
],
"text": [
"Caesar: Life of a Colossus\nAdrian Goldsworthy\n\nFantastic well researched life of quite possibly the worlds most interesting man that is a great approachable read.\n\nAmerican Ceasar\nWilliam Manchester\nSometimes a bit heavy but riveting all the same. Makes this complicated controversial man understandable and appreciable on his merits."
]
} | []
| []
| [
[]
]
|
|
1tvxzq | What did pre-Columbian indigenous nations do with the dead bodies of people who they fought? Did they just leave them on the field? | Say the Aztecs and the Tlaxcalans fought a skirmish, leading to a total rout of the Aztec forces. What would the Tlaxcalans have done with the bodies? Would they have looted them and returned home? Buried them as honored rivals? Parade their heads on sticks as they marched around town? What was the protocol here for fallen enemies? | AskHistorians | http://www.reddit.com/r/AskHistorians/comments/1tvxzq/what_did_precolumbian_indigenous_nations_do_with/ | {
"a_id": [
"cec781k"
],
"score": [
6
],
"text": [
"I can't speak to the specifics of Aztec battles, but the huge diversity of pre-Columbian nations over thousands of years and across two continents gave rise to a variety of methods of treating fallen combatants. There will be as many answers to this question as there are distinct cultural traditions and skeletal remains with evidence of interpersonal violence.\n\nFor example, the Crow Creek Massacre is one of the largest pre-Columbian skeletal assemblages attributed to a violent encounter in North America. Sometime around 900 CE the ancestors of the Mandans built several earthen structures in the south central portion of modern-day South Dakota. Eventually the Caddoan-speaking ancestors of the Arikaras replaced the Mandans (no indication the replacement was by force) and increased the settlement to a decent-sized community of roughly fifty-five earthen lodges. \n\nFor unknown reasons, in roughly 1325 CE, at least 486 individuals at Crow Creek were violently killed. Evidence of the massacre was discovered in 1978 when skeletal remains eroded out of a fortification ditch. Analysis of the remains indicated extreme violence during the attack. I will spare the gory details, the destruction was quite complete, and trophies appear to have been taken by the attackers. The state of the remains indicates they were exposed for a period of time and subject to the typical animal scavenging expected on the Northern Plains. Some time later the remains were gathered together, placed in a communal burial, and covered with a thin layer of clay from the nearby river. We don't know if survivors of the attack, their kin, or someone altogether different were responsible for cleaning up the battlefield.\n\nFor all the brutality of the treatment of the victims at the Crow Creek Massacre, biological archaeologists can illustrate the care taken with the burial preparations for victims of interpersonal violence across the Americas. As in our modern culture, treatment of the dead varied according to immediate circumstances and local traditions."
]
} | []
| []
| [
[]
]
|
|
4gv5a4 | why do we get random bouts of euphoria? | Why do people get random bouts of happiness through out the day even though we may be having a terrible day? | explainlikeimfive | https://www.reddit.com/r/explainlikeimfive/comments/4gv5a4/eli5_why_do_we_get_random_bouts_of_euphoria/ | {
"a_id": [
"d2kzbih",
"d2l3udj"
],
"score": [
2,
3
],
"text": [
"Well, there are a couple of reasons. Number one, when we lower our expectations for something, we are usually pleasantly surprised with the outcome. An example of this is a rainy day. On rainy days, we wake up and go, \"Looks like today is gonna be crappy.\" In thinking like this, we have already lowered our expectations for the day. Then, when you have a good interaction with someone, or something goes your way, it seems even better as you realize, \"Hey! Today ISN'T crappy!\" \n\nThe second, and more troubling answer, is something along the lines of a mental disorder like cyclothymia or bi-polar disorder, which is characterized by marked changed in mood, both lows (depression), and highs (euphoria). \n\nThe way you've posed the question, I'm more inclined to think that the answer is the prior. You've lowered your expectations so that smaller things make you happier. Hope this helps! \n\n",
"Hmm, I've never experienced that. I've known people who had sharp mood swings one way or another, but they usually had issues like hypoglycemia, bipolar disorder, etc.\n\n I've for sure personally moved from down to up, but never very quickly, and never for no apparent reason."
]
} | []
| []
| [
[],
[]
]
|
|
3jit73 | why does ssds have a specific number of reads/writes. | As the title goes.
Thanks for any kind answer. | explainlikeimfive | https://www.reddit.com/r/explainlikeimfive/comments/3jit73/eli5why_does_ssds_have_a_specific_number_of/ | {
"a_id": [
"cupkvvp",
"cupm5b0",
"cups6xd"
],
"score": [
21,
8,
3
],
"text": [
"The way my boss describes it is like this:\n\nWhen you write to a cell, you're basically shooting it with a shotgun (electrons). This makes it return a different signal when you apply power (to read it). \n\nWhen you erase that cell, you take a giant magnet to rip the bullets out. Unfortunately some of the target comes out with the bullets. \n\nEdit: I work in Intel's Non-Volatile Memory Solutions Group",
"When you flip a bit on an SSD, you are essentially forcing electrical current across an insulator. \n\nEventually, the insulator wears out.",
"SSDs use \"flash\" memory, which uses a specialized transistor to store data. The data is written by forcing a charge (electrons) onto a \"floating gate\". It is called floating because it is entirely surrounded by insulators. When something in a circuit isn't connected, it is called \"floating\". \n \nThe electrons are forced onto the floating gate by putting an electric field on either side of it. The process of writing the cell this way, and the process of erasing it by removing those charges causes a bit of damage to the thin insulator around the gate. Eventually, it starts to wear out and charges can leak off the gate. Once it loses too much charge, it can no longer be reliably read. \n \nEven when some of the bits in a flash memory start to wear out, it may still be usable for quite some time. There are a variety of error correction schemes used to overcome having a few bad bits. All of the bits don't wear out at the same rate, so quite a few bits can go bad and the chip can still be usable. \n \nEven when too many bits in a word go bad, the SSD can just tag that location as unusable and start using spares instead. And when too many words in a block or a page or bad, there can be spares for them, too. Once all of these things are exhausted, the chip is no longer considered reliable. \n \nFlash memory makers do a lot of work to characterize the \"read/write endurance\" of their chips, and the SSD maker will do more on top of that. So they have a pretty good idea of how many reads/writes can be done. But that still has a fair amount of variability, since it is dependent upon many factors, such as the nature of the data that was written, the manufacturing quality of each chip, etc. "
]
} | []
| []
| [
[],
[],
[]
]
|
|
322sd2 | Why do so many aerosol products use things like propane and butane as propellants? Why not air or nitrogen? | Title pretty much says it all. Why would they use something so flammable? Is there some reason to not use compressed air? | askscience | http://www.reddit.com/r/askscience/comments/322sd2/why_do_so_many_aerosol_products_use_things_like/ | {
"a_id": [
"cq7e2t7"
],
"score": [
44
],
"text": [
"Propane and butane are liquids when compressed. This allows a lot of the gas to be stored in a small space. Normal compressed air is still a gas, and has to be compressed at much higher pressures to even approach the compact storage of the usual propellants. "
]
} | []
| []
| [
[]
]
|
|
7x9iye | In WWII, did anyone become an ace as either a pilot or gunner of a bomber? | What about in WWI? I'm also curious about gunners in 2 seat fighters. | AskHistorians | https://www.reddit.com/r/AskHistorians/comments/7x9iye/in_wwii_did_anyone_become_an_ace_as_either_a/ | {
"a_id": [
"du6qwg5"
],
"score": [
3
],
"text": [
"There are certainly gunners credited with five or more kills. Albert E. Conder's *The Men Behind The Guns: The History of Enlisted Aerial Gunnery 1917 - 1991* has a section, \"Some of the Aces\"; Conder says \"The thousands of enemy fighters downed by Gunners were counted as a \"team\" effort, rather than crediting individual gunners. The AAF claimed that record keeping was to [sic] difficult.\" He nevertheless goes on to list a number of \"aces\", including S/Sgt Donald Crossley with 11 kills, S/Sgt Michael Arooth with nine, S/Sgt Benjamin Warmer with nine (seven in one mission) and several others including S/Sgt John Quinlan, tail gunner of the Memphis Belle who later flew on B-29s, credited with five German and three Japanese fighters.\n\nAir-to-air victories are enormously difficult to verify, though, even at the best of times with a small number of combatants and gun camera footage; with large formations of bombers, each with multiple gunners, the number of aircraft claimed downed by gunners was invariably higher than the number of actual losses suffered. Conder gives the total claims of Eighth Air Force Bombers as 6,259 destroyed, 1,836 probables, 3,210 damaged; those figures are likely out by a factor of eight or nine (see [a previous question] (_URL_3_) for further details). I'm not sure where Conder gets his individual figures from, but they're difficult to corroborate; [The Unknown Aces of the Eighth] (_URL_1_), for example, credits Arooth with \"at least 17 enemy planes\". \n\nIn the RAF [Wallace McIntosh] (_URL_2_) had the most claims as a bomber gunner (eight and one probable, though again it's hard to verify), [Peter Engbrecht] (_URL_4_) of the RCAF claimed five and a half. The highest claiming RAF gunners were those in Defiant turret fighters, particularly those of 264 Squadron; Ted Thorn (pilot) and Fred Barker (gunner) had the highest total, 12. Once again, though, the \"confirmed\" kills of 264 Squadron are likely too high, not tallying with German losses (for more details on the Defiant, see [another previous question] (_URL_0_)).\n\nI haven't got any details of World War I gunners; *Above the War Fronts: The British Two-Seater Bomber Pilot and Observer Aces, the British Two-Seater Fighter Observer Aces, and the Belgian, Italian, Austro-Hungarian and Russian Fighter Aces 1914-1918* by Norman Franks may be of some help."
]
} | []
| []
| [
[
"https://www.reddit.com/r/AskHistorians/comments/7t2j3j/how_effective_were_aft_cockpit_gun_turrets_in_ww2/",
"http://www.mightyeighth.org/blog-post-4/",
"http://news.bbc.co.uk/1/hi/scotland/north_east/6726347.stm",
"https://www.reddit.com/r/AskHistorians/comments/7sjan2/b17_gunners/",
"http://www.bombercommandmuseum.ca/engbrechtgillanders.html"
]
]
|
|
3mka2j | why should i bother getting married? | explainlikeimfive | https://www.reddit.com/r/explainlikeimfive/comments/3mka2j/eli5_why_should_i_bother_getting_married/ | {
"a_id": [
"cvfnusy",
"cvfo1aq"
],
"score": [
2,
3
],
"text": [
"Do you live in some religious autocracy where that carries the death penalty? If not probably nothing stops you.",
"From a secular point of view, a marriage is a contract between (usually two) people. They agree to pool their resources. The general idea is that both partners gain something from this, for instance financial security, a stable home situation and - not unimportant - a partner in procreation.\n\nThe government (that is, the other people who live in your country) generally find this pooling of resources an excellent idea and try to promote it by giving married couples all sorts of financial and other benefits that unmarried people (couples, single or otherwise engaged) do not receive.\n\nSo, to answer your question: nothing will stop you, and if you choose to not marry, you will miss out on (mainly financial) benefits that you would otherwise be eligible for. \n\nThe down side of marriage is that you enter a legally binding contract, and should you at any point in the future wish to disband it and no longer be held to the responsibilities that stem from it, you cannot simply quit operations and leave but have to go through a legal procedure (a *divorce*) which can set you back financially and emotionally."
]
} | []
| []
| [
[],
[]
]
|
||
80nfb1 | how do mesh networks maintain speed? | If I use range extenders the speed of the extender is cut in half to extend, unless I connect wired. How does a mesh network do the same thing without losing speed? or does it? I have an apple airport extreme and if I extend with airport express the speed is in half. what am I missing? | explainlikeimfive | https://www.reddit.com/r/explainlikeimfive/comments/80nfb1/eli5_how_do_mesh_networks_maintain_speed/ | {
"a_id": [
"dux56tx"
],
"score": [
2
],
"text": [
"Any wireless mesh network will lose speed each hop because of the overhead. Even wired networks you will lose some speed each hop but it is not near as noticeable because there is much less overhead in wired communication. Basically each wireless node takes the data off the wire, encapsulates it in the wireless packet, sends it to the next node which has to process the packet and send it back out for the next node to pick up. So if your PC is 5 nodes away then the wireless packet has to be processed 5 times coming in, and then another 5 times going back out. If there is a lot of traffic then it will be even slower because the radios will have to wait until the air is clear to send each packet. I hope this makes sense. "
]
} | []
| []
| [
[]
]
|
|
1s2i36 | why do i see product placement at big events in the form of only the name of a brand? | At the rare moments I watch soccer on TV, I always asked myself the the point of certain adds at the side of the field. I see for example 'Sony', but I don't understand how this motivates me to buy Sony products. Just seems like wasted money to place that add there. | explainlikeimfive | http://www.reddit.com/r/explainlikeimfive/comments/1s2i36/eli5_why_do_i_see_product_placement_at_big_events/ | {
"a_id": [
"cdt99ce"
],
"score": [
2
],
"text": [
"When you go to buy, say, a TV, you might see a Sony and a Magnavox. If you see Sony's name over and over again, you'll be more familiar with the brand name and think it more reputable."
]
} | []
| []
| [
[]
]
|
|
1icu4a | What was the power relationship between USSR "spy agencies" and the Communist Party up until the fall of the Soviet Union? | There have been many conspiracy theories about who controlled who and what role did the KGB play in the fall of the Soviet Union, but what do we KNOW about their power relationship? Were there ever serious incidents? Blackmail? Did the KGB choose who rose in party ranks?
And by "spy agencies" I mean both domestic and foreign spying, going beyond normal police work. | AskHistorians | http://www.reddit.com/r/AskHistorians/comments/1icu4a/what_was_the_power_relationship_between_ussr_spy/ | {
"a_id": [
"cb4q02d"
],
"score": [
2
],
"text": [
"First off, it's important to remember that the KGB didn't exist until the 1950s. The Cheka, OGPU and NKVD all came before. After NKVD director Lavrenty Beria fell in 1953, intelligence gathering was done by more than one agency, though KGB certainty had the most influence. \n\nThe answer to your question varies greatly with the time period being examined. Having the approval of the NKVD was far more important in the Stalin era than being on the KGBs good side during Gorbachev's reign. "
]
} | []
| []
| [
[]
]
|
|
16psa0 | Could a large object come close enough to Earth to cause people to temporarily float without knocking the planet out of orbit? | I suppose I know how to compute (using Newton's law of universal gravitation) how massive an object would need to be to be, say, x miles from Earth and cancel out the effects of Earth's gravitational pull (on at least one side of the planet). It does not seem like such an object would be able to pull the Earth out of its orbit around the Sun, since it's just *barely* canceling the force due to gravity on the surface. But could such an object penetrate our solar system - despite the gravitational effects of the Sun and other planets to get into that position? And how long would the effect be able to last? This I'm less clear on. I'm guessing a lot of math is going to be necessary to answer this, but I could be wrong... | askscience | http://www.reddit.com/r/askscience/comments/16psa0/could_a_large_object_come_close_enough_to_earth/ | {
"a_id": [
"c7y7m6t",
"c7y7pd0",
"c7y7pgx"
],
"score": [
4,
3,
2
],
"text": [
"For that to happen, the surface gravity of the object would have to be greater than the surface gravity of Earth. Such as object, presumably made of some sort of rocky material, would have to be of comparable mass to the Earth. The only way this object would careen past the Earth at super close range without hitting it is if it was going really, really, really fast.",
"It would have to be more massive than Earth and pass within the Roche limit.\n\nThe Earth (and likely the other body, unless it was much larger) would be destroyed by tidal forces.",
"I'm not sure, but a flyby would have devastating effects on almost every planet's orbit. \n\nTo cancel the earth's surface gravity, you'd need another massive compact object, which has at least the earth's gravity at the same distance from the earth's surface as the earth's surface is from the core. Any further, and it's mass would have to increase by the square of the distance. Such a massive body would perturb orbits horrendously, and may scatter bodies entirely out of the solar system. \n\nAlso, since the earth's surface gravity is going to be zero at the surface, above that point there will actually be a greater attraction to the flyby object- so the atmosphere will actually be stripped from the earth and flow to the new body. This is what astronomers call the Roche Lobe.\n\n[Check this applet out.](_URL_0_)\n\n > But could such an object penetrate our solar system - despite the gravitational effects of the Sun and other planets to get into that position?\n\nSure. Gravity isn't going to repel a flyby, it'll pull it in closer. \n\n > And how long would the effect be able to last?\n\nDepends on the mass and speed of the object. Unlikely that it would last more than a year (or the time for one orbit of whatever planet it came closest to). "
]
} | []
| []
| [
[],
[],
[
"http://janus.astro.umd.edu/cgi-bin/orbits/rstar.pl"
]
]
|
|
1qozzs | What was happening along the rest of the eastern front during the Battle of Berlin? | AskHistorians | http://www.reddit.com/r/AskHistorians/comments/1qozzs/what_was_happening_along_the_rest_of_the_eastern/ | {
"a_id": [
"cdf12ad"
],
"score": [
24
],
"text": [
"On the Balkans, the Croatians and Heeresgruppe F were retreating under the combined pressure of Titos (by now regular) army, the Soviets and elements of the Bulgarian army. They were mostly attempting to make contact with the British in western Austria to surrender to them in a vain hope to not be turned over to Tito or the Soviets.\n\nThe Soviets were pushing westwards from Slovakia into Bohemia and were clearing Hungary of German troops after Budapest finally fell. Towards the end of the Battle of Berlin they were at the outskirts of Vienna.\n\nIn Germany, the Soviets had reached the agreed demarcation line, and were fighting the German 12. and 9. Armee, which had fought their way to unite and then through multiple Soviet attacks and defence line to cross the river together with thousands of civilians to surrender to the Americans.\n\nThe Soviets were also advancing in Pommerania along the coast of the Baltic sea and keeping check on a small German force on the Hel peninsula (outside Gdansk) and a much larger one in Courland (what remained of Heeresgruppe Nord)."
]
} | []
| []
| [
[]
]
|
||
j2pag | can someone explain to me what a tax bracket is? | I hear the term all the time in money speak but idk what the hell it is, help me out reddit? | explainlikeimfive | http://www.reddit.com/r/explainlikeimfive/comments/j2pag/can_someone_explain_to_me_what_a_tax_bracket_is/ | {
"a_id": [
"c28n4xf",
"c28n5qf",
"c28n5yg",
"c28n7ej",
"c28n7fp",
"c28n7nx"
],
"score": [
2,
2,
2,
6,
6,
2
],
"text": [
"When declaring your income while doing your taxes, a \"Tax Bracket\" helps the government know which % of tax you owe based off of said income.",
"The government taxes your income based on a percentage of how much you make. If you make more money, the government takes a larger percentage of your pay. How does it know how much to take? It groups people into 'tax brackets' based on how much you earn. For example, if you make, say, between 10,000 and 30,000, government takes 20%. If you make 30001-50000, government takes 25% etc. etc.\n\nI hope this answers your question, and sorry if i'm completely off. I\"m not IRS agent.",
"As you have higher income, you pay more and more \"marginal income\" in taxes. Your bracket is the highest marginal rate.\n\nSo one person might bay 0% on the first 10k and 10% on the , for a total of $1000 in taxes out of 20k in income. \n\nSomeone making 30k is in a higher bracket, so they might pay 20% of the next 10k they make, which means they'll pay $3000 in taxes on a 30k income. does that help?",
"A tax bracket says the percentage of your income that is taken as income tax when you're earning a given amount.\n\nSo, for example, it might be (taken from the Australian tax scheme):\n\n* 0 - $6,000: Nil\n* $6,001 - $37,000: 15c for each $1 over $6,000\n* $37,001 - $80,000: $4,650 plus 30c for each $1 over $37,000\n* $80,001 - $180,000: $17,550 plus 37c for each $1 over $80,000\n* $180,001 and over: $54,550 plus 45c for each $1 over $180,000\n\nOne thing to note here is that each bracket gets applied in turn. If you look at the $80,001 - $180,000 bracket, you'll notice that it says \"$17,550 plus 37c for each $1 over $80,000\". This is because $17,550 is what you would have owed from the previous tax brackets if you earned $80,000. Generally speaking, you are said to be 'in' your highest tax bracket, as that informs how much of every extra dollar you earn will be taken in tax.\n\nAs an example to see how much tax you'd pay in this, assume that you earn $70,000.\nFrom the previous brackets, you'd pay $4650. Then from the current bracket, you'd pay (70,000 - 37,000) * 0.3 = $9900. So your total income tax on $70,000 would be $4650+$9900=$14,550.",
"There are four categories of tax brackets in the United States: Single, Married filing jointly, Married filing separately (same bracket as single for each spouse), and head of household. Married filing jointly provides the best tax bracket, and each category will have different dollar amounts for the tax brackets.\n\nI'll provide the single tax bracket:\n\n* 10% - $0-8,500\n* 15% - 8,501- 34,500\n* 25% - 34,501 - 83,600\n* 28% - 83,601 - 174,400\n* 33% - 174,401 - 379,150\n* 35% - $379,150+\n\nFor example. If you make $50,000 per year, the first $8,500 is taxed at 10%; all income between $8,501 - 34,500 is taxed at 15%; all income between $34,501 - $50,000 will then be taxed at 25%. \n\nSo if you calculate that, your tax bill (ignoring credits and deductions) will be $8,625.\n\nEdit: four categories, not three",
"So let's say your taxable income was $100,000. The first $8,735 of that income will be taxed at a rate of 10%. Your income from $8,376 to $34,000 will be taxed at a rate of 15%. This goes on until you reach the highest tax bracket that your income reaches. The system is set up this way so that someone just barely slipping into a higher tax rate doesn't suddenly pay much higher taxes."
]
} | []
| []
| [
[],
[],
[],
[],
[],
[]
]
|
|
iu41x | Why is there so little known about seizures? | Two of my family members have varying degrees of Epilepsy. Both are healthy, no drug use, no brain trauma, but both of debilitating seizures.
There are no answers or cures. Doctor's cannot even tell us why it happens.
What makes seizures so elusive and hard to understand? | askscience | http://www.reddit.com/r/askscience/comments/iu41x/why_is_there_so_little_known_about_seizures/ | {
"a_id": [
"c26nqdy"
],
"score": [
3
],
"text": [
"I think we know more about seizures than you'd think, and are learning more and more with the advancement of technology. It's just that this information isn't yet to the point where it is truly helpful to doctors.\n\n > Doctor's cannot even tell us why it happens\n\nDo you mean why seizures happen in general, or why they happen to your specific family members? From a research perspective we are beginning to understand more and more the genetics, heredity, childhood illnesses, previously unknown brain anomalies, etc that are associated with seizures. The problem is that there is a huge gap between where the research is and the clinical side of things. Our treatments (medications) just aren't good enough that it really matters what's causing an idiopathic epilepsy and so there's no reason for the doctors to try to figure it out (though in most cases they likely could with some degree of certainty). If we develop better medications then you might see doctors start to try to determine what is causing the epilepsy. \n\nUsing technology (MRI, fMRI, SPECT, Wada, MEG, scalp EEG, intracranial EEG, EEG with depth electrodes, Video EEG monitoring) we are getting really good at identifying exactly where in the brain the seizures originate for a given individual, and often times can surgically remove that area and drastically reduce the frequency and severity of a person's seizure disorder. \n\n > There are no answers or cures\n\nI'd agree that other than surgery there is no clear cure, but I think there are lots of answers. If you have more specific questions I'm happy to try and answer them.\n\n"
]
} | []
| []
| [
[]
]
|
|
f28oya | how does billing work for toll roads? | explainlikeimfive | https://www.reddit.com/r/explainlikeimfive/comments/f28oya/eli5_how_does_billing_work_for_toll_roads/ | {
"a_id": [
"fhaxksn",
"fhaxz9b"
],
"score": [
2,
2
],
"text": [
"Will be helpful to list the specific system you are talking about \n\nIn many cases in the US, tolls may be assessed on certain entry/exit points and certain points along the road.",
"In Massachusetts we have gantry open road tolling now, every gantry charges you a set rate per axle on the vehicle. If you don’t have a transponder “ez-pass” they snap a photo of your plate and send you a bill in the mail I believe with an added surcharge for not having the ex-pass. The bill will have the break down of every gantry you went under and what time etc."
]
} | []
| []
| [
[],
[]
]
|
||
2gpcqo | You're in a medieval European city and you need to find yourself a place to call home. Who do you talk to? A medieval realtor? | How did people acquire housing in medieval cities? Were there analogues to modern-day real estate agents, or would you just have to ask around seeing if anyone had a room to rent, or a house/estate they wanted to sell? Or would you have to find unoccupied, unclaimed land and build your house yourself?
I know that people moved around a lot less and the concept of buying or selling a house would not be a common one, but certainly there was a demand for new housing of some kind. | AskHistorians | http://www.reddit.com/r/AskHistorians/comments/2gpcqo/youre_in_a_medieval_european_city_and_you_need_to/ | {
"a_id": [
"ckld4cu"
],
"score": [
52
],
"text": [
"Obligatory \"medieval Europe can mean a lot of different times and a lot of different places.\" You should probably make your question a little more specific."
]
} | []
| []
| [
[]
]
|
|
2m3jxq | why does it hardly rain in southern california, despite being adjacent to the pacific ocean? | explainlikeimfive | http://www.reddit.com/r/explainlikeimfive/comments/2m3jxq/eli5_why_does_it_hardly_rain_in_southern/ | {
"a_id": [
"cm0p5gh"
],
"score": [
2
],
"text": [
"Proximity to an ocean is only one of the factors when determining rainfall. There's lost of deserts in the world that are ocean adjacent. \n\nThe overall weather patterns play a much more significant role. So air currents, ocean temperature, as well as land features like mountains. There's more important factors than how close you are to the ocean. "
]
} | []
| []
| [
[]
]
|
||
yvjn1 | Mom says you will catch a cold if you go out in the cold weather or rain without a coat. Do these actually contribute to getting you sick? And how? | askscience | http://www.reddit.com/r/askscience/comments/yvjn1/mom_says_you_will_catch_a_cold_if_you_go_out_in/ | {
"a_id": [
"c5z6q6g",
"c5z6v5c"
],
"score": [
5,
8
],
"text": [
"No, the temperature outside will not cause a viral illness (catching a \"cold\").",
"The 'common cold' is usually caused by a rhinovirus. Cold temperature itself does not cause illness. [However, temperature effects may facilitate viral infection:](_URL_0_)\n > Although not all studies agree, most of the available evidence from laboratory and clinical studies suggests that inhaled cold air, cooling of the body surface and cold stress induced by lowering the core body temperature cause pathophysiological responses such as vasoconstriction in the respiratory tract mucosa and suppression of immune responses, which are responsible for increased susceptibility to infections. \n\nThis is further confounded by the fact that people may stay indoors during the cold months, further allowing spread of the virus."
]
} | []
| []
| [
[],
[
"http://www.ncbi.nlm.nih.gov/pubmed/17705968"
]
]
|
||
qixcl | Instead of a space elevator, how (un) feasible would a railgun /coilgun be to shoot things into orbit? | I realize that aerodynamic drag is a huge issue for any reasonably sized payload. I/m envisioning enclosing the payload in a ferrous shell which will separate outside of the atmosphere.
I know such an endeavor would have limited returns, but it may tide us over until a space elevator can be completed; and it wouldn't have to be located on the equator. I assume we don't have such significant materials issues here either.
Why is this not being pursued?
Perhaps the air in front of the projectile could be superheated with powerful ground-based lasers to reduce drag? | askscience | http://www.reddit.com/r/askscience/comments/qixcl/instead_of_a_space_elevator_how_un_feasible_would/ | {
"a_id": [
"c3xyjlz",
"c3xyl2v",
"c3y0h8m"
],
"score": [
3,
5,
3
],
"text": [
"The basic problem is that the initial speed for shooting something out of orbit is astronomically high. Even ignoring drag the escape velocity of Earth is 11.2km/s. According to [stackexchange](_URL_0_) the escape velocity including drag is roughly 13.5km/s for a 1kg bullet-shaped object (minimizing drag) and for a 1kg sphere, you would need a speed close to the speed of light.\n\nThese extreme speeds leads to two problems. Firstly, the drag forces would pretty much vaporize whatever you shoot due to the extreme heat. Secondly, even with a coil/rail of several kilometers, the g-forces would be too high for almost anything to withstand. For example, a 7km coil let's you accelerate to 14km/s in one second, with 14,000g.",
"Because even without drag, nothing could survive the acceleration required. Let's assume no drag, and you're on the surface of the Earth. In fact, we'll put you right on the equator, to get maximum benefit from the Earth's rotation. Now, the ISS is in LEO (low earth orbit) so that takes the least energy to get into place. It is traveling at 17000 mph around the Earth. So you at least need to accelerate to that speed in order to get into orbit (minus the 1000 mph you get from being on the equator). So, assuming you are accelerated over an entire mile, you must be accelerated at [16,244 m/s^2 ](_URL_0_) over the course of that mile, otherwise known as 1657 g's. \n\nThe human body can only withstand an instantaneous acceleration of 40 g's, and sustained it is somewhere less than 10. So, let's say it is 10 g's is how fast you want to accelerate this capsule. Since the above calculation was inversely proportional to distance accelerated, that means the tube we accelerate the capsule in would have to be 165 miles long. And of course, this is assuming no drag, and that it takes no energy to get to the height required. So in reality, it would have to be much longer than this. ",
"Would it be feasible on the Moon? There wouldn't be any atmosphere problems and the Moon's gravity is a lot weaker than Earth's (in other words, can Heinlein beat Clarke?)"
]
} | []
| []
| [
[
"http://physics.stackexchange.com/questions/2317/including-air-resistance-what-is-the-escape-velocity-from-earth"
],
[
"http://www.wolframalpha.com/input/?i=%28velocity+ISS+-+1000+mph%29^2%2F%282*1+mi%29+"
],
[]
]
|
|
13scsp | Hypothetically, if someone with a cold ate ice cream out of a tub of ice cream and then the rest of that ice cream tub was put back into the freezer, would all the cold germs "freeze to death" making it safe for someone else to eat that ice cream? | askscience | http://www.reddit.com/r/askscience/comments/13scsp/hypothetically_if_someone_with_a_cold_ate_ice/ | {
"a_id": [
"c76qlkd"
],
"score": [
7
],
"text": [
"Bacteria are not going to \"freeze to death\", they are frozen at much lower temperatures on a normal basis (a normal lab freezer is set to -20C). Ice formation in the bacterial cells can lead to them \"freezing to death\", but this is unlikely in your freezer. I suppose if you waited long enough, the tunneling effect would kill them but this is not going to happen on a realistic time scale.\n\nThat said, if you don't want the ice cream send it to me, the immune system is pretty incredible and I'll take my chances."
]
} | []
| []
| [
[]
]
|
||
7mx6gy | how does an iud work? | explainlikeimfive | https://www.reddit.com/r/explainlikeimfive/comments/7mx6gy/eli5_how_does_an_iud_work/ | {
"a_id": [
"drxe55u",
"drxl3f9",
"dry8ys2"
],
"score": [
7,
8,
3
],
"text": [
"There's two basic versions of a an iud; one uses hormones while the other does not. \n\nThe non-hormonal one simply makes the uterus a non hospitable environment for the zygote, even if the egg gets fertilized. The hormonal one also acts as a physical deterrent, with the added bonus of super slow release hormones making the body think it's already pregnant. Or was just pregnant. Either way, the balance is off compared to a person's \"fertile period.\"\n\nPregnancy takes a lot of pretty precise events to occur. If any of those are wrong, even if the sperm meets the egg the body will reject it. ",
"Yer not alone in askin', and kind strangers have explained:\n\n1. [ELI5: How does a IUD work? ](_URL_3_) ^(_18 comments_)\n1. [ELI5: How do hormonal IUDs release a reliable, steady stream of hormones for years? And do they taper off or just stop releasing hormones? ](_URL_0_) ^(_3 comments_)\n1. [How does a copper IUD work? (ParaGard or equivalent) ](_URL_2_) ^(_3 comments_)\n1. [ELI5: how do non-hormonal IUD's work? ](_URL_1_) ^(_2 comments_)\n1. [ELI5: How can IUDs prevent periods? Where does the uterine lining go? ](_URL_5_) ^(_3 comments_)\n1. [ELI5: What are IUDs? ](_URL_4_) ^(_4 comments_)\n",
"Well, lots of answers from previous threads to answer this, but I'll take a hit at it since I'm bored at work. \n\nThe IUD, intrauterine device, is a small, T or anchor like piece of plastic/metal/copper that is inserted into the uterus through the cervix. When not enlarging for the purpose of fetus holding, the uterus is actually pretty small, about the size of a fist. When you put an object in it, it makes the uterus a pretty shitty place to be.\n\nPregnancy itself, despite how much we talk about surprise pregnancies, and how much we as a society take it for granted, is a difficult process, and requires a lot of proper steps to take place in order to be successful. Throwing a big anchor, literally, in the way, makes the uterus a hostile environment, and far less likely to indicate to the body that pregnancy is the right move at this point in time. \n\nThe IUD, prior to insertion, has a long pair of strings attached to the ends of it that are cut down after insertion. Once the IUD is ready to be removed, the gyno reaches in with foreceps, and tugs on the strings. This causes the prongs on the IUD to collapse and then it can be pulled straight out! \n\nIf hormones are involved in the IUD, as others have mentioned, it essentially works in the same vein as any other same-type hormonal birth control, most commonly by making the uterus extra-mucuousy so it doesn't allow for the implantation of eggs on the lining.\n\nSource: Was a army medic and regularly assisted and performed Well Woman's Exams and Birth Control procedures for Mirena, ParaGard, and Nexplanon"
]
} | []
| []
| [
[],
[
"https://www.reddit.com/r/explainlikeimfive/comments/571lr6/eli5_how_do_hormonal_iuds_release_a_reliable/",
"https://www.reddit.com/r/explainlikeimfive/comments/6r03p7/eli5_how_do_nonhormonal_iuds_work/",
"https://www.reddit.com/r/explainlikeimfive/comments/1myznk/how_does_a_copper_iud_work_paragard_or_equivalent/",
"https://www.reddit.com/r/explainlikeimfive/comments/6iq9xo/eli5_how_does_a_iud_work/",
"https://www.reddit.com/r/explainlikeimfive/comments/sdqik/eli5_what_are_iuds/",
"https://www.reddit.com/r/explainlikeimfive/comments/ta9t4/eli5_how_can_iuds_prevent_periods_where_does_the/"
],
[]
]
|
||
3hc45c | When I take medicine that supresses symptoms, do I prolong the sickness? | For example: If I have the flu and I take Dayquil or the related "medicine" does this pro-long the length of time I have the illness because I'm masking the symptoms or its it masking the symptoms which allow my body to heal during that time speeding it up. | askscience | http://www.reddit.com/r/askscience/comments/3hc45c/when_i_take_medicine_that_supresses_symptoms_do_i/ | {
"a_id": [
"cu63o39",
"cu65ilo",
"cu6bce1",
"cu6ftgg",
"cu6qwy0",
"cu6rpjh",
"cu6u4pb",
"cu6utks",
"cu74poi"
],
"score": [
31,
225,
12,
3,
3,
2,
5,
3,
2
],
"text": [
"[Here](_URL_0_) is a study that suggest that the flu could be prolonged by use of antipyretics. More study trials are needed to form a definite conclusion on the matter. It likely depends on the sickness in question, as well as which medicine is being used. ",
"There are basically three classes of symptoms\n\n* caused by the disease \"intentionally\" to help it survive\n* caused by your body to manipulate your behavior \n* caused by your body to promote healing\n\nAnd each symptom is often a mix of the three. There isn't enough research in lots of cases to say surely if a specific symptom is good or bad, and intensity of the symptom is important to consider as well.\n\nIn short: it's complicated, and for your specific question, not well understood.",
"Not necessarily, in some cases when the symptom is damaging, like running a dangerously high fever in an attempt to kill off the sickness, you can do more damage to yourself. So taking some tylenol and knocking out your fever is actually more beneficial to your recovery process. ",
"One aspect of the question can be answered. The symptom of running a fever is the most basic strategy your body employs to destroy viral or bacterial infection. Having fever to high or too long should absolutely be controlled with medicine or other means as you can damage you brain and other tissues, but if you knock down the moderate fever immediately after developing you are actually helping the infection. ",
"I actually have always wondered about a variation of this question, maybe the answer's obvious, but any insight would be appreciated: one of the things that I wonder is whether treating the symptoms can prolong the illness not because of any intrinsic reason, but because people then get up and go about their day as if nothing was wrong, when in fact they should be getting rest. Any research on that?\n",
"For some contexts such as bacterial enteritis (e.g. Drinking contaminated water and having a diarrhea afterwards), the use of antidiarrheic drugs (like loperamide or imodium) might prevent the \"ejection\" of the pathogen load that's inside the gut. Same principle for upper respiratory tract infections and the use of antitussive drugs (cough suppressants) like dextrometorphan. So in some cases, yes it could prolong the \"disease\" (and the accompanying symptoms) ",
"There's a phenomenon known as the [rebound effect](_URL_0_) which causes symptoms to return after a medication is discontinued, often with greater severity than pre-treatment. There are many examples of this.",
"I've heard symptoms likened to smoke alarms.\n\nSometimes smoke alarms go off when there is no fire (e.g. you're just cooking something smoky), but it's preferable for it go off fairly easily, rather than risk it not going off when there is an actual fire and burning to death.\n\nThe logic is that your body tends to over-express symptoms when sick, on the off chance that maybe this time you actually do need it and you would die otherwise. So in many cases, it can be fine (even beneficial) to suppress these symptoms.",
"Masking back pain with pain meds and muscle relaxers can be dangerous. Not only are there other safe options, but these pills are highly addictive and toxic to your body when taken for a while. Not to mention you trash your back even more when on them and they turn you into a zombie. Masking any pain with pain killers might be a quick temporary fix of the pain symptom, but its kinda like cutting the wire to the \"check oil\" light in your car to fix the problem instead of just getting your oil changed."
]
} | []
| []
| [
[
"http://www.ncbi.nlm.nih.gov/pubmed/11130213?access_num=11130213&link_type=MED&dopt=Abstract"
],
[],
[],
[],
[],
[],
[
"https://en.wikipedia.org/wiki/Rebound_effect"
],
[],
[]
]
|
|
1om37o | if what we see and hear has happened in the past does that mean that time outside our sphere of awareness is moving faster? | I read [this article](_URL_0_) about how what we see right now actually happened about 80 milliseconds ago because that is about how long it takes the brain to process the information. So does that mean that time outside of our sphere of awareness is actually moving faster or that we are moving slower? Or do I just completely misunderstand the science behind what the article says? | explainlikeimfive | http://www.reddit.com/r/explainlikeimfive/comments/1om37o/eli5_if_what_we_see_and_hear_has_happened_in_the/ | {
"a_id": [
"cct9wsg",
"cct9yo7"
],
"score": [
2,
2
],
"text": [
"It's all \"going at the same speed\", so to speak. The brain just takes a fraction of a second to 'boot up', and so it is starting out on the back foot.\n\nA bit like... if you have two identical marathon runners in a race, but one of them is given an 80ms handicap. You wouldn't say that time is now \"running slower\" for that person. They are running at exactly the same speed as the person in front of them. They're just suffering the effects of a momentary delay to start with.",
"It means that you're on an 80 millisecond delay. The rate at which time goes by is the same, you just don't notice it until 80 milliseconds after it happens.\n\nFor an analogy, they often use something called a 7-second delay on live TV and radio so that they can bleep swears and stuff. The host says something, and then it gets broadcasted seven seconds later.\n\nThe host is still talking in real time, though. A second to him is a second to you. You just don't hear it until seven seconds after he talks."
]
} | []
| [
"http://www.salk.edu/news/pressrelease_details.php?press_id=31"
]
| [
[],
[]
]
|
|
35lviz | Is there any evidence to suggest that the Coliseum was flooded for sport? | AskHistorians | http://www.reddit.com/r/AskHistorians/comments/35lviz/is_there_any_evidence_to_suggest_that_the/ | {
"a_id": [
"cr5nwgm",
"cr5ooec"
],
"score": [
8,
5
],
"text": [
"Seutonius passes over it quickly in his *Life of Nero*:\n > But he compelled four hundred senators and six hundred Roman knights, some of whom were well to do and of unblemished reputation, to fight in the arena. Even those who fought with the wild beasts and performed the various services in the arena were of the same orders. **He also exhibited a naval battle in salt water with sea monsters swimming about in it;** besides pyrrhic dances by some Greek youths, handing each of them certificates of Roman citizenship at the close of his performance. \n\nThere are a few accounts of it happening in addition to Seutonius', but the logistics and specifics are debated and not precisely known.",
"fyi there are a few comments on flooding arenas for naval battle reenactments in these threads\n\n* [Were naval battles staged in the Coliseum?](_URL_3_)\n\n* [How were the Ancient Romans able to flood and drain the Coliseum multiple times, without major structural damage?](_URL_2_)\n\njust mentions\n\n* [How much would the Roman Colosseum cost if it were to be built today?](_URL_0_)\n\n* [Was there a age limit to watch the games in the Colosseum in rome?](_URL_1_)"
]
} | []
| []
| [
[],
[
"http://www.reddit.com/r/AskHistorians/comments/1jchu2/how_much_would_the_roman_colosseum_cost_if_it/",
"http://www.reddit.com/r/AskHistorians/comments/1n489s/was_there_a_age_limit_to_watch_the_games_in_the/",
"http://www.reddit.com/r/AskHistorians/comments/2o4zew/how_were_the_ancient_romans_able_to_flood_and/",
"http://www.reddit.com/r/AskHistorians/comments/1gx6qg/were_naval_battles_staged_in_the_coliseum/"
]
]
|
||
27d2rq | Were there ever any battles that involved artillery units targeting each other? | AskHistorians | http://www.reddit.com/r/AskHistorians/comments/27d2rq/were_there_ever_any_battles_that_involved/ | {
"a_id": [
"chznzm7"
],
"score": [
9
],
"text": [
"It's a frequent occurrence known as counter-battery fire. It's one of the more basic artillery strategies there is, employed as far back as there's been artillery. I suggest picking up John Norris' *Artillery: A History*. It's a good introductory primer to the history of big guns."
]
} | []
| []
| [
[]
]
|
||
1gcx4e | why does adobe publish so much software for free? | explainlikeimfive | http://www.reddit.com/r/explainlikeimfive/comments/1gcx4e/eli5why_does_adobe_publish_so_much_software_for/ | {
"a_id": [
"caizz8b",
"caizzim",
"caj0agv",
"caj0uwi"
],
"score": [
2,
7,
7,
6
],
"text": [
"They dont. Most of Adobe's software is at least a couple hundred bucks. What software (other than reader) are you referring to?",
"By publishing so much free software, they can get their name known to pretty much every computer user in the world.\n\nThat way, when someone needs some software which is not free, Adobe's offering is a natural choice, because you've already heard of them.",
"The viewer software is free because you want anyone to be able to view the content. It is the content creation software that is so expensive. Yeah, there are millions of people using Adobe Acrobat Reader, Shockwave viewer and Flash Viewer, but to create Shockwave and Flash content you generally have to use the Adobe tools. PDF files though can be created with other free software, but the full version of Adobe Acrobat is for pay.",
"Tangentially related, but Adobe takes a somewhat lax stance on piracy of their software and has even released older versions of their software for free as specific marketing strategy: They don't care if individual college students pirate their copies of Adobe Photoshop, because the big bucks come from charging whole companies hundreds of dollars a pop for legit Photoshop licenses. \n\nLetting the small fish use their software \"for free\" just reinforces their standing as an industry standard."
]
} | []
| []
| [
[],
[],
[],
[]
]
|
||
2a23lq | if the europe-central map projection used in most places is inaccurate, why isn't a more accurate created and used instead? | Also, what is the most accurate map (according to the scientific community)? | explainlikeimfive | http://www.reddit.com/r/explainlikeimfive/comments/2a23lq/eli5_if_the_europecentral_map_projection_used_in/ | {
"a_id": [
"ciqpwr3",
"ciqq24s"
],
"score": [
7,
2
],
"text": [
"Are you talking about the distortion of certain maps, physically? Or are you talking about the fact that it's centered on Europe, and people think it should be centered elsewhere?\n\nIn general, though, there is no \"accurate\" flat map of the earth. The earth is round, and a map is flat. That will cause every map to be inaccurate in some way. We just get to choose WHICH inaccuracy we want, when we use different maps.",
"All 2D (flat) maps are inaccurate because you can't accurately represent a sphere in 2D. Therefore a 2D map will always be a compromise where you choose a type that is accurate enough for your specific use.\n\n_URL_0_"
]
} | []
| []
| [
[],
[
"http://en.wikipedia.org/wiki/Map_projection"
]
]
|
|
4bv9m4 | what is the tech bubble burst that everyone keeps talking about will happen soon? | [deleted] | explainlikeimfive | https://www.reddit.com/r/explainlikeimfive/comments/4bv9m4/eli5_what_is_the_tech_bubble_burst_that_everyone/ | {
"a_id": [
"d1cs9wr"
],
"score": [
2
],
"text": [
"Tech is part of our daily lives now and it's unlikely that will change any time soon so the demand for tech is comfortably high. But what is happening is the nature of that tech is always changing.\n\nConsumers are opting for tablets and smart phones over computers and laptops so the industry has to evolve to meet the new demand, and handle the decreased demand for products which used to be the bread winners. \n\nIronically in spite of all the money being spent on mobile devices, there was already a pretty big tech bubble that burst a few years back in telecomm with some pretty big names like Nortel and Alcatel. Even Nokia and Avaya. \n\nRight now there is a lot of money being thrown at the internet of things and having many tiny, inexpensive, computers operating together rather than big machines being central to the home. If consumers decided they don't really need smart toasters and refrigerators, then that could be a bust and there will be some shaking up of the job market some more. \n\n"
]
} | []
| []
| [
[]
]
|
|
1o3u0p | What was the original motivation, intent, and context of the 14th amendment to the US Constitution, specifically in regards to the public debt? | Section 4 of the 14th amendment says, "The validity of the public debt of the United States .... shall not be questioned." Each time the US Federal "debt ceiling" comes along, the idea of the President invoking the 14th amendment to circumvent it comes up. Leaving the current situation aside, where did this part of the amendment come from, and why?
For reference, the full text in question:
*Section 4. The validity of the public debt of the United States, authorized by law, including debts incurred for payment of pensions and bounties for services in suppressing insurrection or rebellion, shall not be questioned. But neither the United States nor any State shall assume or pay any debt or obligation incurred in aid of insurrection or rebellion against the United States, or any claim for the loss or emancipation of any slave; but all such debts, obligations and claims shall be held illegal and void.*
| AskHistorians | http://www.reddit.com/r/AskHistorians/comments/1o3u0p/what_was_the_original_motivation_intent_and/ | {
"a_id": [
"ccookjz"
],
"score": [
6
],
"text": [
"The primary motivation for the section was unquestionably to confirm that the U.S. would neither assume nor even recognize the validity of Confederate debt. \n\nThis part of the amendment was not heavily contested, so there is a relative lack of contemporary statements explaining its meaning, but here is Senator Howard giving an adequate explanation of Amend. XIV, Sec. 4:\n\n > I take it for granted that no member of this body would oppose the adoption of this section of the amendment. I do not believe the people of the United States will object to declaring that the whole of the rebel debt shall be eternally repudiated and extinguished — a debt contracted in the prosecution of the most wicked war with which the earth was ever cursed, against a Government that was never felt by them except in the benefits it conferred. Such a debt can never be assumed or paid by the loyal people of the United States, and if suffered to remain in *quasi* existence it can only be left in that condition as a subject of political squabbling and party wrangling.\n\n > The assumption of the rebel debt would be the last and final signal for the destruction of the nation known as the United States of America. Whatever party may succeed in so wicked a scheme, by whatever name it may be called and under whatever false guises or pretenses it may operate, if it succeed in assuming this indebtedness, puts an end first to the credit of the Government, and then, as an unavoidable consequence, to the Government itself. I do not propose to spend time upon this branch of the subject. I simply refer to it as a necessity of such magnitude as in my judgment to demand our action and the action of the States of the Union without delay. It is necessary to act, to extinguish this debt, to put it beyond the pale of party controversy, to put it out of sight, and to bury it so deep that it can never again be raised to life in such manner as to become a theme of party discussion. The amount of that debt is probably not less than five billion dollars. We do not know its exact amount, and I am not sure that it is possible ever to ascertain it; but if there should ever be a fair prospect of its assumption by the United States or by the States it is perfectly certain that the evidences of it would multiply thicker than the leaves in Vallombrosa. Those evidences are a great curiosity in the history of commercial affairs.\n\n[Cong. Globe, Sen., 39th Cong. 1st Sess. 2768 (1866)](_URL_1_)\n\nAnd here is what the Supreme Court had to say about Amend. XIV, Sec. 4 in the Gold Clause Cases:\n\n > While this provision was undoubtedly inspired by the desire to put beyond question the obligations of the Government issued during the Civil War, its language indicates a broader connotation. We regard it as confirmatory of a fundamental principle, which applies as well to the government bonds in question, and to others duly authorized by the Congress, as to those issued before the Amendment was adopted.\n\n[*Perry v. United States*, 294 U.S. 330, 354 (1935)](_URL_0_).\n\nIndeed, the divided Court in that case appears to be unanimous in holding that the Fourteenth Amendment's injunction on questioning \"[t]he validity of the public debt\" is merely declaratory or confirmatory of a principle already part of the U.S. Constitution prior to the ratification of the Fourteenth Amendment. And the Justices have the right of it.\n\nSection 4 is a great example of a literary technique the Constitution often employs, which I like to jokingly refer to as \"the passive-aggressive tense.\" The device operates when there are two interacting legal principles, one of which is uncontroversial, and the other of which is either a contested principle and/or a proposed innovation. Instead of stating the *innovation*, the Constitution will reiterate the *uncontested* principle, applied in such a way that it necessarily implies the innovation, while unstated, is now a part of the law.\n\nFor instance, the Eleventh Amendment, says that the sections of the Constitution setting out the subject-matter jurisdiction of the judiciary \"shall not be construed to\" abrogate a State's sovereign immunity. The thing is, everyone already knew that giving jurisdiction over a party doesn't necessarily abrogate immunity; the real debate was whether the States had immunity in the first place. By stating the point about jurisdiction not abrogating immunity, the amendment implies that immunity exists, but it doesn't actually state the principle which grants it.\n\nSimilarly, the Fifteenth Amendment states that the \"right of citizens of the United States to vote shall not be denied or abridged ... on account of race....\" Well, the thing is, the Constitution already prohibited discrimination based on race when it came to rights of the citizen. The contested question was whether voting was even a right of the citizen in the first place, such that a racially-restrictive voting law denied someone the right based on race. By stating the point about discrimination, the amendment implies that voting *is* a right, and that denying it based on race *is* racial discrimination against that right. But, again, it doesn't actually state the operative principle, leaving us to sort out if there any other implications of the principle implicitly being affirmed.\n\nSection 4 of the Fourteenth Amendment is doing a similar thing. The uncontroversial principle at the time was that the grant of power to Congress \"[t]o borrow Money on the credit of the United States,\" Art. 1, Sec. 8, cl. 2, implied that money properly borrowed became an obligation that continued until repaid, and that the obligation could not be cancelled by a subsequent law. (Note that actually *paying* the debt is another matter, which I will explain below.) The controversial matter was how this should apply to the Confederacy:\n\nIf the Confederacy were thought of as an independent nation which had been defeated and annexed by the U.S., then the U.S. was the successor state to the Confederacy under international law, and therefore ought to assume its debts. Likewise if the State governments of the Confederacy were legitimate, then the Reconstruction governments were their successors and still owed their debts. But if the Confederacy was thought of as invalid from the start, then there was never a legitimate government there, and the U.S. was not in any sense a successor to the Confederacy or its debts.\n\nThis latter interpretation is the one that the Reconstruction-era government consistently takes. And it is this interpretation that Section 4 aims to enforce in the context of sovereign debt. The amendment does not say that \"the public debt ... shall not be questioned\" because that principle needs stating. It says that the debt shall not be questioned because the correct *application* of that principle needs stating. The amendment gives an authoritative gloss to the principle by asserting that it does not mean that the U.S. has (or even could have) an obligation to pay the Confederate debts.\n\nNow, I should add as an afterthought that affirming the validity of debt is different from actually paying it. An analogy to the States illustrates the difference:\n\nThe Constitution also provides that the States cannot invalidate contracts by power of law. (\"No State shall ... pass... [a] Law impairing the Obligation of Contracts....\" Art, 1, Sec. 10). And this extends to a prohibition on invalidating its debts. Nevertheless, States have permanently defaulted on debts on numerous occasions. The States do this not by questioning the validity of the debt, but by acknowledging the wrong and providing no remedy for it. The States simply deny anyone access to their courts to collect on the debt, and rely on their immunity from suit in federal courts. That leaves only the very rare case where it is possible to sue a State in *another* State's court, and to then seize assets of the first state found within the second state.\n\nCurrently the U.S. government grants its permission to be sued on its debts (waiving its sovereign immunity), but conceivably it too could withdraw this permission to be sued on its debts, never questioning the *validity* of its debts while still failing to actually *pay* them. The proposition has never been tested in court, and is the subject of current (if infrequent) debate."
]
} | []
| []
| [
[
"http://scholar.google.com/scholar_case?case=3388791031923623137",
"http://memory.loc.gov/ll/llcg/072/0800/08502768.tif"
]
]
|
|
w1513 | why do we pronounce the "w" in words like "swore," but not in "sword?" | explainlikeimfive | http://www.reddit.com/r/explainlikeimfive/comments/w1513/eli5_why_do_we_pronounce_the_w_in_words_like/ | {
"a_id": [
"c59c6vq",
"c59c7bb",
"c59cdgb",
"c59d15e",
"c59f7r4",
"c59j7rv"
],
"score": [
2,
27,
12,
2,
2,
3
],
"text": [
"Not sure about those two words specifically, but common English words come from many different langauges, often using the same letters. If the words of origin are vastly different (like not just different words but from different langauges) it is not surprising the pronunciations would be different.",
"Because English is a stupid language.\n\n_URL_0_",
"I'm no expert, but linguistics is always something that has been a major interest for me. First of all, English is a weird language. It has tons of strange rules that complicate the language significantly. I think that in this case, it depends on the etymology of the words, or what language the words came from. Some languages pronounce letters in different ways, and when the words pass into English, they tend to somewhat retain the same pronunciations as they had in their previous language. For example, the word \"swear\" has kept its pronunciation [as it transferred from Proto-Germanic to Old English to English](_URL_2_). \"Sword\" is a bit more difficult. Looking at the [etymology](_URL_0_) it appears that it retained it's hard \"sw\" sound even in Old English, but in recent times has lost that hard \"w\" sound. I cant explain that. Take the word \"wreck\" however. Looking at the [etymology](_URL_1_) one can see that it gained the silent \"w\" when it passed from the Swedish \"vrak\" (pronounced we-rek (although its really one syllable, but I'm emphasizing that the \"v\" is pronounced like a \"w\")) to Anglo Norman \"wrec\". During that time period, the sound that the \"w\" made probably became less pronounced as it is a difficult sound to produce being next to the \"r\". over time, the sound was completely phased out of the word and is now pronounced as we know it: \"rek\". \n\n\n**TLDR: It depends on which language the words come from.**\n\nHopefully this helped, If you have any further questions, Ill do the best I can to try to answer them.\n",
"I can't say what's true for any specific case, but for a lot of words the reason English spelling is so whacky is because the spellings were made concrete before the pronunciations were. The printing press was invented before the radio and as such widely available books for all parts of England caused certain spellings to become standard, even though people all over the country still pronounced the words very differently to each other. Eventually how you pronouce each word also became standard, but that had little to do with the way the spelling became standard.\n\n**TL;DR:** How you spell words and how you say words in English became standard independently.",
"We used to. \n\nThe K in Knight, for example. Kuh-nicht. Similar to German Knecht.",
"British English speakers do pronounce the \"w\". "
]
} | []
| []
| [
[],
[
"http://english.stackexchange.com/questions/32918/why-is-the-w-silent-in-sword"
],
[
"http://en.wiktionary.org/wiki/sword#Etymology",
"http://en.wiktionary.org/wiki/wreck#Etymology",
"http://www.etymonline.com/index.php?search=Swear+in"
],
[],
[],
[]
]
|
||
eg0mdx | how can you get pregnant when not taking the pill every day at the same time/missing a day or when leaving the hormonal ring out for more than 3 hours? doesn't hormonal bc have a longer-term effect on women's bodies? | explainlikeimfive | https://www.reddit.com/r/explainlikeimfive/comments/eg0mdx/eli5_how_can_you_get_pregnant_when_not_taking_the/ | {
"a_id": [
"fc3lvjt",
"fc3mdl3"
],
"score": [
3,
2
],
"text": [
"As I understand it (and I'm not an expert or anything close), hormonal birth control stops you from ovulating, and the part that it stops doesn't take very long. If it gets messed up, that little part will happen, then when whole process starts and you can be fertile.",
"No form of birth control is 100% effective, even when used correctly. Biologic processes have high variability and quality control isn't as precise as you might expect from mechanical or electronic systems.\n\nThe NuvaRing has the advantage of releasing hormones in the right place, as opposed to the systemic technique of a pill you swallow. With this advantage, lower doses can be used to reduce side effects.\n\nAll these hormones have clean-up mechanisms in the body that break them down, that's how the system works."
]
} | []
| []
| [
[],
[]
]
|
||
167oe5 | why is it we find walking outside in 90 degree heat unbearable, yet sitting in a 90 degree sauna is so relaxing? | much obliged. | explainlikeimfive | http://www.reddit.com/r/explainlikeimfive/comments/167oe5/eli5_why_is_it_we_find_walking_outside_in_90/ | {
"a_id": [
"c7thewr",
"c7tjdse"
],
"score": [
8,
4
],
"text": [
"Part of it probably has to do with the sun. The temperature isn't really the unbearable part, the burning from the sun is. In a sauna it is dark and moist. Even so I still hate them. ",
"I find saunas insanely stressful! \n\nIt's the ability to step out of them in to the cool that relaxes me. "
]
} | []
| []
| [
[],
[]
]
|
|
2kzsaj | How did the US become solely a British Colony when originally there were so many different colonial powers in the area? | AskHistorians | http://www.reddit.com/r/AskHistorians/comments/2kzsaj/how_did_the_us_become_solely_a_british_colony/ | {
"a_id": [
"clq5py6",
"clqjc8k"
],
"score": [
19,
4
],
"text": [
"Remember that the French controlled Louisiana (a territory that does not match the boundaries of the State of Louisiana) until well after the American Revolution. We bought it during the presidency of Thomas Jefferson. \n\nEach area has its own history of turning from non-English controlled areas to either English or American control. \n\nIt is also a little misleading to say that the US became a British Colony. The US was an amalgamation of pre-existing colonies that had already revolted from colonial control and gone through one failed attempt at a common government by the time the Constitution finally knitted them together. \n\nBy the time the US was created, each of those colonies had already become sovereign states (little s to denote the fact that they were essentially countries) which agreed to give up a portion of their sovereignty in order to become the United States.",
"Essentially, the other colonial powers on the Eastern Seaboard either gave up and went home (in the case of Sweden) or had their colonies taken by force in wars against England (French Canada and the Dutch in New Amsterdam). Eventually the British had occupied everything from Canada down to Georgia, which was created as a buffer with Spanish-owned Florida."
]
} | []
| []
| [
[],
[]
]
|
||
6i7jl3 | Why isn't Juneteenth a national holiday in the United States? | I know this may seem like a very obvious question, but I'm curious about what specific historical contexts and events have resulted in the seemingly limited celebration of Juneteenth to the African-American community in the United States. I am struck that the emancipation of slaves in the United States is not a more widespread cause for national reflection and celebration.
Thank you! | AskHistorians | https://www.reddit.com/r/AskHistorians/comments/6i7jl3/why_isnt_juneteenth_a_national_holiday_in_the/ | {
"a_id": [
"dj44dba"
],
"score": [
66
],
"text": [
"Juneteenth does not commemorate the actual emancipation of slaves; that happened in January 1863 for the states in rebellion, and with the ratification of the Thirteenth Amendment in Dec. 1865.\n\nJuneteenth celebrates when news of the Emancipation Proclamation finally reached Texas in June 1865. Initially a celebration only among Negroes in Texas, the commemoration first spread to a couple of adjacent states, and in the 20th century traveled with African-Americans from Texas as they migrated to industrial cities elsewhere. In particular, it seems to have taken hold among those who moved to the Bay Area during World War II to work in shipyards and defense industries. It seems likely that its enduring popularity owes something to the summer date, suitable for large outdoor gatherings and celebrations. Only in the last 25 years has the celebration become known nationwide.\n\n[The *Handbook of Texas* has a good summary.](_URL_0_)"
]
} | []
| []
| [
[
"https://www.tshaonline.org/handbook/online/articles/lkj01"
]
]
|
|
135g5i | the expulsions of the germans in european countries during and after ww2. | What was the ultimate goal of having these germans deported from their countries? | explainlikeimfive | http://www.reddit.com/r/explainlikeimfive/comments/135g5i/eli5_the_expulsions_of_the_germans_in_european/ | {
"a_id": [
"c70zf2d"
],
"score": [
2
],
"text": [
"By the time of the World Wars, German speaking populations existed in many places throughout Europe, including Central Europe and along the Baltic Sea. These maps both show Prussia and other German states [before WWI](_URL_4_) and [before WWII](_URL_0_). Note how the territory is larger than modern Germany, and includes much of modern day Poland.\n\nAs WWII drew to a close, many German speaking residents of Eastern Europe/the Baltic migrated west with the retreating front line, fearing dire ramifications from the Soviets. Eventually by the war's end a lot of German speakers (regardless of ethnicity) and German citizens (regardless of language) ended up in Allied-Occupied Germany or Austria.\n\nAccording to the Potsdam Agreement by Allied leaders on the state of post-war Europe/boundary changes/etc, this population migration continued up until 1950 or so. About 12 million Germans are estimated to have been moved. Up to 500,000 may have died during this migration.\n\nIf you look at a map of Europe, you can see notable changes, especially in Russian territory. [Koenigsberg](_URL_3_) for example, the former capital of East Prussia was heavily war-torn and repopulated with Russians. Today, it's called [Kaliningrad](_URL_2_) it's about 0.7% German by ethnicity.\n\nEDIT: For more reading I'd recommend this _URL_1_\n"
]
} | []
| []
| [
[
"http://en.wikipedia.org/wiki/File:Deutsches_Reich_1925_b.png",
"http://en.wikipedia.org/wiki/Flight_and_expulsion_of_Germans_(1944%E2%80%931950)",
"http://en.wikipedia.org/wiki/Kaliningrad",
"http://en.wikipedia.org/wiki/File:East_Prussia_1923-1939.png",
"http://en.wikipedia.org/wiki/File:Prussiamap.gif"
]
]
|
|
9uptiz | Why wasn't Augustus one of the "5 good Roman emperors"? | He had a very long stable reign, expanded the empire, "found Rome in brick and left it in marble" etc. And I know that he was a master propagandist and he killed off most of the people that opposed him(but this one was before he became emperor), but he still deserves a spot here in my opinion. | AskHistorians | https://www.reddit.com/r/AskHistorians/comments/9uptiz/why_wasnt_augustus_one_of_the_5_good_roman/ | {
"a_id": [
"e97dvp1"
],
"score": [
11
],
"text": [
"I think the best explanation in this case is the simplest one: the term “5 Good Emperors” refers specifically to the chronological succession of five emperors (Nerva, Trajan, Hadrian, Antonius Pius, and Marcus Aurelius) who were lauded for their expansion of the empire and the relative stability of their reigns. It’s not simply a list of “the five best Emperors of Rome — number 4 will shock you!” it’s a non-dynastic succession of notable emperors. Augustus is widely regarded as a successful and comparable emperor, but he wasn’t part of the succession that involved the aforementioned 5, he reigned much earlier and as part of the Julio-Claudian dynasty. "
]
} | []
| []
| [
[]
]
|
|
1hscck | How does the fact that heat moves from warmer objects to cooler objects relate to IR emitted from greenhouse gases? | So someone I know is arguing that even though IR associated with CO2 has been observed to increase at earth's surface this shouldn't have an effect on temperature because the earth's surface was the "source" of that heat. Is this a reasonable argument and if not what is wrong with it? | askscience | http://www.reddit.com/r/askscience/comments/1hscck/how_does_the_fact_that_heat_moves_from_warmer/ | {
"a_id": [
"caxiw74",
"caxixcc"
],
"score": [
6,
2
],
"text": [
"No, that is a entirely unreasonable argument. \n\nAs an illustration, which do you think would heat up to higher temperatures? An oven with the door open or an oven with the door closed?\n\nIf you consider the surface of the planet to be the IR source in the oven, and greenhouse gasses to be the oven door, you have a fairly reasonable (albeit exaggerated) analogy on your hands.\n\nNow the person you are talking to is onto *something* with the point that you can't increase the heat of an object by trapping smaller amounts of heat around it... but in the real world that really only applies to very specific and simple situations and is totally irrelevant here. The surface of the planet (rocks, soils, buildings, etc) are already in balance between heat gathered from the sun and heat lost to the atmosphere. As the atmosphere increases in temperature, the physical surface of the planet will have a harder time shedding heat, and it can be expected to increase in temperature too.",
"So, the surface of the earth radiates long-wavelength, infrared heat upwards, from the surface.\n\nEach layer of atmosphere with greenhouses gases absorbs some of the heat being radiated upwards **from lower layers**. It re-radiates in all directions, both upwards and downwards, **equally**\n\nThe lower layers are absorbing more heat, from the earth and also from the heat being radiated downwards from the upper layers. The lower the layer, the warmer it is.\n\nHope this answers your query."
]
} | []
| []
| [
[],
[]
]
|
|
3jeee7 | How did Plato's name become root word for non-sexual friendship? Platonic. | AskHistorians | https://www.reddit.com/r/AskHistorians/comments/3jeee7/how_did_platos_name_become_root_word_for/ | {
"a_id": [
"cuoo7ik"
],
"score": [
32
],
"text": [
"Mainly because of his dialogue the *Symposium*, which is effectively a prose drama showing several people making speeches about the nature of love/desire. All of the speakers are in agreement that physical desire is the \"lowest\" kind of love, and as the drama progresses there's a kind of escalation of what \"higher\" kinds of love look like. One speaker casts \"higher\" love as a relationship between minds, not bodies; another casts love as a cosmic force, representing all forms of attraction; another casts love as the desire for wholeness; Socrates (the hero) casts different kinds of love on an ascending scale that reaches up, at its most philosophical \"heights\", to the meditation on and contemplation of abstractions that are usually known as [Platonic forms](_URL_0_).\n\nThe term \"Platonic love\" was coined by Ben Jonson in his 1631 play *The New Inn*. Act III scene 2 is essentially a discourse on love, and explicitly summarizes a number of passages in Plato's *Symposium*. At lines 73-6 the character Lovel starts off by defining love as follows:\n\n > For, what else \nis *Love*, but the most noble, pure affection \nof what is truly beautiful, and faire? \nDesire of union with the thing beloved?\n\nThis is essentially a paraphrase of an opening section in Socrates' speech. A few lines after that, Beaufort paraphrases Aristophanes' speech in the *Symposium*, and Lovel makes the attribution explicit (\"It is a fable of *Plato's*, in his Banquet, / and utter'd, there, by *Aristophanes*\"). Lovel goes on to cite ideas from other speeches in the *Symposium*:\n\n > **Beaufort.** I relish not these philosophicall feasts; \ngive me a banquet o' sense, like that of Ovid... \n\n > **Lovel.** They are the earthly, lower forme of lovers, \nare only taken with what strikes the senses!\n\nAnd so on, until we get to Lovel coining the phrase at line 278, with the twist of simultaneously demanding a kiss as a fee for his time:\n\n > **Lovel.** Most *Socratick* Lady! \nOr, if you will *Ironick*! gi'you joy \no'you *Platonick* love here, Mr *Lovel*. \nBut pay him his first kisse, yet, i'the Court, \nwhich is a debt, and due: for the houre's run.\n\nPlato's/Jonson's philosophization of love, and Jonson's emphasis on the non-sexual nature of \"Platonic\" love (a view that Plato did not share!), provided a useful way for people to rationalize homosocial relationships and categorize them as non-sexual. A lot of western homosocial culture depends on the existence of that non-sexual category. Platonic love as sexual did get somewhat rehabilitated, largely thanks to the invention of homosexuality and elevation of \"Greek love\" in the 19th century, but Jonson's phrase has stayed with us."
]
} | []
| []
| [
[
"https://en.wikipedia.org/wiki/Theory_of_Forms"
]
]
|
||
2bprp3 | how do the "double irish" and "double irish w/ dutch sandwich" tax avoidance techniques work? | [Disclaimer]: This isn't exactly a repost. The infographics on the earlier posts of this nature aren't loading properly, and I would appreciate any helpful diagrams with your explanations. Thanks! | explainlikeimfive | http://www.reddit.com/r/explainlikeimfive/comments/2bprp3/eli5_how_do_the_double_irish_and_double_irish_w/ | {
"a_id": [
"cj7rupq"
],
"score": [
5
],
"text": [
"The EU operates a free trade regime such that a state can't attempt to tax an entity based in another country selling to its citizens. In the US context its very similar to the way US states behave, if I live in NY and buy something from a company in MA then NY doesn't tax the company in MA for profits they make but instead they are taxed in MA.\n\nIn the EU Ireland has the most favorable tax rate so many corporations HQ there in order to pay the lower tax rate. If you buy a Dell computer in France you are not doing business with Dell France but with Dell Ireland. In effect businesses funnel all sales through an Irish office to pay a lower rate of corporation tax.\n\nIreland has a tax treaty with the Netherlands which massively reduces Irish tax burden for profit transfers there (technically its an entire exemption but some revenue will be subject to taxation due to licensing arrangements). Once in the Netherlands there is a small tax imposed (again licensing arrangements) but then it is transferred back to Ireland to a separate entity (2nd Irish subsidiary, hence the \"Double\").\n\nA quirk in Irish tax law means that a company that is actually controlled by foreign managers can transfer all profits to that country tax free so the 2nd Irish subsidiary is actually controlled by another subsidiary in another country (outside the EU, Bermuda is popular).\n\n[This](_URL_0_) illustrates the process fairly well.\n\nSome countries (most of the Middle-East, Africa, Australia, Asia) has lax revenue shifting controls so companies also shift earnings from these subsidiaries through Ireland to similarly reduce their tax exposure.\n\nWhile this is typically mentioned in the context of US corporations it actually has no impact on US tax revenue at all, its impossible to shift US based revenue out of the US and due to IP transfer fees (fees a subsidiary has to pay to its US parent to account for brand, technology etc developed in the US) some of the non-US revenue does make it back in to the US and instead primarily impacts EU income. In the Google example in 2012 they had an EU effective corporate tax rate of 4.4% while a US effective corporate tax rate of 29.1%."
]
} | []
| []
| [
[
"http://static.theglobalmail.org/cache/42/36/4236909b7927edfad707fb72e2d5d013.jpg"
]
]
|
|
1ix976 | how do they get water up large skyscrapers | I've always wondered how they get water up massive skyscrapers like in new york or in india, please help!
Edit: Thanks for all the answers and videos! I appreciate having my question answered by a great community! and made my Front page! Thanks again!!! | explainlikeimfive | http://www.reddit.com/r/explainlikeimfive/comments/1ix976/eli5_how_do_they_get_water_up_large_skyscrapers/ | {
"a_id": [
"cb8xb98",
"cb8z0lf",
"cb8zt87",
"cb95tsq",
"cb971hf",
"cb9a4xg",
"cb9bqzd",
"cb9c21l",
"cb9d0p5"
],
"score": [
547,
17,
149,
48,
15,
2,
3,
3,
5
],
"text": [
"with pumps! the pressure that is supplied by the water main available in most cities is usually only adequate to get water up around 6-8 stories. any building higher than that needs to supplement the water pressure in its system with pumps. ",
"_URL_0_\n\nThank you for asking this; I really never thought about it before. ",
"Water controls designer here. \n\nAnother way they do it is they will have a hydropneumatic tank every couple floors. Basically what you do is have a big tank and you fill half of it with water and the other half you pressurize with air. Then the air pressure pushes the water out to your faucets. When the water level drops in a tank, a pump is called to fill it back up and an air compressor turns on. \n ",
"Plumber here, pumps have a thing called an impeller that can increase water pressure by about 100kpa per impeller, you put as many impellers on as you need to increase the pressure.\n\n* On small building to about 10 stories you would install a electronically controlled variable speed pump, they have like a water wheel on the outlet the more demand there is for this the more the pump increase the RPM on the impellers.\n\n* To control pressure you install a pressure limiting valve on the floors to limit the pressure to a set amount.\n\n* For really large high risers the same principal applies except you install resoviors every 10-20 stories, these are trickle fed from pumps below or tanks on the roof and then the pump takes over to pressurise the floors below. All floors have pressure limiting valves as this is code requirements to prevent overpressurisation of the pipework and fittings per floor, you cant have basin taps popping and flooding.\n\n*You dont do gravity fed from a roof tank to a whole building as with gravity fed you have to install very large pipes to get the flow up per floor, this is why you install resovoirs per floor and pump accordingly.\n\n* Luckily for us plumbers we have hydraulic engineers who work out the diversity factor and the amount of water required per floor and therefore the flow rates and the pressure required to deliver the flow rates. Once you have the flow rates and pressures you turn to your tables to work out your pipe sizing,\n* The cost of the pipes determine how often you need to install a tank and resovoir to bring the cost down to reasonable to install.\n\n* EG installing a 150mm water main riser is much easier and cheaper than installing a 300mm water main riser.\n\n* Im not sure if I made it sound simple or complicated. Anyway after 20 years you know what to do, and by the design on the building there is normally only a couple of options that will work,,,,\n",
"Hey OP! From the replies it looks like you got a pretty great answer! Therefore, with your new knowledge, I'll share a cool efficiency pump solution and a really neat fact about trees!\n\nNow you've learned that they either pump the water alllll the way up to the top of the building with multiple pumps or they store the water in tanks on they way to the top floor. But how can we make this efficient? Well if money isnt too much of an issue you can replace these impeller pumps (pumps that use spinning blades like a fan or boat propeller to push water) with hydraulic pistons! Assuming you know that big machinery can lift giant amounts of dirt or other things (duh lol), well they use hydraulics to do this. If we apply this technique to a giant piston at the base of the building and we carefully watch and maintain pressure we can efficiently push large amounts of water right up to the top with only ONE \"pump\".\n\nOkay, time for the cool fact! Maybe after reading through your responses you've pondered \"why cant we suck water from the top like a straw?\". Well there's a good reason for this and its because it's impossible! To understand why this is impossible, you need to understand a bit about pressure. So first how does a straw work? Well when we \"suck\" on a straw we are lowering the pressure on top of the water inside the straw and the top of the water wants to get back to where it once was (our atmosphere's pressure) so it climbs up the straw while you suck that air out until it gets to your mouth.\n\nSecond, how do we measure pressure? Well one method we have been doing the same way for years has been the difference in the height of water inside a U tube called a Manometer. If one end of that U tube is connected to what we are measuring and the other end is connected to what we are measuring against (usually the open atmosphere) then the water levels in the U (in the up-down direction from the earth only) will change and we can calculate pressure from it using gravity and the density of the water in the Manometer.\n\nOkay so we know how a straw works, and we are less than fuzzy on calculating pressure, but we still dont know why sucking water up a skyscrapper is impossible, it sounds pretty possible so far... Well here's where things get tricky. You see if we have such a low pressure (TONs of air sucked out) the liquid at the highest point inside our straw or manometer begins to \"fall apart\". That is, it actually starts changing state into a gas. Since there's no pressure keeping the liquid together, it starts expanding to try and fill in the empty space (by empty we mean litterally nothing. No air or any matter in that space).\n\nSo here's where we hit a limit on how far up a straw or a skyscrapper you can suck water. From our little Manometer lesson, we know that a difference in water-height means theres a pressure difference. Well the same thing applies to a straw, or in our case a reeeaallly long straw. If we keep sucking on a super long straw (straw is directed up-down from earth), eventually the top of the water inside the straw will hit such a low pressure that it will begin to change into a gas! Technically \"steam\", but its not hot.\n\nSo at what height difference does this happen? Well it all depends on gravity, what the liquid's density is, and what our atmosphere's pressure ks. So with pure water and on Earth you can only suck up 10.3 metres until it starts turning into steam! Therefore sucking water up a skyscrapper is impossible!\n\nNow here's the crazy cool fact. Many trees are taller than 10.3m and we've all learned that their roots \"suck\" water out of the ground. But if that's true, how does a tree get the water to the top? Wouldnt the water turn to steam inside the tree? They do it not by sucking, but by actually pumping the water up. The roots actually just absorb the water then the tree has thousands of very very very tiny tubes that expand and contract to push the water up the tree. So a tree is basically a collection of thousands of pumps!\n\nHope you enjoyed my little essay post and maybe learned something while I car pool to work! Im sorry I couldnt link sources or videos or images to further explain, as im using my phone. Have a good day :)",
"Here is a list of video links collected from comments that redditors have made in response to this submission:\n\n|Source Comment|Score|Video Link|\n|:-------|:-------|:-------|\n|[zahnerphoto](_URL_0_)|11|[A crab getting sucked in a pipe subsea](_URL_4_)|\n|[somnolent49](_URL_13_)|7|[Delta P](_URL_15_)|\n|[mickeydjw](_URL_14_)|1|[Anna Faris Cindy- Scary movie](_URL_6_)|\n|[mrzar97](_URL_1_)|1|[Strip The City Dubai - Desmontando a Cidade Dubai Legendado](_URL_11_)|\n|[gearaholic](_URL_2_)|1|[Dirty Jobs - Safety Third - Water Tank Replacement](_URL_16_)|\n|[z-fly](_URL_7_)|1|[Skyscraper Water Supply Strip the City](_URL_9_)|\n|[FTpotato](_URL_8_)|1|[The Most Amazing Thing About Trees](_URL_3_)|\n\n* [VideoLinkBot FAQ](_URL_12_)\n* [Feedback](_URL_10_)\n* [Playlist of videos in this comment](_URL_5_)",
"They don't just pump it to the top floor, they pump to every so-many floors to holding tanks, and then pump it from that floor's tank to the next tank so many floors above. Usually, it is something along the lines of every 25-27 floors has a holding tank that supplies the floors below it with water and water pressure.\n\n_URL_0_",
"Buckets. Lots and lots of buckets.",
"Really big elephants."
]
} | []
| []
| [
[],
[
"http://www.humansinvent.com/#!/3366/extreme-plumbing-pumping-water-up-the-shard/"
],
[],
[],
[],
[
"http://reddit.com/comments/1ix976/_/cb9a4sq",
"http://reddit.com/comments/1ix976/_/cb99tyt",
"http://reddit.com/comments/1ix976/_/cb97twk",
"http://youtu.be/BickMFHAZR0",
"http://youtu.be/XoZ_aVj7P9U",
"http://radd.it/comments/1ix976/_/cb9a4xg?only=videos&start=1",
"http://youtu.be/wx2NdrhDcME",
"http://reddit.com/comments/1ix976/_/cb96ojm",
"http://reddit.com/comments/1ix976/_/cb95g61",
"http://youtu.be/PgxSD6H799Q",
"http://www.reddit.com/r/VideoLinkBot/submit",
"http://youtu.be/gZpQzcQezvY",
"http://www.reddit.com/r/VideoLinkBot/wiki/faq",
"http://reddit.com/comments/1ix976/_/cb9aage",
"http://reddit.com/comments/1ix976/_/cb9cl73",
"http://youtu.be/AEtbFm_CjE0",
"http://youtu.be/H0u56J3EFks"
],
[
"http://i.imgur.com/VPsucSR.jpg"
],
[],
[]
]
|
|
6epw9w | how did media control symbols become standard (play, pause, fast forward, etc.)? do other parts of the world use different symbols? | explainlikeimfive | https://www.reddit.com/r/explainlikeimfive/comments/6epw9w/eli5_how_did_media_control_symbols_become/ | {
"a_id": [
"dic4jvk",
"dici3tk",
"dicmd17"
],
"score": [
26,
2,
3
],
"text": [
"_URL_0_\n\nThere is actually an international standard for all forms of icons.\nI'm going off of personal knowledge here, so correct me if I'm wrong, but during the 80s when electronics started to become more widespread globally and production switched from the west to the east, many of the manufacturers understood they would need a standardized way of denoting controls on electronics. They held a convention to draft up a list of virtually any kind of control one would need as a symbol that could be understood. There like a huge book of hundreds of symbols.\n\nTL;DR the symbols we used are arbitrary and standardized internationally.\n\nEDIT:removed the extra \"d\".",
"The media control symbols for play, fast forward and pause originate from tape players. \nThe play button pointed to where the direction the tape would go, pressing it would move the tape. \nThere used to be a button that was a triangle pointing the opposite direction that would cause the device to play backwards. \nThe fast forward button is just the play button repeated, meaning the tape would move faster than play. \nSince there was a backwards play button, it was repeated for fast rewind. \n(Also note the term \"rewind\" refers again to tapes being wound up again.) \nThe pause button just looked like the tape head (thing that read the tape) and would mean the tape head would do nothing. \n\nThe square stop symbol actually comes from musical notation, and means [\"large rest\"](_URL_0_)",
"They come from audio tape players. Audio tape used to be on a supply reel on the left, with a take-up reel on the right. So, to play it, the tape moved from left to right, in the direction of the arrow. Fast forward and fast rewind are just two arrows showing \"double speed\" and which direction the tape moves in. \n\nPause probably comes from a similar background: In order to keep the tape moving smoothly and precisely, there is a device called a pinch roller. The tape is pinched between a rubber roller and a vertical metal spindle called a capstan. On bi-directional tape players, there is a capstan/pinch roller on either side of the play head. When the pause button is pressed, the tape remains pinched between the pinch rollers and the capstans. The pause button is reminiscent of the two capstans holding the tape in place. On many decks, you couldn't fast-forward or rewind from pause because the tape was held in place by the pinch rollers. You had to disengage them to allow the tape to move. That's why the Stop button doesn't have the capstan images.\n"
]
} | []
| []
| [
[
"https://www.iso.org/standard/30820.html"
],
[
"https://en.wikipedia.org/wiki/List_of_musical_symbols#Notes_and_rests"
],
[]
]
|
||
2wkmeo | what does 'all property is theft' mean? | explainlikeimfive | http://www.reddit.com/r/explainlikeimfive/comments/2wkmeo/eli5_what_does_all_property_is_theft_mean/ | {
"a_id": [
"corpr8n",
"corrn3c"
],
"score": [
8,
4
],
"text": [
"It is a saying that implies that anything you can own involves the exploitation and/or abuse of someone or something that is not being compensated, and that you can't separate the property itself from it's origins... That you can't, for examply, own a pair of shoes without separating them from the fact that it likely used child workers to make them, or the rubber in them came from rubber trees planted on land taken generations ago from the indigenous people originally there.",
"This statement is a basic tenet of anarchism, that everything belongs to everybody, or rather, belongs to no one, so that all are equal. No government or individual can own the land in which they reside, and therefore no government can exist. If that seems strange to you, you're probably not an anarchist."
]
} | []
| []
| [
[],
[]
]
|
||
6fwbo1 | why has the global temperature spiked so dramatically in the last 10-20 years even though that's the timespan we've become aware of global warming and started trying to combat it? | explainlikeimfive | https://www.reddit.com/r/explainlikeimfive/comments/6fwbo1/eli5_why_has_the_global_temperature_spiked_so/ | {
"a_id": [
"dilhmvo",
"diljaeu",
"dilju2n",
"dimtlcy"
],
"score": [
5,
5,
2,
2
],
"text": [
"Guess what happened before 20 years ago? Rampant pollution while being oblivious. \"Just exhaust it, planet Earth will pick up the slack and balance out\" was the mentality back then.",
"Because awareness doesn't imply doing something about it, the efforts to combat it are tiny, oil was cheap, and the alternative tech wasn't there.\n",
"Combat it? Hell, we can't agree it's real. We're putting more co2 into the air every year, didn't notice til the shit hit the fan. Way too late now. \nWe're xxxxscreweded! (Edit4eli5)",
"We've actually been aware of global warming for [a long time](_URL_0_).\n\nThere are several problems, though:\n\n* By the time we were aware of it a lot of damage was done.\n* Even today a lot of people still don't believe it's real, or don't believe it's caused by humans.\n* Nobody can agree on the right way to fix it, but any solution will likely be difficult and expensive."
]
} | []
| []
| [
[],
[],
[],
[
"http://www.snopes.com/1912-article-global-warming/"
]
]
|
||
h6fal | How do trees grow in such a way as to not fall over? | How do trees know how to grow branches in such a way as to maintain their balance? I've never heard of a tree falling over because it grew incorrectly. | askscience | http://www.reddit.com/r/askscience/comments/h6fal/how_do_trees_grow_in_such_a_way_as_to_not_fall/ | {
"a_id": [
"c1sxafm",
"c1sxck9",
"c1sz67e",
"c1szbpf"
],
"score": [
7,
2,
2,
2
],
"text": [
"I think it has more to do with the roots than with the branches. If trees had to balance their branches in order to stay upright, they would get blown over all the time.",
"I'm not a scientist in any way, but all I can guess at is that any trees that did not effectively balance themselves due to genetics would die and not reproduce. ",
"Woody plants have a terrific and interesting property---they grow in response to mechanical stress (force per unit area). Thus, when a situation arises which changes the local stress anywhere (a big branch falls off, the ground tilts, a branch comes to rest on a fence, etc.), the tree/plant reacts and adapts by growing more or less support in those affected areas. Obviously, these stresses arise mainly from gravity, but not exclusively.\n\nIt is a truly interesting adaptive mechanism, and several books have been written about it. Look for \"Trees: The Mechanical Design\" and \"Design In Nature: Learning from Trees\" by Claus Mattheck, and, more generally, \"Trees: Structure and Function\" by Zimmerman and Brown. The book \"The Adaptive Geometry of Trees\" by Horn, though, deals only with organization of the crown and small branches according to the influence of light.\n\nTerms relating to plant growth phenomena are \"geotropism\", \"phototropism\", and \"apical dominance.\"",
"Clearly you've never tried to dig up a tree before.\n\nThe answer is the roots. They grab onto the ground and they never let go. "
]
} | []
| []
| [
[],
[],
[],
[]
]
|
|
j6md7 | How did the idea of the atom advance beyond a theory? (other questions inside) | Hi /r/askscience,
I'd like to get a better understanding of atoms. How was this idea proposed in the first place, and how did they end up proving it? Was the atomic bomb and nuclear bomb the first instances of being able to smash/slice an atom?
I've also noticed that for many elements they have the same amount of protons/neutrons/electrons. Is there any relation between them? As from my understanding uranium has 92/92 and 146 neutrons. Is there any equation or reason to the way that atoms are structured?
Why must an atom have a neutron if it already has a proton and electron?
Finally, quarks are the smaller version of atoms, no? | askscience | http://www.reddit.com/r/askscience/comments/j6md7/how_did_the_idea_of_the_atom_advance_beyond_a/ | {
"a_id": [
"c29knjl",
"c29kt54",
"c29kwkm",
"c29l33z",
"c29knjl",
"c29kt54",
"c29kwkm",
"c29l33z"
],
"score": [
18,
4,
10,
8,
18,
4,
10,
8
],
"text": [
"Atomic theory came from chemistry, from the realization that there must be little indivisible bits of each element. (Actually atomic theory is often said to go back to antiquity and the Greeks, but that's a bit misleading, because until the 19th century it all amounted to little more than \"I bet stuff has bits in.\")\n\nSo the chemists did their thing for about a hundred years, and came to the conclusion that you can't ever have less than *this much* of any element, and in fact all stuff is made of integer multiples of these little indivisible bits.\n\n*Subatomic* theory didn't come until about a hundred years later, with the discovery of the electron.\n\nI'm not sure where you got the idea that different elements can have the same number of protons. That's the *opposite* of the truth. Elements are *distinguished* by the number of protons in their atoms. That's why the periodic table of the elements has one entry for each number of protons, starting at one — hydrogen — and counting up to 118 or whatever we're up to now.\n\nThe number of *neutrons* in an atom, for a given number of protons, distinguishes *isotopes.* Some isotopes are stable, and others are unstable and will decay into more and more stable forms over time. This is the source of radioactivity.\n\nThe number of *electrons* in an atom can change essentially any time, through a process called ionization. Chemistry is concerned almost entirely with the way the electrons in different atoms interact.\n\nAnd no, quarks are not \"the smaller version of atoms.\" They're in fact completely different. *Completely* different. Once you get past \"here are some things that are too small to see,\" you're basically done comparing the atomic model to the quark model.",
"[This series](_URL_0_) is an excellent overview of how chemistry was developed, and which experiments supported the theories.",
"Let's be clear, your question is really, \"How did atomic theory advance beyond a hypothesis?\"",
"Theory isn't a pejorative in science.",
"Atomic theory came from chemistry, from the realization that there must be little indivisible bits of each element. (Actually atomic theory is often said to go back to antiquity and the Greeks, but that's a bit misleading, because until the 19th century it all amounted to little more than \"I bet stuff has bits in.\")\n\nSo the chemists did their thing for about a hundred years, and came to the conclusion that you can't ever have less than *this much* of any element, and in fact all stuff is made of integer multiples of these little indivisible bits.\n\n*Subatomic* theory didn't come until about a hundred years later, with the discovery of the electron.\n\nI'm not sure where you got the idea that different elements can have the same number of protons. That's the *opposite* of the truth. Elements are *distinguished* by the number of protons in their atoms. That's why the periodic table of the elements has one entry for each number of protons, starting at one — hydrogen — and counting up to 118 or whatever we're up to now.\n\nThe number of *neutrons* in an atom, for a given number of protons, distinguishes *isotopes.* Some isotopes are stable, and others are unstable and will decay into more and more stable forms over time. This is the source of radioactivity.\n\nThe number of *electrons* in an atom can change essentially any time, through a process called ionization. Chemistry is concerned almost entirely with the way the electrons in different atoms interact.\n\nAnd no, quarks are not \"the smaller version of atoms.\" They're in fact completely different. *Completely* different. Once you get past \"here are some things that are too small to see,\" you're basically done comparing the atomic model to the quark model.",
"[This series](_URL_0_) is an excellent overview of how chemistry was developed, and which experiments supported the theories.",
"Let's be clear, your question is really, \"How did atomic theory advance beyond a hypothesis?\"",
"Theory isn't a pejorative in science."
]
} | []
| []
| [
[],
[
"http://www.youtube.com/watch?v=L2zzH7K7eeE"
],
[],
[],
[],
[
"http://www.youtube.com/watch?v=L2zzH7K7eeE"
],
[],
[]
]
|
|
5uf2rk | AskScience AMA Series: We're a group of paleontologists here to answer your paleontology questions! Ask us anything! | Hello /r/AskScience! Paleontology is a science that includes evolution, paleoecology, biostratigraphy, taphonomy, and more! We are a group of invertebrate and vertebrate paleontologists who study these topics as they relate to a wide variety of organisms, ranging from trilobites to fossil mammals to birds and crocodiles. **Ask us your paleontology questions and we'll be back around noon - 1pm Eastern Time to start answering!**
***
Answering questions today are:
- **Matt Borths, Ph.D.** (/u/Chapalmalania): Dr. Borths works on the evolution of carnivorous mammals and African ecosystems. He is a postdoctoral researcher at Ohio University and co-host of the [PastTime Podcast](_URL_1_). Find him on Twitter @PastTimePaleo.
- **Stephanie Drumheller, Ph.D.** (/u/UglyFossils): Dr. Drumheller is a paleontologist at the University of Tennessee whose research focuses on the processes of fossilization, evolution, and biology, of crocodiles and their relatives, including identifying bite marks on fossils. Find her on Twitter @UglyFossils.
- **Eugenia Gold, Ph.D.** (/u/DrEugeniaGold): Dr. Gold studies brain evolution in relation to the acquisition of flight in dinosaurs. She is a postdoctoral researcher at Stony Brook University. Her bilingual blog is _URL_0_. Find her on Twitter @DrNeurosaurus.
- **Talia Karim, Ph.D.** (/u/PaleoTalia): Dr. Karim is the Invertebrate Paleontology Collections Manager at the University of Colorado Museum of Natural History and instructor for the Museum Studies Program at CU-Boulder. She studies trilobite systematics and biostratigraphy, museum collections care and management, digitization of collections, and cyber infrastructure as related to sharing museum data.
- **Deb Rook, Ph.D.** (/u/DebRookPaleo): Dr. Rook is an independent paleontologist and education consultant in Virginia. Her expertise is in fossil mammals, particularly taeniodonts, which are bizarre mammals that lived right after the non-avian dinosaurs went extinct! Find her on Twitter @DebRookPaleo.
- **Colin Sumrall, Ph.D.**: [Dr. Sumrall](_URL_2_) is an assistant professor in the Department of Earth and Planetary Sciences at the University of Tennessee. His research focuses on the paleobiology and evolution of early echinoderms, the group that includes starfish and relatives. He is particularly interested in the Cambrian and Ordovician radiations that occurred starting about 541 and 500 million years ago respectively. | askscience | https://www.reddit.com/r/askscience/comments/5uf2rk/askscience_ama_series_were_a_group_of/ | {
"a_id": [
"ddti2p9",
"ddtidhb",
"ddtidx6",
"ddtie3z",
"ddtig83",
"ddtigsr",
"ddtiip1",
"ddtij3r",
"ddtije4",
"ddtijxy",
"ddtinjq",
"ddtio9d",
"ddtip45",
"ddtiqv8",
"ddtir7u",
"ddtiraj",
"ddtixhn",
"ddtj26y",
"ddtj3bb",
"ddtj4g9",
"ddtja5n",
"ddtjc5b",
"ddtjdyt",
"ddtjeq3",
"ddtjf6z",
"ddtjg30",
"ddtjikc",
"ddtjoaf",
"ddtjoqa",
"ddtjoy3",
"ddtjs0p",
"ddtjt5i",
"ddtk0va",
"ddtk1w4",
"ddtk702",
"ddtkaos",
"ddtkg2n",
"ddtkkf2",
"ddtkko7",
"ddtkllm",
"ddtktfh",
"ddtku1m",
"ddtl533",
"ddtl5b0",
"ddtlgxm",
"ddtlte8",
"ddtlv0p",
"ddtlwig",
"ddtm8yb",
"ddtmao3",
"ddtmg91",
"ddtmhno",
"ddtmj1i",
"ddtmp37",
"ddtmpkp",
"ddtmr1l",
"ddtmthu",
"ddtmtyg",
"ddtn0ng",
"ddtn51s",
"ddtn8qr",
"ddtngsk",
"ddtnhtj",
"ddtnome",
"ddtnrdl",
"ddtntoc",
"ddtnwna",
"ddto8nq",
"ddtoaku",
"ddtowlb",
"ddtp7vs",
"ddtqlta",
"ddtqwt5",
"ddtr1np",
"ddtrevc",
"ddtrptt",
"ddtscx0",
"ddttm1q",
"ddtujsq",
"ddtunnf",
"ddtv7d0",
"ddtvnby",
"ddtvxyk",
"ddtyf5i",
"ddtyty6",
"ddu6t3h",
"ddu7nsi",
"ddu7vij",
"ddua66z",
"dduaewp",
"ddub9fw",
"ddubhf1",
"ddubief",
"dducm22",
"ddudhy9",
"ddugfzx",
"dduitk6",
"ddujcz7",
"ddum6gg",
"ddupqxs",
"dduptnw",
"ddut2zp"
],
"score": [
22,
48,
27,
117,
15,
6,
19,
241,
29,
10,
6,
39,
6,
4,
76,
3,
4,
3,
126,
17,
13,
20,
24,
46,
4,
5,
3,
3,
66,
30,
47,
2,
104,
17,
2,
4,
8,
4,
8,
23,
7,
4,
10,
9,
18,
22,
8,
13,
2,
14,
7,
2,
3,
6,
17,
5,
83,
2,
2,
3,
2,
2,
3,
3,
2,
13,
3,
9,
3,
4,
2,
2,
2,
2,
3,
3,
4,
5,
3,
2,
4,
7,
2,
3,
2,
5,
3,
2,
2,
2,
2,
3,
2,
2,
4,
2,
2,
2,
2,
2,
2,
2
],
"text": [
"How did the ancestors of crocs fare during the mass extinction event 65M years ago?\n\nDid they actually survive, or was it just a case of convergent evolution? If yes, why and how did they survive?",
"What do you think is some of our greatest evidence supporting human evolution?\n\nAlso, in the age of anti-intellectualism, how do we help people trust experts again?",
"What are all of your favorite dinosaurs, and what time period are they from? Why?",
"Which mass extinction do you think had the biggest impact on the evolution of life? I know there have been several and the one that occured 65 million years ago is by far the most well known, but maybe not the most important?",
"Are any of you worried about the current administration defunding your research? ",
"Are there any lesser known but important human ancestors like lucy? ",
"Why is it that birds survived the K/T extinction, while other, similar species did not? Along with this question, did other bird-like animals survive the extinction event, and die out afterwards?",
"In your field, what is the biggest unsolved mystery or unexplained fossil that you want answered? ",
"Dr. Drumheller, I feel like I may have asked you this years ago, but did your name have anything to do with your career path?\n\nGeneral question, what is the current consensus on how bird-like dinosaurs are on the lizard-bird spectrum? Are basically all dinosaurs thought to have been feathered now, or just certain upright walking ones? Is there a transition in the fossil record where dinosaurs starting seeming more like birds?\n\nAnd, what is the most surprising thing you've learned from your research?",
"What \"big break\" is the paleontology world waiting on / in your respective fields? What finding has been hypothesized and \"it's only a matter of time\" before its found?\n\nOr, what is the \"big debate\" going on right now in paleontology?",
"Do you think that humans are due for a mass extinction event or have we (some of us) evolved enough to adapt? And if humans went extinct what do you think would supersede us as the dominant species?",
"I live in Eastern Kentucky. One of my favorite pastimes is walking down the creek hunting for fossils. How is there so much variety of fossilized aquatic life in southern Appalachia? Often I find several different varieties, sometimes even salt and freshwater remains on the same matrix. Very exciting to find but makes you go \"huh?\"",
"Dr. Sumrall - if there were large species radiations in the Cambrian and Ordovician for echinoderms, what is the current thinking about the triggering event? Did this occur immediately after a mass extinction?",
"Is it possible to extract the dna out of a fossil and make a new creature",
"Did any dinosaurs survive the mass extinction event 65m years ago?",
"How do early echinoderms look different from modern echinoderms? Considering how morphologically diverse deuterostomes are today, how similar or different are early deuterostome groups?",
"1. What do you think was the reason for the Cambrian explosion ? In comparison, how fast / slow is evolution today ?\n\n2. What lessons for humanity do you find in the sands of time ?",
"In a dinosaur death match which features one of every kind of dinosaur, which dinosaur would you bet on?",
"What is the predominant theory right now as to when, how, and why warm-bloodedness developed in both early mammals and birds? Since they are two different branches off of reptiles is being warm blooded a case of convergent evolution? Or did it develop in a common ancestor of birds and mammals? ",
"Do you think dinosaurs tasted like chicken?",
"What do you think it would be the biggest paleontology discovery that would rock the world right now? (Besides the transitional fossil for humans)",
"What's your favorite megafauna?",
"Would you have rather seen the movie titled Cretaceous park for accuracy, even though it doesn't sound as cool as Jurassic?",
"What is the general process of becoming a palaeontologist and where will you find your career path going generally? \nHow did you all decide which species you wanted to specialize in and how did you come to that point through the education process? \n\nWhat's the most exciting/most boring part of your job and how much education did it take to kickstart your career? :)\n\nThanks! \nEdited for clarity",
"Do any of you subscribe to the phylogenetic species concept, why/why nor?",
"I have an Anthropology degree, what are some entry level jobs in the field?",
"What period is mboscodictiasaur from?",
"How many \"Ross Geller\" have you meet ?",
"What prehistoric creature would you be most scared of if it was still around today?",
"I have two questions.\n\nFirst what species have undergone the greatest changes in the past 20,000 years and what are the likely reasons behind those changes?\n\nSecond, I have a bachelors in wildlife and conservation biology. Would a paleontology masters program accept someone without a directly related degree?",
"My son loves dinosaurs. What is the coolest dinofact I can tell him? \n\nAlso, how do you combat the indominus rexification of dinosaurs? He used to loves t-Rex but all the kids at school try to tell him that indominus rex is better. Every day he has to re-learn that indominus rec is not real.",
"Is it possible for intact DNA from dinosaurs to be preserved and if so, what would the conditions for it to be preserved, where would you find it and what would the chances of it being found or appearing be?",
"I have a question about dinosaur intelligence. I've heard it said that the current scientific consensus is that even dinosaurs with a reputation for being \"smart\", like Velociraptor (and maniraptorans in general) were actually probably only about as intelligent as a cat. The reason usually given is that their brains were not large enough.\n\nMy question is, some avian dinosaurs such as corvids and parrots are known to be very intelligent. Do these species of birds have notably large brains compared to their bodies? If not, how do we know that there were no dinosaurs with, for example, crow-level intelligence?",
"I feel like I'm wasting your time, but anyways, how often (if ever) do you utilize geophysics to locate fossils? I figure some form of seismology and/or GPR could be useful? Also, Dr. Drumheller is in the right field with a name like that.\n\nEDIT: how many species of dinosaurs are showing evidence of having had feathers!?",
"Is it scientifically imposible to one day clone (in lack of a better word) some of the extinct prehistoric species? ",
"In all seriousness, what came first: the chicken or the egg? What are the leading theories to explain this?",
"What are you opinions about the ability to clone an extinct species? Would it be ethical to try and reintroduce it to its previous environment, or could they just be used as test subjects or in zoos? If ethics and logistics were out of the picture, what species would you most like to bring back? ",
"Can you recommend some good books on the subject for someone who doesn't have a lot of exposure to the field? Kind of like A Brief History of Time did for cosmology?\n\nI always love learning in new fields but a lot of academic works usually require having already read a whole truck load of other works.",
"Hey guys! \n\nThanks for doing this AMA with us today! \n\nWhere would you say our current understanding of the extent of the biological component of Early Paleozoic and even Precambrian paleosoils now stands? Was life generally abundant in these soils or was it a hit-and-miss localised occurence?\n\nWhen I was an undergraduate, there was a pretty broad consensus that surface environments older than, say ... about the Devonian, were pretty barren, to the point where they might as well be considered abiotic. This, of course, had all kinds of implications for modelling weathering, erosion and sedimentology in general.\n\nBut I'm getting the idea that there is growing evidence that soils as far back as the Precambrian were replete with biological activity, mostly from [isotopic evidence but also microfossils](_URL_0_). I've even come across sparse reports of Paleoproterozoic macrofossils (e.g.: [*Diskagma buttonii*](_URL_1_)) which sort of blew my mind (although I'm still skeptical it's really an organism and not some microbially induced sed-structure).\n\nSo : what kind of \"creepy-crawlyness\" should one expect from a Cambrian soil, or a mid-Proterozoic one? Rare extremophiles in the pore space or abundant complex communities of microbes in pores, mats and biofilms? Or something else?\n",
"Specific question here, because plesiosaurs are my favorite... \nIf a plesiosaur is not a \"dinosaur,\" is it simply called a marine reptile? Are they catagorized different because of the time period, and being aquatic?",
"If a mass extinction were to occur due to humans in the recent future, which species would you expect to survive and why?",
"1. What is the most interesting thing/fossil you've ever worked with? What made it interesting?\n\n2. Do you have a favourite fossil?\n\n3. Are there any books regarding paleontology you'd recommend?\n\nAlso: I want to thank you for this AMA, Paleontology is by far my favourite science, I hope you have a great day!",
"Is Brontosaurus really \"accepted\" again in the paleontology community or is there a lot of controversy still after the studies done by Emanuel Tschopp?",
"For Dr. Karim: I heard once that part of the reasons museums still have outdated positions for dinosaurs, graphics, etc. is just a basic lack of funding to update the collections. Given the advent of, say, 3D scanning and printing substances, augmented reality and the like, how possible would it be to keep such collections up-to-date for the public experience? Example: Say a kind of museum peer-to-peer wikipedia to display common information of a species on-hand or for part of a traveling exhibit, but also integrate specifics for an individual piece (especially identifying markers on a particular specimen)?\n\nI feel that the public still harbors a '90s view of dinosaurs / prehistoric life and for that to change, museums need to change; so what would be a good way to help facilitate that?",
"My son is absolutely fascinated with dinosaurs and other prehistoric creatures. What kind of resources are there if any to help support his hobby and desire to be a paleontologist some day?\n\nHe has his own book shelf that's almost full and devoted to the subject, and we're talking about taking a trip to South Dakota to see a dig site. Any other suggestions or resources that might be available?",
"What do you think is the biggest myth that semi-educated people believe when it comes to prehistoric times. \n\n",
"My six year old wants to be a paleontologist when he grows up (more specifically, a fire-fighting paleontologist!). He's been consistent on this for the past 4 years, so he seems as serious as a six year old can be.\n\n Aside from trips to museums, what can we do to nurture his interest?\n\nBonus points if you can provide answers that also include firefighting! ",
"Is there any evidence at all as to what color dinosaur feathers may be?",
"Which came first? The dinosaur or the egg?",
"I have a degree in biology, but work in a completely different field. Since I was a child, I always wanted to be a paleontologist. Any suggestions as to how I could volunteer in your field? I live in Florida, and have frequented phosphate mines in the past, but that's the extent of my specimen hunting experience.... digging around the excavated material (I wouldn't disturb something untouched). My current job pays the bills, but I do miss the things I was passionate about when deciding to major in biology. Any suggestions?",
"Dr. Borths - How have modern ecological conditions impacted the time line of evolution in carnivorous mammals in African ecosystems? Have anthropogenic influences effectively \"selected\" for a different type of mammal in the last 50kya? 100kya?\n\nDr. Gold - What kind of correlation is there between brain evolution and the capacity for flight in dinosaurs that you have found?\n\nDr. Karim - What are some of the most significant challenges you have come across in both interpreting archeological finds and curating them in a way that makes the data easily digestible by non-scientists? What are some of the new technologies you are utilizing to digitize these collections?\n\nDrs. Drumheller, Rook, and Sumrall - Your specialties sound incredibly interesting, but I know absolutely nothing about them! How did you come across such niche specialties, and if you had to pave your own way by creating a subfield, how did you do it?\n\nThank you all so much for doing an AMA! Your research is instrumental in understanding the vast scope of our planet's evolutionary history and I hope to one day contribute to science like you all do. :)",
"Is it true we don't know what color dinosaurs really were?",
"Hiya. Do you ever look for fossils in areas that have a lot of crude oil deposits? Is there any correlation there? Conversely do you ever find surface crude oil to be an indicator of fossils? ",
"Could you describe how/why the evolution of feathers proceeded differently than hair? I.e., how do you theorize that each incremental step of feather evolution increased fitness enough to disproportionately get inherited by the next generation until we reached fully flighted birds?",
"Do you find the character of Ross Geller in Friends, surely TV's most high profile paleontologist, a flattering comparison? Was there a boom in interest in the field that people, to this day, call the \"the Friend's boom\"? ",
"What are your theories about when humans developed a conscious mind and why/how? Is it possible other animals could develop this as well?",
"My 5 year old daughter has two key questions: How do we know what dinosaur's names were? How do we know what sounds dinosaurs made?",
"Do we have any fossil evidence of when or how vascularization in plants evolved?",
"Would you consider there to he a difference between a paleontologist and a paleobiologist?",
"What is the most prevalent and/or frustrating misconception you wish you could clear up about your field?",
"Would it be possible to \"clone\" dinosaurs at all. From what I've heard the DNA actually breaks down (even preserved in amber.) Is there any way at all to reconstruct a dinosaur by using current animals and tweaking them or something like that? Then, would they survive an amusement park type captivity if we spared no expense? (asking for a friend.)",
"Of all the species that disappeared during the Holocene extinction, which one's loss do you think had the biggest impact on what our world looks like today? ",
"Hi Dr. Sumrall. I've read about stalked echinoderms attached to fossil drift wood. If this is true, do you believe that these creatures were floating around or do you think the wood had already sunk to the seafloor when the creatures attached?",
"I was just reading about the unit of measurement , the Darwin, and didn't really understand it. Do you really use that? How does it work on a practical level? How do you determine how much, say humans have changed compared to a chimpanzee since their common ancestor?",
"Sorta towards Dr. Karim ... I live in Colorado, specifically in the Western Slope area. A museum supposedly in the GJ area went out-of-business and sold-off most of their collections, but had boxes of dinosaur bone-parts/pieces that no one seemed to know what to do with, so they took them out into the desert and dumped them. People showed up and started taking \"samples\" of these bones and there was even a guy there directing people to the pile of bones. I ended up with 2 big coffee cans filled with pieces of dinosaur bones. Is this something that happens a lot? The guy sitting there said the bones came from this area and the museum wanted to return them to approximately where they were dug out. I posted some of the fragments on Imgur which I found there: _URL_0_",
"Was Ross's version of a paleontologist in friends, accurate?",
"What is your stance on the aquatic ape hypothesis?",
"I really enjoyed the \"Walking with Dinosaurs/Beasts/etc\" series of documentaries, even though I've heard they're not entirely accurate. What are some dinosaurs/extinct species documentaries you'd recommend? Also interested in book and especially online resources you'd recommend for an interested layman.",
"What organism have you discovered to be extinct that if alive today would be our greatest benefit or vice Versa, our biggest threat predator(excluding T-Rex's and other obvious things)",
"1. Do you feel threatened by the new administrations' religious zealotry, especially in the figure of the VP? He said some things like \n\n > ...And I also believe that someday \nscientists will come to see that only the theory of intelligent design \nprovides even a remotely rationale explanation for the known universe...\n\n while addressing congress. [\\(source\\)](_URL_0_) There's also the additional travel restrictions that may prevent foreign scientists from entering the US and american scientists from entering other countries as they take retaliatory measures. Does any of this impact your work? If so, how will you circumvent it?\n\n2. How big a problem for you are the people who insist on keeping fossils and other historical evidence in their private collections, where they can't be researched, instead of in museums or universities' collections, where they will be well kept and available for researchers?",
"I wanna know if your appreciation for the job outweighs the instability and the lack of available jobs in the field and how you have managed that in your lives.... you guys have my dream job!",
"I've been applying for PhDs in palaeo for a while and had no luck. I have both a degree and a Masters in the subject. What advice could you give to those starting and searching for a career in research? \n\n",
"Visiting the Natural History Museum at the Smithsonian in D.C., what can't I miss??",
"I've always wondered, how do you get into the career of paleontology? As a kid Ive loved dinosaurs and now that I need to choose a path for my future life I thought I could check out this again. Studying fossils does sound cool. ",
"hello, can you look for microscopic fossils? how do people find such small fossils and be certain it was life and not something else.",
"Not to be rude or anything but how much funding does the field get?",
"I'll be defending my thesis on mysticete whale paleoecology in a few hours. Any last-minute master's defense advice?",
"Colin, I heard you used to have a pony tail, what happened man??",
"How often do you get asked about Ross?",
"Thanks for doing this AMA!\n\nI'm a super amateur fossil hunter in MA and am having the hardest time with:\n\n\n-identifying fossil digging locations that are public access (only know of 2 locations in MA) near me\n\n\n-extraction techniques for extremely hard shale\n\n\n-identifying which layers are fossiliferous \n\n\nAny tips on the above topics?\n\n\nAlso, I found a bird fossil in Utah about 10 years ago. It's an OK fossil but, my 13 year old brain couldn't figure out how to extract the entire fossil properly so I'm stuck with an mostly complete baby bird sans back feet. I figure it's not particularly useful to the scientific community since I can't recall where I pulled the fossil from...what should I do with it?",
"Are there any names of creatures you've researched that kinda make you chuckle/laugh at when you think about it? ( Like just a silly sounding name )",
"My 6 year old son is obsessed with dinosaurs, and says he wants to study them when he grows up...his specific plans are to make a dinosaur zoo, and be their zookeeper. Anyways, he has questions for you guys, if you don't mind...\n1. What is your favorite dinosaur? \n2. Why are t-rex arms so short? \n3. Did all of them die? \n4. How do I make more?\n5. What grade do you have to be in to study dinosaurs? (I believe he means how much schooling is required to become a paleontologist?) \n6. Do you want to work at my zoo?",
"What's your favorite paleontology joke?",
"Sorry if this is a dumb question, but how do \"big traits\" evolve from an animal that never had it? For example the evolution of a dinosaur that doesn't fly to a bird with wings. How does something that never had anything that looked like wings start to develop them, when the in-between stage of the wing which cannot fly wouldn't be helpful at all? Thanks and I have been wondering his for a long time.",
"Would paleontologists identify different dog breeds as the same species, just from the bones?\n\nSuppose a distant future where all cultural evidence of dog breeds has been lost, and only fossilized bones remain. (No \"transistional\" fossils are found, either. Just a snapshot of dog breeds as they exist today.) It seems that in paleontology, even minute differences in bone structure can be very important for distinguishing related species. So would a scientist recognize the bones of a bulldog (short nose, short legs, barrel-chested) and German Shepherd (long nose, long legs, narrow rib cage) as being from the same species? Or is it likely that without DNA analysis, they would be miscategorized as being completely distinct species?",
"Can you please write an easy to follow explanation on how carbon dating works? I want to know why I should believe that a 125 million year old bone is 125 million years old. \n\nThank you :) ",
"I found [this](_URL_0_) in the tailings pile of an excavation. The guy I was with (who brought me there looking for fossils) said he thought it was bone, but the Manning Canyon Formation wasn't supposed to have anything with bones this big.\n\nI figure my best bet is to take it to the local university, short of that is there any characteristic I can look for to determine if it is bone?",
"In an effort to understand metazoan evolution, I am interested in the Cambrian explosion. I've read about drops in sea level that lead to the exposure of nutrients from continental crust. What other kind of geologic events took place around that time that made it possible for metazoans to evolve? Thanks for doing this! ",
"Have we found a complete or near complete spinosaurus skeleton?\n\nHow big was predator X?\n\nWere velociraptors actually small?\n\n",
"Do any of you accept that creation is a viable way in which life on earth could've come about?\n\nIf not, why don't you believe that it's viable?\n\nAnd would the scientific community look down on an individual for choosing to believe in a creator? What would the general attitude toward such individuals be? ",
"Are there any large animals alive today, that evolved after the dinosaurs went extinct, that could've evolved if they hadn't died out? I've always heard that if the asteroid didn't hit we wouldn't have been here and neither would pretty much any mammal bigger than a lemur.\n\nEDIT: A few more questions. \n\n1. What, other than lack of feathers, do most dinosaur movies get wrong? Compared to other dinosaur franchises is Jurassic Park pretty good in terms of accuracy, or do you know of any dino-based movies/games that really are pretty accurate?\n\n2. Are there any living animals (individual or pack) that have a chance at beating large dinosaurs like the tyrannosaurus, ankylosaurs, spinosaurus, or triceratops in a fight?\n\n3. If, theoretically, the dinosaurs/extinct creatures of an area of the world (say, the Indian subcontinent) were spared from the extinction, would humans be able to live there? Could they maybe have been domesticated, or would they not meet the requirements for domestication?\n\n4. Let's say we brought a dinosaur back here. Would it have trouble breathing or was the air at the time not too different? How about even earlier eras, like a gigantic dragonfly?",
"Hi. My daughter is five, and loves dinosaurs. She knows more six and seven syllable words than I do at this point. All she has talked about for almost a year now is that she wants to be a paleontologist. What can/should I do as her Dad to help her expand upon her interest in the subject?",
"You must be a fossil sample, and I'm an impatient paleontologist - because I want to date you badly.",
"Sorry not even sure if this AMA is still happening right now but I had a question or two about you guys. First, did you find it difficult to get a secure job and actually into the field doing work? And second, (if allowed hehe) I am a marine science uni student, finishing my degree this May, and I would really be interested in palaeontology, is there anyway you would suggest to start the journey? (Like is another degree course the right thing to do ((please say no more debts sake))) Thanks very much :)",
"My little girl is 5, she has autism and she wants to be a palaeontologist when she grows up. She can name nearly every dinosaur, as well as their family groups and features, her favourite dinosaurs are the pachyrhinosaurus and the muttaburrasaurus. We live in Melbourne, Australia, and we are currently planning a trip to outback Queensland where there is (I believe) the largest collection of fossilised dinosaur footprints in one place, as well as the largest collection of marine fossils from what used to be an inland sea.\n\nMy questions are:\n apart from lots of reading about dinosaurs, and watching documentaries (she loves watching dinosaur docos!) what else would you suggest to foster her love of fossils?\n\nDo you have many women in your field?\n\nAnd\n\nDo you have many autistic palaeontologists that are successful?\n\nThanks\n\nETA: my daughter is also always asking me how the first mammals (like cynadonts) evolved - do they have any ancestors today or did they diversify too greatly?",
"I'm trying to nurture my niece 9, and nephew 7, love for science and I seem to be winning them over. I'm fascinated by paleontology and was wondering if there are any kid oriented projects, kits, magazines or books you would recommend to educate them? Any words of advice or encouragement young scientists?",
"In terms of the dinosaurs we think of today (not birds), how long did dinosaurs last after the meteor crash that initiated the K/T extinction? Was it for a very short period of time or perhaps many millenia?",
"Am I too late to ask a question? If not (I hope not);\nSay we find 1 fossil of any given animal, and it's the only fossil we've ever found of its species, ever. However, it has a mutation like polydactyly, dwarfism, or something similar - noticeably different from the rest of its kind. \nBut, because we only know its species from 1 fossil, would we just assume that every animal of its species looked like that? Or would we be able to look at a fossil and say \"yeah this whateverodon has 1 more toe than the rest of its species, even though we've never seen any other whateverodon fossils before\"?\n\nI mean, there're a lot of animals out there with super weird anatomy, and a lot of animals we only know from 1 or 2 fossil specimens... Would this be a completely impossible scenario? \n\nAlso, have we ever actually found a fossil of a \"mutant\" animal at all, one that we *could* say is different from the rest of its species?",
"Hi Ross! I'd like to know if you still think you were on a break. Thanks!",
"I know very little about paleontology but something I've always wondered is if paleontologists are religious. If so, what religions are most common?",
"Are there any areas of paleontology that are hampered because the only/most abundant source for fossil evidence is in an unstable and/or war torn country? ",
"What's Ross up to these days? "
]
} | []
| [
"www.DrNeurosaurus.com",
"http://www.pasttime.org/",
"http://eps.utk.edu/faculty/sumrall.php"
]
| [
[],
[],
[],
[],
[],
[],
[],
[],
[],
[],
[],
[],
[],
[],
[],
[],
[],
[],
[],
[],
[],
[],
[],
[],
[],
[],
[],
[],
[],
[],
[],
[],
[],
[],
[],
[],
[],
[],
[
"https://blogs.uoregon.edu/gregr/files/2013/07/Beraldi-Campesi-and-Retallack-2016-Precambrian-life-on-land-18qz7jl.pdf",
"https://www.researchgate.net/profile/Gregory_Retallack/publication/276944235_Problematic_urn-shaped_fossils_from_a_Paleoproterozoic_22Ga_paleosol_in_South_Africa/links/55c37e3e08aeb975674014d4/Problematic-urn-shaped-fossils-from-a-Paleoproterozoic-22Ga-paleosol-in-South-Africa.pdf"
],
[],
[],
[],
[],
[],
[],
[],
[],
[],
[],
[],
[],
[],
[],
[],
[],
[],
[],
[],
[],
[],
[],
[],
[],
[],
[
"https://imgur.com/a/eSdHW"
],
[],
[],
[],
[],
[
"https://www.congress.gov/congressional-record/2002/7/11/house-section/article/h4527-1?q=%7B%22search%22%3A%5B%22%5C%22mike+pence%5C%22%22%5D%7D&resultIndex=190"
],
[],
[],
[],
[],
[],
[],
[],
[],
[],
[],
[],
[],
[],
[],
[],
[],
[
"http://imgur.com/a/ifms7"
],
[],
[],
[],
[],
[],
[],
[],
[],
[],
[],
[],
[],
[],
[],
[]
]
|
|
9ue6yk | would hiv/aids cancel out leukemia? | explainlikeimfive | https://www.reddit.com/r/explainlikeimfive/comments/9ue6yk/eli5_would_hivaids_cancel_out_leukemia/ | {
"a_id": [
"e93jn6k"
],
"score": [
8
],
"text": [
"Not really, HIV is a virus, AIDS is a deficiency of CD4 white blood cells, and leukemia is a cancer causing elevated abnormal nonfunctioning white blood cells (there are more types than just CD4) \nThat about as ELI5 as I can get, there is alot more science behind it, between the progenitor cell lines and such. "
]
} | []
| []
| [
[]
]
|
||
3ttoqe | Why were the Channel Islands not liberated soon after D-day? | AskHistorians | https://www.reddit.com/r/AskHistorians/comments/3ttoqe/why_were_the_channel_islands_not_liberated_soon/ | {
"a_id": [
"cx9afim"
],
"score": [
86
],
"text": [
"There was a plan to liberate them as early as 1943, called Operation Constellation. Drawn up by Lord Louis Mountbatten, based on aerial reconnaissance.\n\nThe problem was that, as it stood, the Channel Islands weren't actually much of a strategic advantage. Obviously if the Luftwaffe were to establish a bombing base there, that would change, but until 1945 the Germans hadn't made much of an advantage of the occupied islands. Really, the only purpose they had served for the Germans, was as a slight bloodied nose for the British.\n\nMountbatten observed that while the islands could be retaken by force, they were also heavily fortified, and it would require a fairly large landing force to retake these strategically unimportant islands.\n\nThe other consideration was the civilian population. They hadn't fared too badly under the Germans, there were no reports of abuses or reprisals, but attempting an invasion might put their lives at risk. Certainly their lives would be at risk if a battle broke out.*Mountbatten also noted that either aerial or naval bombardment would be necessary in order to break the islands defences, again at a potentially high civilian loss. \n\nMountbatten's plan was eventually rejected on the grounds that the cost would likely be far too high, and that the sensible option was to have them peacefully returned as part of Germany's unconditional surrender.\n\nI'm afraid the only source I have for this is Lord Louis Mountbatten's biography, *\"The Life and Times of Lord Mountbatten\"*, John Terraine, 1968.\n\nThere is also a wikipedia entry for Operation Constellation, and most of it seems to back up what I've read in Mountbattens biography, for what it's worth:\n\n_URL_0_\n\n*Edit"
]
} | []
| []
| [
[
"https://en.wikipedia.org/wiki/Operation_Constellation"
]
]
|
||
fdbhvi | how did mining work back in the day? did people just picaxe long tunnels into the rock hoping they’ll find something useful? | explainlikeimfive | https://www.reddit.com/r/explainlikeimfive/comments/fdbhvi/eli5_how_did_mining_work_back_in_the_day_did/ | {
"a_id": [
"fjgbby4",
"fjgbhwm",
"fjgc38c",
"fjggkoe"
],
"score": [
26,
13,
8,
5
],
"text": [
"Yes and no. Depending on what's being mined.\nTypically they'd find a vein and surface level evidence. (Think of gold washed down a creek, it comes from a vein somewhere)",
"Historically, prospecting involved thoroughly searching a wide area looking for mineral deposits on the surface. The best places to look were creeklines or ridges/hills, and any mineral deposits signified that there was likely more beneath the surface.\n\n\nThe Romans used aquaducts for hydrolic mining, where they'd basically use a ton of water to wash away the soil to expose the rock and any veins of metal.\n\nThe first mines were largely for stone, and required less precise methods to locate good areas to mine.",
"It depends a bit on how long ago \"back in the day\" is. For the mines I've visited here in Norway that's around the 1600s. At that time, the mining was mostly accomplished with heat (wood fires) to make the rock brittle and manual picks or chisels. Using fires to heat the rock and then flash-cooling with water can also break rock faces apart. As chemistry advanced and explosives became cheaper and safer, they gradually took over. \nMining was a craft and geological experts were used to survey the mine and identify where to dig in what direction. Norway largely imported these experts from Germany and France, and you can still see some evidence of this foreign influence in for instance street names in [Røros](_URL_0_).",
"Before modern methods of geology, finding ore in the rock was often a matter of luck. The ore might have shown itself on the exposed surface, or it might wash out and deposit on the surface. If they found a large ore deposit, they could follow the ore veins deep into the rock.\n\nBy the way, pickaxes we know today aren't a typical mining tool, they're more for breaking heavy dirt and rock on the surface. Instead, they used hammers and chisels in order to be able to get highly precise strikes on the same spot over and over. That's why you can still see [this logo](_URL_0_) on the coat of arms of many historical mining towns in Europe."
]
} | []
| []
| [
[],
[],
[
"https://en.wikipedia.org/wiki/R%C3%B8ros"
],
[
"https://upload.wikimedia.org/wikipedia/commons/thumb/a/ac/Schlaegel_und_Eisen_nach_DIN_21800.svg/800px-Schlaegel_und_Eisen_nach_DIN_21800.svg.png"
]
]
|
||
4ajoum | how does someone go from being christian to jewish and vice-versa? | explainlikeimfive | https://www.reddit.com/r/explainlikeimfive/comments/4ajoum/eli5_how_does_someone_go_from_being_christian_to/ | {
"a_id": [
"d10wa3s",
"d10wqjz"
],
"score": [
2,
3
],
"text": [
"It really depends on the sect of either faith. I'm not an expert on Judaism so I won't comment there.\n\nTo become Christian you have to either (depends on the group):\n\n* Accept Jesus Christ as your Lord and Personal Savior. Either publicly or privately depending on the group.\n\n* Have a testimony or religious conviction that you have been saved by Jesus.\n\n* Be baptized. Some denominations might ask you to take classes/go through a program before doing so.",
"Judaism discourages conversion (but encourages people once they've been accepted for conversion). So you'd first have to convince a Rabbi that you're serious about wanting to convert. Then you'd have to engage in study so that you know enough about what it means to be and live as a Jew (which will vary depending on denomination). When your Rabbi thinks you're ready, you'll appear before a Rabbinic Court, and convince them that you're ready. If they agree, then you can go through the ritual immersion (in a ritual bath called a *mikveh*) and, in the case of men, circumcision. "
]
} | []
| []
| [
[],
[]
]
|
||
44q1tm | Can you recommend any sources (book or journal articles) that summarize issues related to Oral Tradition? | I'm researching John Rolfe, Thomas Rolfe and Pocahontas and I'm trying to evaluate Native American accounts and reconcile them with the standard historical accounts. I'm out of my depth re. the issues surrounding Oral Tradition. This is not homework, and I'm a teacher. Just trying to understand the issues better. | AskHistorians | https://www.reddit.com/r/AskHistorians/comments/44q1tm/can_you_recommend_any_sources_book_or_journal/ | {
"a_id": [
"czs5y2z",
"czsj1kf",
"czvz8do"
],
"score": [
3,
3,
2
],
"text": [
"All sources - every bit of evidence about the past - requires source criticism to evaluate what flaws have insinuated themselves into the weave of information presented. The difference between a written primary source and recorded oral testimony that is not first hand can be summed up with the following:\n\nHistorians continually find that primary source documents about the past have flaws that need to be identified and isolated from the telling of an accurate history and perception of the past.\n\nHistorians (and folklorists) are often surprised to find that a historical legend includes information that is accurate and can be relied upon in the telling of history and perception of the past.\n\nIn other words, historical legends are sometimes surprisingly accurate and primary sources are sometimes surprisingly inaccurate. Well, OK, maybe we're actually not that surprised about either, but there is a core assumption that primary sources depict the past (but we find they are flawed) and that historical legends are inaccurate and inflated beyond use (but we sometimes find they are useful).\n\nOf course folklorists don't often look to historical legends to understand the period described. Rather, they look at a historical legend to understand the people who told it - it is a different use of the source. But that aside, there has been ample analysis of historical legends and their surprising value in capturing the nature of past events.\n\nOf particular use here is a collection of essays edited by Reimund Kvideland and Henning K. Sehmsdorf: Nordic Folklore (1989). This includes essays by Lauri Honko, who is one of the great modern theoreticians when it comes to the nature of legend. A particularly good essay for your question is a chapter by Brynjulf Alver, \"Historical Legends and Historical Truth.\"\n\nThe bottom line is that oral tradition can be a surprising conveyor of accurate information about the past. As I began - all sources require source criticism. It is simply important to understand that written primary documents require a certain type of analysis while oral tradition requires another species of analysis. The filter for oral tradition must be more intense if it is to be used to understand an historical event, and without the anchor of written documents, it is difficult to use oral tradition, alone, to peer into a remote past.",
"/r/IndianCountry founder, mod, and Virginia Indian reporting in. The Custalow family at Mattaponi circulated and promoted a series of \"sacred oral traditions\" concerning this *exact* subject matter. A recent publication from NMAI afforded them too much credibility; one of many problems with that publication.\n\n[Disclosure: I am related to the author, the families, and the subject matter, so please pardon my brevity and generalities. And **kindly** don't mention me, rather, verify for yourself.]\n\nThe same is true for the supplemental oral histories promoted by the Tayac faction (the Piscataway Indian Nation), if you are also going to be teaching about entities contemporary to the Powhatan Paramountcy in the early colonial period.\n\nI'll defer to /u/itsallfolklore and /u/Reedstilt on the latest developments in academia, however I would recommend [Powhatan Foreign Relations](_URL_0_) if you're interested in the political landscape, in particular. Other works by Dr. Helen Rountree also provide generous footnotes (where her works *truly* live) for further inquiry. \n\nI'd also like to hear other recommendations to keep current, anyone would oblige.",
"You have received a message from on high! By which I mean someone on twitter had a suggested resource for you. [Check it out here!](_URL_0_) "
]
} | []
| []
| [
[],
[
"http://www.amazon.com/Powhatan-Foreign-Relations-1500-1722-Rountree/dp/0813914094"
],
[
"https://twitter.com/julian_thompson/status/697607494307332096"
]
]
|
|
atk6l6 | why is feet bone structure much more complex and composed of so many bones, while hands are much more simple? | Wouldn’t make sense for hands to be more complex than the foot due to the fact that hands are the much more used than feet are? | explainlikeimfive | https://www.reddit.com/r/explainlikeimfive/comments/atk6l6/eli5_why_is_feet_bone_structure_much_more_complex/ | {
"a_id": [
"eh1ixky",
"eh1q61n"
],
"score": [
25,
3
],
"text": [
"There are 27 bones in each hand. There are 26 bones in each foot. Can you please explain what you are talking about?",
"What makes you say the structure of a foot is more complex than a hand?"
]
} | []
| []
| [
[],
[]
]
|
|
1rte90 | what exactly happens to my body when i get the flu. | I know the symptoms of the flu, I want to know why? I am interested in the reason our body react the way it does (Body aches, Chills, Dizziness, Headache, lethargy, nausea and vomiting, sore-throat, fever, to name a few) I heard before is a way your body has to force you to take it easy and rest so that all your energy goes to fight the virus. I'd like an explanation of why specifically we have the symptoms we do with the flu, as opposed to, for instance, be momentarily paralyzed until the virus is gone. | explainlikeimfive | http://www.reddit.com/r/explainlikeimfive/comments/1rte90/eli5_what_exactly_happens_to_my_body_when_i_get/ | {
"a_id": [
"cdqpiu4"
],
"score": [
15
],
"text": [
"For most viruses, all the symptoms are your body's attempt to slow the progression of the virus while it searches for an antibody.\n\nThe virus is taking over cells and reprogramming them to produce more of itself. The only way to actually stop it is to produce the right antibody. I don't really know how that whole system works, but your body essentially produces millions of randomly shaped antibodies until the right one is made to stop the virus.\n\nTo explain some of the symptoms; a sore throat is acid being produced to kill off infected throat cells, this is a non-direct attack that kills healthy cells too. A fever raises the body temperature hopefully killing off or slowing weaker infected cells, chills make you seek warmth to crank your body temperature up even more, shivering produces more heat.\n\nVomiting speeds up the expulsion of dead cells. Your lymphatic system deposits them in your stomach, normally you would digest these and they would exit your body through stool and urine. But vomiting can get rid of a lot of stuff quickly, and it reduces strain on your kidneys that are also taking an increased load.\n\nThese responses are not perfectly utilized, over reaction of the immune system can make the symptoms way worse than they need to be. You vomit when there is nothing to vomit, and a high fever can kill you etc.\n\nSome viruses directly cause side effects that are not an immune response.\n\nIt is a lot more complicated than this and I don't know what I am talking about really, there are a lot more systems and doo-dads doing complex things in your body."
]
} | []
| []
| [
[]
]
|
|
c2tryo | The M60 was the only machine gun authorized by the Army to be used on gun trucks in Vietnam. How did officers react when they saw gun trucks armed with M2s and miniguns? | I just watched a short video about gun trucks in Vietnam on the Smithsonian's youtube channel. In the video they describe why and how gun trucks were built and they claim that the M60 was the only machine gun which soldiers were authorized to install on them.
However, soldiers preferred the M2 or the minigun so they found "alternative ways" to obtain them and then these "irregularly" armed gun trucks were used to protect convoys driving around the country.
I assume a high number of Army officers must have seen these gun trucks and that these officers were aware of the regulations regarding M2s and miniguns.
How come these gun trucks were still allowed to exist? Were there any consequences for using unauthorized weapons in this particular situation? And in general? | AskHistorians | https://www.reddit.com/r/AskHistorians/comments/c2tryo/the_m60_was_the_only_machine_gun_authorized_by/ | {
"a_id": [
"erosw8y"
],
"score": [
11
],
"text": [
"The officers were the ones leading the development of the Vietnam era gun trucks. Company, Battalion, and Group (Brigade equivalent) level transportation officers in Vietnam were heavily involved in these field modifications, and they also were the ones creating the doctrine to allow their units to use these gun trucks as effectively as possible. Not only that, many of them recorded their experiences to share and preserve the lessons learned and at least one Captain went through the effort to have a gun truck sent to the US Army Transportation Corps Museum to serve as an example for future generations. So this wasn't a dirty little secret the soldiers tried to keep to themselves, it was leadership pushing this as a reaction to the conditions in Vietnam (along with other tactics to protect convoys).\n\nNow when they say \"authorized\", they're referring to what's listed on the official Army \"TO & E\" (Table of Organization and Equipment) for that type of unit.\n\nA TO & E is a document that lays out the organization and weapons/vehicles/equipment for a given type of unit according to Army doctrine. Everything from how many soldiers they're supposed to have and how the unit is organized, to what type of weapons each individual soldier is assigned, to the types and numbers of vehicles used, what (if any) weapons are assigned to those vehicles, and so on. If something isn't on the TO & E, the Army isn't going to issue it to your unit, hence the backdoor deals to get added weapons in Vietnam. \n\nWhile the Army didn't adopt the idea of the gun truck at a higher level (it was never added to the TO & E and mostly forgotten after Vietnam), I'm not aware of anyone getting in trouble for any of these field modifications in Vietnam. While it was not necessarily viewed by everyone as the ideal long term solution to convoy security, it was also not viewed as a problem that needed to be stamped out.\n\nRichard Killblane, a historian for the US Army Transportation Corps (featured in the Smithsonian video you referenced) has written two papers on the history of US Army convoy security.\n\n[Circle the Wagons: The History of US Army Convoy Security](_URL_0_)\n\n[Convoy Ambush Case Studies](_URL_1_)"
]
} | []
| []
| [
[
"https://transportation.army.mil/history/publications/circle-the-wagons.pdf",
"http://www.vietnam-guntrucks.com/wp-content/uploads/2014/02/2227ConvoyAmbushVol1Proof141.pdf"
]
]
|
|
2hs5vg | air conditioning | explainlikeimfive | http://www.reddit.com/r/explainlikeimfive/comments/2hs5vg/eli5air_conditioning/ | {
"a_id": [
"ckvi87n",
"ckvj094"
],
"score": [
2,
2
],
"text": [
"[This](_URL_0_) is the basic cycle of the refrigeration cycle. There is a lot of thermodynamics but basically it works on the fact that the liquid inside the air conditioner has a low boiling point and can transfer thermal energy well/ as it it pumped through the cycle the liquid experiences phase changes and these changes cause a transfer in energy causing the heat from the outside to be taken out of the air and this cooler air blown out of the air conditioner\n",
"The air conditioner has a compressor which pumps a fluid called Freon into tubes outside the house which raises the Freon's pressure. And the opposite side of the compressor pulls the fluid into tubes inside the house through a tiny hole called an orifice, lowering the Freon's pressure. \n\n\nLow pressure causes boiling fluids which take in heat, but high pressure causes condensing liquids which give off heat. \n\n\nSo the low pressure soaks up the heat from the air inside the house while the high pressure sheds the heat into the air outside of the house. "
]
} | []
| []
| [
[
"http://upload.wikimedia.org/wikipedia/commons/5/5d/Refrigeration.png"
],
[]
]
|
||
17svrl | is there such a thing as a material that is completely impermeable? For example, can some molecules get though a plastic baggy? | my friend works at a museum and she was telling me that some water leaked in the ceiling of the museum so she had to give some of the exhibits a special treatment. She put lots and lots of silica jel in a plastic baggy, and placed it in an exhibit to absorb excess moisture in the air. I was wondering why she didn't have to leave the plastic baggy open to make it penetrable so that the moisture can be absorbed? | askscience | http://www.reddit.com/r/askscience/comments/17svrl/is_there_such_a_thing_as_a_material_that_is/ | {
"a_id": [
"c88nx91"
],
"score": [
2
],
"text": [
"Small molecules such as water may diffuse through most plastics, so even an airtight container is \"porous\" on a microscopic level. In any case, unless the bag is perfectly sealed by melting the plastic, chances are that the moisture can diffuse through small leaks at the sealing."
]
} | []
| []
| [
[]
]
|
|
4by43y | why is it a mark of quality for a watch to be "swiss-made"? couldn't a swiss company make a crappy watch? | explainlikeimfive | https://www.reddit.com/r/explainlikeimfive/comments/4by43y/eli5_why_is_it_a_mark_of_quality_for_a_watch_to/ | {
"a_id": [
"d1depmu",
"d1der15"
],
"score": [
3,
2
],
"text": [
"Of course - but like much advertising it trades on beliefs and mystique rather than facts.\n\nSwitzerland does produce some truly amazing watches, but it doesn't have a monopoly on it.\n\nNot sure whether they'd bother to produce really poor quality stuff these days simply because anything they could do badly the Far East could do just as badly but cheaper.",
"Cheap watches use what are called, \"stock\" movements. As to say, they didn't develop the actual movement, they just packaged it. Good watch manufacturers create their own movements. \n\nBeyond movements, complications make the watch more desirable and prestigious. Chronograph, world time, power reserve, stuff like that. \n\nIs any of this specific to Swiss watches? No, not really. "
]
} | []
| []
| [
[],
[]
]
|
||
80zclg | Is it possible for an astronomical body to have a molten core, without orbiting a star? | Basically i was wondering if a "planet" could have a molten core (and thereby have a theoretical source of energy) without being bound to a star. | askscience | https://www.reddit.com/r/askscience/comments/80zclg/is_it_possible_for_an_astronomical_body_to_have_a/ | {
"a_id": [
"duzgcq5"
],
"score": [
10
],
"text": [
"Sure. If a planet forms and then gets ejected from its star system, there's no reason it couldn't have a molten core for a while. The Earth's core isn't molten because of the Sun, it's molten because of residual heat from planet formation plus radioactive heating from decay within the crust. "
]
} | []
| []
| [
[]
]
|
|
5jiwxf | if working out is just damaging and repairing muscle tissue, couldnt we just mechanically damage the tissue and recouperate saving time and energy? | explainlikeimfive | https://www.reddit.com/r/explainlikeimfive/comments/5jiwxf/eli5_if_working_out_is_just_damaging_and/ | {
"a_id": [
"dbgip62",
"dbgiucu",
"dbgobhr",
"dbgq8jo",
"dbgv6st",
"dbhdyk4",
"dbhm4i2"
],
"score": [
105,
123,
4,
4,
12,
2,
2
],
"text": [
"You can actually build muscle with electro stimulation. By putting electrodes on your skin in the right place the muscles will contract with each pulse of electricity. It's not a pleasant feeling. My roommate had a stimulator for his back muscles when he was injured and to fuck with me he just threw the contacts on me. Fucker, it's unpleasant. Supposedly Bruce Lee use electrostimulation to work his muscles out when he was doing something he had to do that wasn't physical. ",
"Finally something I might have insight on! First off, you can use electric pulses to build and repair damages tissue. I have had terrible knees for years, I started physical therapy at around 17. However, the cartilage damage was so bad in my knee, that I couldn't build the muscle in a healthy way to keep my knee in better traction. Long story short I had surgery. I am 22, and recovering. Because of the nerve and tissue damage I cannot build muscle in my leg through exercise alone. Instead, when I go to PT they hook me up to a machine that sends a small electric charge into my knee, making the muscles contract, nerves reactivate, and swelling in my knee go down. Because of this machine I am able to do my exercises after with little pain. So, regular exercise is needed to build muscle quickly and healthy, but you can use electric shock therapy to help your body out! It is really amazing.",
"A big part of fitness is good heart health. You won't get that from electro stimulation etc. Fitness is called a lifestyle for a reason. It takes a lot of time and is a mix of cardio exercise, muscle exercise, and above all else proper diet. You cannot work yourself out thin. Not all calories are consumed at the same rate despite the undying and factually incorrect tautology that a calorie is a calorie. You aren't a Bunsen burner. You don't process all fats and sugars in the same way. Trying to work myself out thin and buying into a calorie is a calorie really held me back when I first started trying to get into shape. \n\nThere are no short cuts. That's why fitness is universally respected aside from in some groups of contrarians. You can pay to make it easier and you can live the life of leisure that makes it easier to find time, but at the end of the day rich or poor being in good shape represents a large amount of time and effort spent on your fitness. ",
"Depends on what you're thinking of - several of the comments refer to electro stimulation, which could work. If you're thinking 'I'll just cut my muscles and get stronger', that won't work. Your body adapts to the environment it experiences, so if you try it by cutting muscles, you won't get stronger, just a little more resistant to being cut (and this isn't much good either, because while it might make your skin/muscles slightly tougher, a knife can do more damage than you could ever hope to prevent with mere human tissue).",
"I have absolutely no credentials to back what I am saying outside my amateur understanding of fitness and weight lifting, so take what I say with a grain of salt. \n\nAs others have said, it is possible to build muscle with electrical stimulation, but I have to believe there is probably a limit to how accurate or efficient that stimulus can be.\n\nMoreover there is actually a brain body connection that is also trained when weight lifting. When gaining strength as a result of weight lifting some of those gains don't come from additional muscle, but instead from your brain learning how to more efficiently use those muscles. \n\nSo assuming you could force the body to build 50lb of muscle absolutely perfectly, it probably wouldn't be able to use much if any of it.\n\nA probably bad example would be riding a bike. Someone who can't ride a bike probably has all of the physical necessities to do so, but their brain hasn't learned how to use them to actually ride a bike. The same could be said of the artificial muscle growth above. The body might be physically capable of lifting 300lb, but the brain wouldn't know how to utilize the muscles to do so.",
"It's not damaging muscle and repairing it. There's also a lot of cellular signaling involved for nutrient/energy sensing that feeds into a complicated mostly opaque box of anabolic/catabolic programming that takes additional inputs such as immune signaling, DNA synthesis/repair, reactive oxygen sensing, etc. Also hormones and other signaling factor triggered through the brain. Working out is very complex. ",
"In the future, could we just have machines doing this to us and all have six-pack abs and great muscles from getting shocked while laying around watching TV? I hope so!"
]
} | []
| []
| [
[],
[],
[],
[],
[],
[],
[]
]
|
||
juqgy | Chemistry - A few questions about pH buffers. | I've been trying to work out a suitable buffer for my microbe experiment, which has forced me to try and remember my high school chemistry. I understand the basic principal of how a molecule can act as a conjugate base/acid. I've spent a good hour Googling and discussing this with fellow grad-students in my lab, but apparently apart from our own specialised fields, we've forgotten how to tie our shoe laces.
My question is, when we have a mixture as a buffer, e.g. KH2PO4 + K2HPO4 (can't seem to get the subscript going...), what's happening here? Apart from being able to adjust the pH by changing the ratio of the ~~primer~~ buffer (how does that work?), is there any increase in buffer capacity or range?
Thank you.
EDIT: Wow, I typed primer instead of buffer... too much PCR in my blood. | askscience | http://www.reddit.com/r/askscience/comments/juqgy/chemistry_a_few_questions_about_ph_buffers/ | {
"a_id": [
"c2f9q3k",
"c2f9qx0",
"c2f9q3k",
"c2f9qx0"
],
"score": [
2,
3,
2,
3
],
"text": [
"Your monobasic phosphate (KH₂PO₄) will dissociate into H₂PO₄^- , while your dibasic phosphate (K₂HPO₄) dissociates into HPO₄^2- . Your mixture will buffer by going in between these two ions, by adding and removing protons.\n\nChanging the ratio of the primer (by primer I assume you mean the ratio of each of the basic salts when mixing the solution) affects only the pH of your buffer. It's the total molarity of all your phosphate salts that determine the buffer capacity.",
"normally the range is roughly the pKa +/- 1. you can extend the range if you really need to, but i wouldn't recommend it. any more than that and it starts to lose effectiveness. to increase the capacity, use a higher concentration. \n\nthe way it works is that since the two \"versions\" of the molecule exist in equilibrium, when you add acid, the base absorbs the hydrogen ions and removes them from the system. the opposite if you add base: the \"acid version\" becomes protonated. the reason the entire system works is because there are so many more molecules of the weak acid/conjugate base than there are for the acids or bases that you add to the system.",
"Your monobasic phosphate (KH₂PO₄) will dissociate into H₂PO₄^- , while your dibasic phosphate (K₂HPO₄) dissociates into HPO₄^2- . Your mixture will buffer by going in between these two ions, by adding and removing protons.\n\nChanging the ratio of the primer (by primer I assume you mean the ratio of each of the basic salts when mixing the solution) affects only the pH of your buffer. It's the total molarity of all your phosphate salts that determine the buffer capacity.",
"normally the range is roughly the pKa +/- 1. you can extend the range if you really need to, but i wouldn't recommend it. any more than that and it starts to lose effectiveness. to increase the capacity, use a higher concentration. \n\nthe way it works is that since the two \"versions\" of the molecule exist in equilibrium, when you add acid, the base absorbs the hydrogen ions and removes them from the system. the opposite if you add base: the \"acid version\" becomes protonated. the reason the entire system works is because there are so many more molecules of the weak acid/conjugate base than there are for the acids or bases that you add to the system."
]
} | []
| []
| [
[],
[],
[],
[]
]
|
|
2lcmrg | When fat is burned via exercise, which is burned first: the most recently stored fat or the oldest fat in the body? | Say somebody weighs 200 lbs. If they work out and lose a pound, would this be a pound of fat they very recently consumed (while weighing 199 lbs), or is it fat that has been in the body since they became overweight (at ~170 lbs)?
Or maybe all fat is just burned indiscriminately. | askscience | http://www.reddit.com/r/askscience/comments/2lcmrg/when_fat_is_burned_via_exercise_which_is_burned/ | {
"a_id": [
"clusub2"
],
"score": [
3
],
"text": [
"Links, and abstracts to three papers pertinent to the question. Its a bit long, but (I think) very interesting. I have **bolded** a few key statements for those who would just like to skim\n\n_URL_1_\n\"**Fatty acids are the most abundant source of endogenous energy substrate.** They can be mobilized from peripheral adipose tissue and transported via the blood to active muscle. During higher intensity exercise, triglyceride within the muscle can also be hydrolyzed to release fatty acids for subsequent direct oxidation. Control of fatty acid oxidation in exercise can potentially occur via changes in availability, or via changes in the ability of the muscle to oxidize fatty acids. We have performed a series of experiments to distinguish the relative importance of these potential sites of control. The process of lipolysis normally provides free fatty acids (FFA) at a rate in excess of that required to supply resting energy requirements. At the start of low intensity exercise, lipolysis increases further, thereby providing sufficient FFA to provide energy substrates in excess of requirements. However, lipolysis does not increase further as exercise intensity increases, and fatty acid oxidation becomes approximately equal to the total amount of fatty acids available at 65% of VO2 max. When plasma FFA concentration is increased by lipid infusion during exercise at 85% VO2 max, fat oxidation is significantly increased. **Taken together, these observations indicate that fatty acid availability can be a determinant of the rate of their oxidation during exercise. However, even when lipid is infused well in excess of requirements during high-intensity exercise, less than half the energy is derived from fat.** This is because the muscle itself is a major site of control of the rate of fat oxidation during exercise. We have demonstrated that the mechanism of control of fatty acid oxidation in the muscle is the rate of entry into the mitochondria. We hypothesize that the rate of glycolysis is the predominant regulator of the rate of carbohydrate metabolism in muscle, and that a rapid rate of carbohydrate oxidation caused by mobilization of muscle glycogen during high intensity exercise inhibits fatty acid oxidation by limiting transport into the mitochondria. **During low intensity exercise, glycogen breakdown and thus glycolysis is not markedly stimulated, so the increased availability of fatty acids allows their oxidation to serve as the predominant energy source. At higher intensity exercise, stimulation of glycogen breakdown and glycolysis cause increased pyruvate entry into the TCA cycle for oxidation, and as a consequence the inhibition of fatty acid oxidation by limiting their transport into the mitochondria.**\"\n\n_URL_0_ **The rate of carbohydrate utilization during prolonged, strenuous exercise is closely geared to the energy needs of the working muscles. In contrast, fat utilization during exercise is not tightly regulated, as there are no mechanisms for closely matching availability and metabolism of fatty acids to the rate of energy expenditure.** As a result, the rate of fat oxidation during exercise is determined by the availability of fatty acids and the rate of carbohydrate utilization. Blood glucose and muscle glycogen are essential for prolonged strenuous exercise, and exhaustion can result either from development of hypoglycemia or depletion of muscle glycogen. Both absolute and relative (i.e. % of maximal O2 uptake) exercise intensities play important roles in the regulation of substrate metabolism. The absolute work rate determines the total quantity of fuel required, while relative exercise intensity plays a major role in determining the proportions of carbohydrate and fat oxidized by the working muscles. **As relative exercise intensity is increased, there is a decrease in the proportion of the energy requirement derived from fat oxidation and an increase in that provided by carbohydrate oxidation. During moderately strenuous exercise of an intensity that can be maintained for 90 minutes or longer ( approximately 55-75% of VO2max), there is a progressive decline in the proportion of energy derived from muscle glycogen and a progressive increase in plasma fatty acid oxidation.** The adaptations induced by endurance exercise training result in a marked sparing of carbohydrate during exercise, with an increased proportion of the energy being provided by fat oxidation. The mechanisms by which training decreases utilization of blood glucose are not well understood. However, the slower rate of glycogenolysis can be explained on the basis of lower concentrations of inorganic phosphate (Pi) in trained, as compared to untrained, muscles during exercise of the same intensity. The lower Pi level is a consequence of the increase in muscle mitochondria induced by endurance exercise training. A large increase in muscle glycogen concentration, far above the level found in the well-fed sedentary state, occurs in response to carbohydrate feeding following glycogen depleting exercise. It was recently found that this muscle \"glycogen supercompensation\" is markedly enhanced by endurance exercise training that induces an increase in the GLUT4 isoform of the glucose transporter in skeletal muscle.\n\n_URL_2_ **Endogenous triacylglycerols represent an important source of fuel for endurance exercise. Triacylglycerol oxidation increases progressively during exercise; the specific rate is determined by energy requirements of working muscles, fatty acid delivery to muscle mitochondria, and the oxidation of other substrates.** The catecholamine response to exercise increases lipolysis of adipose tissue triacylglycerols and, presumably, intramuscular triacylglycerols. In addition, increases in adipose tissue and muscle blood flow decrease fatty acid reesterification and facilitate the delivery of released fatty acids to skeletal muscle. Alterations in fatty acid mobilization and the relative use of adipose and intramuscular triacylglycerols during exercise depend, in large part, on degree of fitness and exercise intensity. Compared with untrained persons exercising at the same absolute intensity, persons who have undergone endurance training have greater fat oxidation during exercise without increased lipolysis. **Available evidence suggests that the training-induced increase in fat oxidation is due primarily to increased oxidation of non-plasma-derived fatty acids, perhaps from intramuscular triacylglycerol stores. Fat oxidation is lower in high-intensity exercise than in moderate-intensity exercise, in part because of decreased fatty acid delivery to exercising muscles.** Parenteral lipid supplementation during high-intensity exercise increases fat oxidation, but the effect of ingesting long-chain or medium-chain triacylglycerols on substrate metabolism during exercise is less clear. This review discusses the relation between fatty acid mobilization and oxidation during exercise and the effect of endurance training, exercise intensity, and lipid supplementation on these responses."
]
} | []
| []
| [
[
"http://www.ncbi.nlm.nih.gov/pubmed/9740552",
"http://www.ncbi.nlm.nih.gov/pubmed/9781322",
"http://www.ncbi.nlm.nih.gov/pubmed/10919960"
]
]
|
|
5q092t | how do composers write music? | [deleted] | explainlikeimfive | https://www.reddit.com/r/explainlikeimfive/comments/5q092t/eli5_how_do_composers_write_music/ | {
"a_id": [
"dcvb2pt",
"dcvb5nw",
"dcvdwkp"
],
"score": [
16,
2,
4
],
"text": [
"It's all a grand formula called music theory. With Hans Zimmer and most other composers the steps go kind of like this: find a theme, build a central melody and build around the melody. Finding a theme is first step of the creative process and a theme can be deliberately sought after by a composer or it can just come to them. Hans Zimmer wrote the score to the Dark Knight because he was paid to write a Dark Knight soundtrack. Beethoven wrote his fifth symphony because he was contemplating his life after going near deaf. Think of this as the overall spirit or personality of the piece. The next step is developing a melody or motif, this step often includes most of the technical structuring of the piece. This can include time signature, playing directives, key signature and tempo. Think about the iconic Dark Knight theme and how it fits the theme of the movie itself. Or perhaps think about the easily recognizable opening of the Fifth Symphony (short short short long) that reappears throughout the piece. This step is probably the most important step in the composition process and is the heart of the piece. The last step is usually to develop a harmony and 'background'. This includes most of minor instrumentation, bass line and beat. This step is important and it's what separates a good composer from a great one, however it isn't as central to the piece as the first two steps.",
"There are common patterns, some are regional, others are from a certain era of music. \n\nMost pop music today (and for the past 60 years) follows simple progressions of harmonies (also called chords) that are satisfying to the ear. \n\nComposers will employ knowledge of what has worked and blend it with their own tastes and the feeling they're trying to project. \n\nSource: music student\n",
"Music teacher here, who often composes her own music (short ear tests for students) and helps students write arrangements as part of teaching. Your question has various facets, because music is a very broad subject.\n\n > How do composers write music?\n\nOriginally I presumed you meant actual, physical writing on staff paper. I'm old-fashioned and use physical staff paper. Most use software to mix instruments and sounds and write out the notes.\n\n > Is it by \"trial and error\" until something sounds good? Or is there some kind of \"recipe\" they follow to get what they're looking for?\n\nThere's no set method or recipe for when I compose. Sometimes I hear it in my head and write it down. Sometimes I'm playing something, play the wrong notes, and realize it sounds good. Sometimes I need to compose something on a musical idea.\n\nAs for Hans Zimmer's case, I don't know exactly what his process is, but I presume he has a copy of the script or descriptions of what happens in the movie. Based on that and his experience, he composes something to suit each theme (the incidental music on a stormy night, the motif that follows a character, etc.).\n\nThere are a few basic ideas most musicians are aware of, even if they're not specifically instructed on it. Playing in a minor key, for example, is creepy. Progressing upwards in the scale sounds hopeful. 3/4 time sounds elegant (waltzing).\n\n > how does he know what will sound good?\n\nThe short answer: a lot of practice and experience.\n\nThe slightly longer answer: music theory. There's a lot of background knowledge applied behind the final piece you hear. You learn what notes sound good together (chords) and next to each other (progressions), along with how music is built and structured, common types, combinations of instruments, etc. You can learn it by ear without formal training, but only a select few (Mozart) manage that. "
]
} | []
| []
| [
[],
[],
[]
]
|
|
28sfsi | Did Sherman purposely let the Confederate garrison in Savannah escape? | Did Sherman purposely let the Confederate garrision in Savannah escape? Sherman's army reached the outskirts of Savannah by Dec. 10, 1864, and found it defended by a garrison of 10,000 men commanded by Gen. Hardee. Despite a Confederate attempt to stop him, Sherman managed to make contact with the USN by going around Savannah. That enabled the navy to supply him with the artillery that Sherman's army hadn't been able to bring with them on the March to the Sea, as well as ammo, etc. Sherman had the city surrounded, yet on the night of Dec. 20 the entire Confederate garrison managed to escape, a process that included crossing a hastily built pontoon bridge across the Savannah River.
It seems incredible that 10,000 men could escape without Sherman's forces noticing it. Hence there has long been a theory that Sherman chose to let them escape. Other than speculation, is there any historical evidence that Sherman purposely let them escape? If so, why?
| AskHistorians | http://www.reddit.com/r/AskHistorians/comments/28sfsi/did_sherman_purposely_let_the_confederate/ | {
"a_id": [
"cie3rz6"
],
"score": [
5
],
"text": [
"\"Sherman, along with Federal forces in Hilton Head, could have prevented Hardee's escape. Sherman was aware that the Confederates were building a pontoon bridge to South Carolina, yet upon his departure for Hilton Head Island on December 19, he left orders for his army not to attack the Savannah works until he had returned. Sherman's subordinates clearly observed the Confederate evacuation on December 20 yet did nothing to interfere with it. Sherman's victory in Savannah was won through default, not brilliant tactical maneuver.\"\n\nsource: [National Park Service](_URL_0_)\n\nThe article also argues that he was somewhat nervous about his army being divided by a river, which may have stopped him from attacking earlier. \n\nI have not heard of the theory you mention. Where might I find a discussion of it? There is no conceivable reason for Sherman to allow Hardee's men to escape. "
]
} | []
| []
| [
[
"http://www.nps.gov/history/history/online_books/civil_war_series/12/sec5.htm"
]
]
|
|
8xwx0x | Are there more planets or more stars? | Is there a way to find out? Is it even close? | askscience | https://www.reddit.com/r/askscience/comments/8xwx0x/are_there_more_planets_or_more_stars/ | {
"a_id": [
"e26gg86"
],
"score": [
28
],
"text": [
"Based on what the Kepler space telescope has found, there are almost certainly more planets out there than stars.\n\nBefore Kepler it was hypothesized that few enough star systems had planets that there might be more stars than planets, but it turns out there are a lot more planets out there than expected. A few planets have even been found around Neutron stars which to this day has astronomers scratching their heads."
]
} | []
| []
| [
[]
]
|
|
a44r4v | if atoms have a positive nucleus with electrons that orbit it, why don’t they collapse into themselves? | explainlikeimfive | https://www.reddit.com/r/explainlikeimfive/comments/a44r4v/eli5_if_atoms_have_a_positive_nucleus_with/ | {
"a_id": [
"ebbmh8z",
"ebbnpfy",
"ebbo5vz",
"ebbo7mg"
],
"score": [
4,
9,
2,
2
],
"text": [
"There are 4 forces in an atom. Strong, weak, gravitational and electromagnetic. It's the electromagnetic force that's responsible for keeping the electron orbit stable. Think of a rope with a ball on the end and you're holding the other end. Your hand is the nucleus and the ball is the electron, now throw the ball around your hand. If you pull the rope the ball won't collide with your hand, instead, the ball continues to travel around your hand. The rope represents the electromagnetic force. \n\nAnother issue you can think about is that the nucleus has protons in it so how are they held together when they should repel? The strong force acts like glue to hold the protons together. \n\nTL;DR: The electromagnetic force and strong force hold an atom together.\n\n(Correct me if I got anything wrong)",
"There are many levels of understanding this.\nIn the simplest terms, one can think of the electrons as spinning around the nucleus in circles. In the same way as you feel pressed outwards on a carousel, the electrons don’t want to stay on circles but move outwards. This outwards force is cancelled by the attraction between the positive nuclei and negative electrons (opposite charges attract) so the electrons stay on their circular path. In fancy physicist terms, the electric attraction acts as the centripetal force to keep the electrons in their orbits but is not so strong as to make the electrons fall into the nucleus. \nLook up the Rutherford model of the atom for more detail. \n\nIf we want to be a bit more advanced (don’t worry if you don’t understand everything from here on, this is starting to get into university level physics), there is one problem with that description. We know from other fancy physics that when a charged particle moves in a circle, it loses energy and slows down, which would mean that the electron falls into the nucleus after all! The way we solve this problem is by saying that there are a certain group of distances between electron and nucleus where the electrons don’t lose energy after all, and the electrons are only ever allowed to be at these distances (this doesn’t seem like a good answer but it works well with the experiments - and since this idea we have gained a much better although much more complicated understanding of why some distances are special). \nThese different distances have different energies, just as an object has a higher energy when we lift it up (which increases the distance between earth and the object). The electrons can jump between those distances (or ‘levels’ as physicists call them) and can emit or absorb light (they emit light when jumping down and need to absorb light when jumping up to higher energy levels - you can’t get more energy out of nothing). This is what happens in neon lights, the electrons in the atoms jump to levels with lower energies and we see the light they emit. Look up the Bohr model of the atom if you want more details or mathematics. \n\nIf we want to get even more technical, we need some more hardcore quantum mechanics. Basically, the electrons aren’t really like billard balls but more like a diffuse electron mist, or electron cloud. The reason we often think of them as Billard balls is that when this cloud is very bunched up it looks like a solid ball from far away and it is easier to think and calculate with this image (even if it isn’t technically correct). The shape of this cloud that forms the electron (also called the save function of the electron) is determined by a certain equation, the Schrödinger equation (in fact, all matter and not just electrons work like this!). If we calculate the form of this cloud, we find that only certain energies are possible, for which the cloud has different form. And the distance where most of the cloud is turns out to be the Special distance that is allowed for the electron (nice when things work out). And none of these forms have any cloud right at the nucleus. If this last explanation feels a bit like “the maths says so, so it’s true”, that’s because it is - quantum mechanics is really weird and often the only insight one can get is to do the calculations and see that something must be true. ",
"Quantum mechanics is weird and quite counter-intuitive and there isn't really an ELI5 version of the physics behind it, but some explanations help more than others.\n\nWhile the \"solar system\" analogy of electrons as particles orbiting the nucleus in circular orbits persists, it's not really a good explanation of what's really going on at a quantum level.\n\nThe electrons are actually behaving in a more wave-like manner at an atomic level, smeared out over a certain distance - not so much that the electron is physically larger (it's not) - but that its state is described by a quantum mechanical wave function whose interpretation can be somewhat probabilistic.\n\nIn terms of position, the electron can be thought of as *probably* being where the wave function is strongest. In terms of momentum, though? It turns out that the more certain we are about the position (e.g. if we were to say the electron collapsed into the nucleus), we must accept a very high uncertainty of momentum. And as momentum is linked to kinetic energy, the total energy of an electron that is definitely \"too\" close to the nucleus is not only not certain, but almost certainly far larger than an electron that is further out but with a lower kinetic energy.\n\nIf you solve the wave equations, it turns out that the lowest possible energy state is a \"smeared\" cloud, and as you add electrons, they start filling the more complex orbital patterns you see in textbooks (as only one electron can be in the exact same state at once).",
"Gosh there are a lot of different ways to conceptualize this.\n\nThis is actually a pretty historically important question. Ernest Rutherford, the physicist who actually created the model of the atom we learn early in elementary school (Where there is a nucleus that has a positive charge and orbiting electrons with a negative charge) asked this question himself straight away. He personally believed that it was momentum that kept the electrons orbiting the nucleus, just like how the earth is kept in a steady orbit by it's momentum around the sun instead of just falling in.\n\nBut that conceptualization has some flaws. Namely, electrons routinely emit energy in the form of photons, so if it was only momentum keeping them orbiting the nucleus, that momentum should be radiated away in the form of light and the electron should still collapse into the nucleus.\n\nThe problem remained unsolved for a while, and actually helped spark others in the field of what became quantum physics. You see, that model of the atom isn't very accurate it turns out. The electron isn't just a particle in orbit around a nucleus, but rather it's a cloud of probabilities of being at specific places around the nucleus and with specific momentums. Note that the electron still does in fact have a much higher chance of being closer, or even INSIDE the nucleus because of the electromagnetic force!\n\nHowever, here's the thing. Heisenberg discovered that, for various reasons, you can't both know the position and the momentum of an electron. If we constrain the position of an electron to \"closer to the nucleus\", the probability density cloud shrinks in size and, as a result, the momentum of the electron increases. This increased momentum means it has a higher energy, and actually increases the probability density cloud, because it can suddenly \"afford\" to be further away! These two \"forces\" actually sort of balance each other out and create an equilibrium point where the electron is most likely to be that ISN'T actually as close to the nucleus as possible. We call this the bohr radius, and it's where an electron in a hydrogen atom in ground state ends up \"orbiting\" in the traditional model. It's, of course, not so much that the electron orbits at that distance but rather, statistically, it is most likely to be right at that distance.\n\nBut like I said, there are several ways to conceptualize this, and that's just one. You can also learn about it in terms of potential energy vs kinetic energy for instance, but I'll leave that to someone else. "
]
} | []
| []
| [
[],
[],
[],
[]
]
|
||
ax7z5u | Does cold have diminishing returns, e.g. does -40 degrees generally really feel that much colder than zero degrees? | askscience | https://www.reddit.com/r/askscience/comments/ax7z5u/does_cold_have_diminishing_returns_eg_does_40/ | {
"a_id": [
"ehrvfnx",
"ehrwvvc"
],
"score": [
4,
9
],
"text": [
"Yes, it's hard to explain but the wind chill plays a big part in it too, before the polar vortex in Wisconsin a few weeks ago it was -10 or so with no wind and you could go outside in a sweatshirt. But with the wind a week later -10 felt like -50 and it was horrible. You had to wear aparka with a scarf and mittens, when you breathe in it feels like your nose is freezing and your hands get so cold they don't feel like they're moving. ",
"I can tell you first hand from living in North Dakota that base temp is not the end all be all of feeling cold. \n\n The air from these arctic high pressure systems that have had a strangle-hold on us this whole winter is typically very dry and *crisp* if that makes sense. It's a lot different than that damp, wet cold you get on the East Coast in winter. \n\nThis weekend it was sunny, calm, and -15F out. Today, it's snowy, windy, and 3F out, and it feels much colder today than it was this weekend even though the base temp was about 20 degrees colder over the weekend. "
]
} | []
| []
| [
[],
[]
]
|
||
g2dfxm | what makes rent so high in some areas for small apartments, and so low in others for relatively large apartments? | I don't understand how rent prices change so drastically based on the area. Does it relate to taxes? | explainlikeimfive | https://www.reddit.com/r/explainlikeimfive/comments/g2dfxm/eli5_what_makes_rent_so_high_in_some_areas_for/ | {
"a_id": [
"fnkrmtj",
"fnksnrb",
"fnkwdo6"
],
"score": [
6,
4,
3
],
"text": [
"It can be largely based on surrounding areas income but it’s mostly just location. Are you in close proximity to a large employer (like a medical center?), close enough that people wouldn’t *have* to drive? Then it’s gonna be higher, marketed towards medical employees. It’s based on a lot of factors, but for most apartments that aren’t really near any significant employer but are surrounded by general urban sprawl, just check the average income for the surrounding neighborhoods and such. If it’s high, probably in a ‘better’ part of town. Low, ‘worse’ part of town and the rent reflects it",
"Supply and demand. Some people are willing to lay down more cash in congested urban areas, where that same amount would pay for much more of an apartment in rural towns with low populations that aren't as competitive with their rent money.",
"it boils down to: location, supply and demand mostly.\n\nthat tiny place near the beach from which you can walk to the city centre with the cool research lab where you work right next door will probably have more people willing to live there than the large appartement on the other side of the city, somewhere out in the suburbs, next to a sewage plant."
]
} | []
| []
| [
[],
[],
[]
]
|
|
4uh618 | why do bulbs (that glow white) become green and stays illuminated for a while? | explainlikeimfive | https://www.reddit.com/r/explainlikeimfive/comments/4uh618/eli5_why_do_bulbs_that_glow_white_become_green/ | {
"a_id": [
"d5poxxb"
],
"score": [
6
],
"text": [
"I assume you're talking about fluorescent bulbs. They work by having a gas inside which generates intense ultraviolet light when you pass an electric current through it.\n\nThe inside of the glass tubes is coated with phosphor, this is a range of chemicals which glow in different colours when they absorb energy, in this case the mix of phosphors has been chosen to generate white light when uv light pumps energy into it. \n\nWhen you turn off the power the uv generation stops, but the phosphor still has enough energy to glow faintly. The phosphors that glow greenish are most efficient at using the stored energy to glow so they go on longer and the light glows pale green for a while.\n"
]
} | []
| []
| [
[]
]
|
||
21ln84 | can anyone explain if there is any actual valuable take-away from schrödinger's cat? or just an abstract thought experiment? | I would like to clarify that although my question does infer the question "What is Schrödinger's Cat all about?" my question is **What does the cat do to help explain a unique phenomena we otherwise wouldn't know or understand, and what significance there is to all of it?**
I've read a bunch of documents, even saw the simple youtube video below
_URL_0_
but honestly the only way it comes across to me is as **"A tree in the forest that has a fell made a sound and didn't make a sound at the same"**. Is it saying that 50/50 events that aren't observable to people are inconsequential? And if that is correct, why would anyone even care about this analogy? Did people previously think this was neat or something? Very confused and my brain is like I'm 5 :)
Im genuingly curious. Thanks for your help. | explainlikeimfive | http://www.reddit.com/r/explainlikeimfive/comments/21ln84/eli5_can_anyone_explain_if_there_is_any_actual/ | {
"a_id": [
"cge827r",
"cge86z5",
"cge8dmn"
],
"score": [
2,
2,
7
],
"text": [
"Schrodinger's Cat helps explain the idea of superposition in quantum theory -- superposition being the idea that things can be in many states at once, and it's only once they're observed or measured that they take on a specific form (which can be influenced by their observation). It's one of those quirky things about quantum mechanics, but Schrodinger's Cat accurately describes how it works in a way that the layperson can understand.\n\nEdit: Formatting",
"To elaborate on /u/doc_daneeka 's point, the disturbing implication is that, in the box, the cat **is** dead and not dead. Not just it could be either. \n\nHe was in part pointing out that it's easy to say \"particles are weird so they can \"exist and not exist\" and just segregate that from normal experience. But if you tied that \"quantum\" aspect of the particle to something in regular experience, like being alive or dead, then it seemed like madness. ",
"Firstly, the 'answer' to the Schrödinger's Cat thought experiment is not agreed upon. We simply don't have a universal answer to what actually happens. If you read the Wikipedia page, you get a short list of what the different interpretations on quantum mechanics say. It's not like actual physicists have one opinion and outsiders have another. Actual high level physics professors don't agree.\n\nOkay. When quantum mechanics came around it was realized it said really absurd things about tiny particles. You couldn't know whether a particle was here or there. In fact, it's even worse. It seemed like the universe hadn't decided yet. When you actually looked directly at the particle, it suddenly seemed to be only at one place. Einstein was extremely upset and said that he may not know what God thinks, but God surely doesn't play dice with reality(he was talking metaphorically about God).\n\nIt's somewhat okay to accept these weird things about tiny particles. I mean, we can't really see them anyway. But what Schrödinger pointed out was that that's a mistake. He thought, what is true of the small must also be true of the large. And to illustrate this he came up with his cat experiment. If a particle's position can be at different places at the same time, why can't a cat be dead and alive at the same time? His experiment poses exactly that question. What is the physical mechanism that allows weirdness at the small scale, but not the large? \n\nAs I said, this is actually unresolved and there are a number of different possible answers. All of them really absurd.\n\n_URL_0_"
]
} | []
| [
"https://www.youtube.com/watch?v=IOYyCHGWJq4"
]
| [
[],
[],
[
"http://en.wikipedia.org/wiki/Schr%C3%B6dinger's_cat"
]
]
|
|
540bkc | Should the Chinese Emperors' mausoleums be excavated? | Ever since the 60s/70s, the Chinese government has not allowed any of the Emperor mausoleums found to be excavated. This include the tombs of the Ming, Tang and Qin dynasties, including the crown jewel: the mausoleum of the first emperor, due to concerns about preservations and past failures at excavations (such as the catastrophy at the Dinling mausoleum). There have many discussion, for an example see:
_URL_0_
My question to this sub is, do you agree with the Chinese government's policy to not excavate any of these imperial sites? Is this a policy followed by other countries as well? Or do you think this policy is too cautious and excavation should commence? If not, at what point should they be excavated, if ever? How far are we from that point?
And how much do you think these excavations will impact our understanding of Chinese history? | AskHistorians | https://www.reddit.com/r/AskHistorians/comments/540bkc/should_the_chinese_emperors_mausoleums_be/ | {
"a_id": [
"d7y44zv"
],
"score": [
2
],
"text": [
"That's a very hard question to answer and honestly - I am\nNot quite sure if I am qualified for it. Especially not for the imperial tombs in china. \n\nWhat I can answer is what happens if things get done too fast, without a clear preservation plan and without a clear idea of what you want to achieve.\n\nI am from Vienna and I studied art history there - and my dad was an archeologist at the Vienna city museum. \n\nDuring his job he oversaw the excavation of Michaeler Platz (Michaeler square) if you have ever been in Vienna you can't really miss it since it's right in front of the Hofburg.\n\nThis is what it looks like : _URL_0_\n\nSo the square has quite an interesting history , with buildings from roman times , the Middle Ages and the old imperial theater being located there - and when a new subway line was built they excavated part of the square in the course of the construction.\n\nThey found some frescos on the medical buildings - the city wanted to have them displayed in Situ - my dad wanted to cover it up completely or at least transfer the Freskos to a safe spot. \n\nPolitically the city won - and the result is that the frescos are basically gone and the ruins are falling more and more into ruin every year since they are exposed to the weather year round. \n\nAnd the can't really cover it up and keep it visible since this would destroy the sight of the Hofburg from\nThe Kohlmarkt which is a famous view of the Hofburg itself. \n\nSo basically even on a very small excavation, in a very rich city, preservation was not done as it should have been done and the buildings are falling apart more and more.\n\nAnd I think that this is one of the lessons the Chinese learned from a lot of similar projects in Europe - that you need a perfect plan, a perfect idea of what to do and goals and to have the means (both in money and technology) to achieve those goals.\n\nAnd as long as they are not 100% sure that they can conserve whatever they find in the tomb they won't dig it up - the risk is basically to high for them. \n\n\n\n"
]
} | []
| [
"http://www.chinaheritagequarterly.org/articles.php?searchterm=008_qianling.inc&issue=008"
]
| [
[
"http://www.hollein.com/var/ezwebin_site/storage/images/projekte/michaelerplatz-archaeologiefeld/350_michaelerplatz_02.jpg/5436-1-ger-DE/350_MICHAELERPLATZ_02.jpg_projectimage.jpg"
]
]
|
Subsets and Splits
No community queries yet
The top public SQL queries from the community will appear here once available.